You are on page 1of 107

Motion in a Plane

1. Motion in a Plane
BOARD + MHT-CET THEORY
1. Define the term displacement. (2 Marks) ii. Instantaneous velocity:
Ans: Displacement:
i. Displacement of a particle for a time interval is ( )
®
a. The instantaneous velocity v is the limiting
the difference between the position vectors of value of the average velocity of the object
the object in that time interval. over a small time interval (Dt) around t when
ii.
® ®
Let x and x be the position vectors of a particle the value of time interval goes to zero.
1 2
r r
at time t 1 and t 2 respectively. Then the r D x dx
b. v = lim =
® Dt ® 0 Dt dt
displacement s in time Dt = (t 2 - t1 ) is given by
r
® ® ® ® dx r
s = D x = x 2 - x1 where = derivative of x with respect to t.
dt
iii. Average speed:
2. Define the term path length (1 Mark) a. Average speed of an object is the total path
Ans: length (distance) travelled by the object
i. Path length is the actual distance travelled by during the time interval over which average
the particle during its motion. speed is being calculated, divided by that
time interval.
ii. It is a scalar quantity.
iii. Dimensions of path length are equal to that of Total path length
length i.e., [L1M0T0 ] b. Average speed = Totaltimeinterval

iv. Instantaneous speed:


3. Define the terms:
The instantaneous speed is the limiting value of
i. Average velocity (1 Mark) the average speed of the object over a small time
ii. Instantaneous velocity (1 Mark) interval ' Dt ' around t when the value of time
iii. Average speed (1 Mark) interval goes to zero.
iv. Instantaneous speed (1 Mark)
Ans: *4. Define average velocity and instantaneous
i. Average velocity: velocity .when are they same? (2 Marks)

a. Average velocity ( v ) of an object is the


®
av
Ans.: Definition: Average velocity ( v ) of an object
®
av

( )
®
displacement D x of the object during the ( )®
is the displacement D x of the object during
time interval ( Dt ) over which average the time interval ( Dt ) over which average
velocity is being calculated, divided by that velocity is being calculated, divided by that time
time interval. interval.

æ Displacement ö æ Displacement ö
b. Average velocity = ç Time int erval ÷ Average velocity = ç Time int erval ÷
è ø è ø

® ® r ® ® r
® x 2 - x1 D x ® x 2 - x1 D x
v av = = v av = =
t 2 - t1 Dt t 2 - t1 Dt

1
Motion in a Plane
iii. Instantaneous acceleration:
( )
®
Definition: The instantaneous velocity v is the r
The instantaneous acceleration a is the limiting
limiting value of the average velocity of the value of the average acceleration of the object
object over a small time interval (Dt) around t over a small time interval ' Dt ' around t when
when the value of time interval goes to zero. the value of time interval goes to zero.
r r r r
r D x dx r D v dv
v = lim = a inst = lim =
D t ® 0 Dt dt Dt ®0 Dt dt
r
dx r
where = derivative of x with respect to t. *7. If the motion of an object is described by x =
dt
f(t), write the formulae for instantaneous
Reason: In case of uniform rectilinear motion velocity and acceleration (2 Marks)
i.e. when an object is moving with constant Ans.:
velocity along a straight line, the average and
i. Instantaneous velocity of an object is given as,
instantaneous velocity remain same.
r r
r D x dx
v = lim =
5. Distinguish between uniform rectilinear D t ® 0 Dt dt
motion and non-uniform rectilinear motion. ii. Motion of the object is given as,
Ans: (3 Marks) x = f(t)
No. Uniformly Non-uniform iii. The derivative f'(t) represents the rate of
rectilinear motion rectilinear motion change of the position f (t) at time t, which is the
i. The object is moving The object is moving instantanesous velocity of the object.
with constant velocity. with variable velocity r
r dx
ii. The average and The average and v= = f '(t)
\
instantaneous instantaneous dt
velocities are same. velocities are iv. Acceleration is defined as the rate of change of
different. velocity with respect to time.
iii. The average and The average and v. The second derivative of the position function
instantaneous speeds instantaneous speeds
f"(t) represents the rate of change of velocity
are the same. are different.
i.e., acceleration.
iv. The average and The average speed
r r
instantaneous speeds will be different from r D v d2 x
\ a= = = f "(t)
are equal to the the magnitude of Dt dt 2
magnitude of the average velocity.
velocity
8. Draw and explain the position-time graph of:
6. Define the terms: i. An object at rest. (3 Marks)
i. Acceleration (1 Mark) ii. An object moving with uniform velocity along
ii. Average acceleration (1 Mark) positive x-axis. (3 Marks)
iii. Instantaneous acceleration (1 Mark) iii. An object moving with uniform velocity along
Ans: negative x-axis. (3 Marks)
i. Acceleration: iv. An object moving with non-uniform velocity.
Acceleration is the rate of change of velocity (3 Marks)
with respect to time. v. An object performing oscillatory motion with
ii. Average acceleration: constant speed. (3 Marks)
Average acceleration is the change in velocity Ans:
divided by the total time required for the change. i. The position-time graph of an object at rest:
a. For an object at rest, the position-time graph
2
Motion in a Plane
is a horizontal straight line parallel to time
axis.
b. The displacement of the object is zero as there
is no change in the object's position.
c. Slope of the graph is zero, which indicates
that velocity of the particle is zero.

iv. Position-time graph of a particle moving with


non-uniform velocity:
a. When the velocity of an object changes with
time, slope of the graph is different at
different points. Therefore, the average and
instantaneous velocities are different.
b. Average velocity over time interval from t1
ii. The position-time graph of an object moving to t4 around time t0 = slope of line AB.
with uniform velocity along positive x-axis: c. Average velocity over time interval from t2
a. When an object moves, the position of the to t3 = slope of line CD
particle changes with respect to time. d. On further reducing the time interval around
b. Since velocity is constant, displacement is to, it can be deduced that, instantaneous
proportional to elapsed time. interval at to the slope of the tangent PQ at
c. The graph is a straight line with positive slope, t0.
showing that the velocity is along the positive
x-axis.
d. In this case, as the motion is uniform, the
average velocity and instantaneous velocity
are equal at all times.
e. Speed is equal to the magnitude of the
velocity.

v. Position-time graph of an object performing


oscillatory motion with constant speed:
For an object performing oscillatory motion with
constant speed, the direction of velocity changes
from positive to negative and vice versa over
fixed intervals of time.

iii. Position-time graph of an object moving with


uniform velocity along negative x-axis:
a. The graph is a straight line with negative
slope, showing that the velocity is along the
negative x-axis.
b. Displacement decreases with increase in
time.

3
Motion in a Plane
9. Example the velocity-time graphs of an object:
i. Moving with zero acceleration. (3 Marks)
ii. Moving with constant positive acceleration.
(3 Marks)
iii. Moving with constant negative acceleration.
(3 Marks)
iv. Moving with non-uniform acceleration.
(3 Marks)
Ans:
iii. Object is moving with constant negative
i. Object is moving with zero acceleration:
acceleration:
a. The velocity-time graph is linear.
b. Velocity decreases with increase in time as
acceleration is negative (opposite to the
direction of velocities).
c. The area under the velocity-time graph
between two instants of time t1 and t2 gives
the displacement of the object during that
time interval.

Dv
d. Slope of the graph is = negative
Dt
acceleration
a. As the acceleration is zero, the graph will be
a straight line parallel to time axis.
b. Velocity of the particle is constant as the
acceleration is zero.
c. Magnitude of displacement of object from t1
to t2 = v0 ´ (t 2 - t1 ) = shaded area under
velocity-time graph.
ii. Object is moving with constant positive
acceleration:
a. The velocity-time graph is linear.
b. Velocity increases with increase in time, as
acceleration is positive (along the direction iv. Object is moving with non-uniform acceleration:
of velocity)
c. The area under the velocity-time graph
between two instants of time t1 and t2 gives
the displacement of the object during that
time interval.

Dv
d. Slope of the graph is = positive
Dt
acceleration

4
Motion in a Plane
a. Velocity-time graph is non-linear. at = v - u
b. The area under the velocity-time graph
1
between two instants of time t1 and t2 gives \ s = ut + at 2
2
the displacement of the object during that
time interval. This is the second equation of motion.
area under the velocity-time curve = iv. The velocity is increasing linearly with time as
t2 t2 t2
acceleration is constant. The displacement is
dx
Area = ò v dt = ò dt dt = ò dx = x(t 2 ) - x(t1 ) given as,
t1 t1 t1

æv+uö
= displacement of the object from t1 s = vav t = ç ÷t
to t2 è 2 ø

(v + u)(v - u)
*10. Derive equations of motion graphically for a = t
2(v - u)
particle having uniform acceleration, moving
along a straight line? (3 Marks) (v + u)(v - u)
=
Ans.: 2a

\ s = (v 2 - u 2 ) / (2a)

\ v 2 - u 2 = 2as
(v) Q
This is the third equation of motion.
velocity

11. Explain the concept of relative velocity along


P(u) R
a straight line with the help of an example.
(4 Marks)
S Ans:
O t
(0,0) time i. Consider two trains A and B moving on two
parallel tracks in the same direction.
ii. Case 1: Train B overtakes train A.
i. Consider an object starting from position x = 0
For a passenger in train A, train B appears to be
at time t = 0. Let the velocity at time (t = 0) and moving slower than train A. This happens
t be u and v respectively. because the passenger in train A perceives the
ii. The slope of line PQ gives the acceleration. velocity of train B with respect to him/her i.e.,
Thus the difference in the velocities of the two trains
which is much smaller than the velocity of train
v-u v-u A.
\ a= =
t -0 t iii. Case 2: Train A overtakes train B.
\ v = u + at ……(1) For a passenger in train A, train B appears to be
This is the first equation of motion. moving faster than train A. This happens because
iii. The area under the curve in velocity-time graph the passenger in train A perceives the velocity
of the train B w.r.t. to him/her i.e., the difference
gives the displacement of the object.
in the velocities of the two trains which is larger
\ s = area of the quadrilateral OPQS than the velocity of train A.
= area of rectangle OPRS +
® ®
area of triangle PQR. iv. If v A and v B be the velocities of two bodies then
relative velocity of A with respect to B is given
1
= ut + (v - u)t ® ® ®
by v AB = v A - v B .
2
But, from equation (1)
5
Motion in a Plane
v. Similarly the velocity of B with respect to A is ®
given by,
® D x æ x 2 - x1 ö ˆ æ y 2 - y1 ö ˆ
v av = = i+ j
® ® ®
Dt çè t 2 - t1 ÷ø çè t 2 - t1 ÷ø
v BA = v B - v A .
Thus, relative velocity of an object w.r.t. another \ v av = (vav ) x ˆi + (vav ) y ˆj
object is the difference in their velocities
x 2 - x1
vi. If two objects start form the same point at t = 0, Where, (vav ) x = t - t
with different velocities, distance between them 2 1

increases with time in direct proportion to the


relative velocity between them. y 2 - y1
(vav ) y =
t 2 - t1
12. Derive the expression for average velocity and vi. The magnitude and direction are given by,
instantaneous velocity for the motion of an
object in x - y plane. (4 Marks) vy
v= v 2x + v 2y and tan q =
Ans: vx
i. Consider an object to be at point A at time t1 in vii. The instantaneous velocity of the object at point
an x-y plane A along the trajectory is along the tangent to the
curve at A. This is shown by the vector AB.

viii. The instantaneous velocity of the object is given


as,
r r
r æ D x ö dx æ dx ö ˆ æ dy ö ˆ
ii. At time t1 , the position vector of the object is v = lim ç ÷= = ç ÷ i+ç ÷ j
t ® 0 è Dt ø dt è dt ø è dt ø
given as,
ix. The magnitude and direction are given by,
®
x 1 = x1iˆ + y1ˆj .......(1)
2 2
æ dx ö æ dy ö
iii. At time t2, let the position of the object be B and v= v 2x + v 2y = ç ÷ +ç ÷
è dt ø è dt ø
®
its position vector be x 2
æ dy ö
®
vy ç ÷ dy
\ x 2 = x 2ˆi + y 2ˆj .......(2)
tan q = = è dt ø =
v x æ dx ö dx .......(3)
iv. The displacement of the particle from t1 to t2 ç ÷
è dt ø
shown by AB, i.e., in time t = t2 - t1 is given by
r r
Dx = x 2 - x1 = (x 2 - x1 )iˆ + (y 2 - y1 )ˆj Equation (3) is the slope of the tangent to the
curve at the point at which we are calculating
.....[From (1) and (2)] the instantaneous velocity.
v. The average velocity of the object is given as,
6
Motion in a Plane
13. Derive the expression for average acceleration 2 2
and instantaneous acceleration for the motion æ d2 x ö æ d 2 y ö
a= a 2x + a 2y = ç 2 ÷ +ç 2 ÷
of an object in x-y plane. (4 Marks) è dt ø è dt ø
Ans:
i. Consider an object moving in an x-y plane. Let æ dv y ö
ç
ay ÷ dv
= è dt ø=
® ® y
the velocity of the particle be v1 and v 2 at time tan q =
a x æ dv x ö dv x .......(1)
t1 and t2 respectively. ç ÷
è dt ø
ii. The average acceleration ( a ) of the particle
®
av Equation (1) is the slope of the tangent to the
between t1 and t2 is given as, curve at the point at which we are calculating
the instantaneous acceleration.
® ®
® v 2 - v1 æ v 2x - v1x ö ˆ æ v 2y - v1y öˆ
a av = = i +ç ÷j
t 2 - t1 çè t 2 - t1 ø÷ è t 2 - t1 ø
*14. Derive equations of motion for a particle
moving in a plane (with uniform acceleration)
® and show that the motion can be resolved in
\ a av = ( a av ) x iˆ + ( a av )y ˆj two independent motions in mutually
perpendicular directions. (4 Marks)
iii. Thus, the x and y components of the average
acceleration are given by, Ans.:
i. Consider an object moving in an x-y plane.
v 2x - v1x ö æ v 2y - v1y ö r
( a av ) x = æç ÷ and ( a av ) y =ç ÷ Let the initial velocity of the object be u at t = 0
è t 2 - t1 ø è t 2 - t1 ø r
and its velocity at time t be v .
iv. The magnitude and direction of the average
acceleration are given by, ii. As the acceleration is constant, the average
acceleration and the instantaneous acceleration
a av = ( a av )2x + ( a av )2y will be equal.
r r
r v 2 - v1 æ v2x - v1x ö ˆ æ v 2y - v1y öˆ
( a av ) y a av = =ç
t 2 - t1 è t 2 - t1
÷i + ç ÷j
tan q = ø è t 2 - t1 ø
( a av ) x
r r
r v-u( )
v. The instantaneous acceleration ( a ) of
®
the \ a=
(t - 0)
r r r r r
æ D v ö dv
®
\ …..(1)
particle is given by, a = lim ç ÷= v = u + at
t ®0 è Dt ø dt
This is the first equation of motion in vector form
r æ dv ö æ dv y ö ˆ iii. Let the displacement of the object from time
\ a=ç x ÷ ˆi + ç ÷j r
è dt ø è dt ø t = 0 to t be s .
r r
r d æ dx ö d æ dy ö æ d2x ö æ d2y ö r v+u
\ a = ç ÷ iˆ + ç ÷ ˆj = ç 2 ÷ i + ç 2 ÷ ˆj For constant acceleration, vav =
dt è dt ø dt è dt ø è dt ø è dt ø 2
r r r r r
vi. Thus, the x and y components of the r r æ v + u ö æ u + u + at ö
instantaneous acceleration are given by, \ ( )
s = vav t = çç ÷÷ t = çç ÷÷ t
è 2 ø è 2 ø
d2 x d2 y
ax = and a y = r r 1r 2
dt 2 dt 2 \ s = ut + a t ……(2)
2
vii. The magnitude and direction of instantaneous
acceleration are given by, This is the second equation of motion in vector
form.

7
Motion in a Plane
iv. Equations (1) and (2) can be resolved into their 16. Define projectile motion. (1 Mark)
x and y components so as to get corresponding Ans:
scalar equations as follows. i. An object in flight after being thrown with some
vx = u x + ax t …….(3) velocity is called a projectile and its motion is
called projectile motion.
vy = u y + a yt …….(4) ii. Example: A bullet fired from a gun, football
kicked in air, a stone thrown obliquely in air
1
sx = u x t + a x t 2 ……..(5) etc.
2

1 *17. Show that the path of a projectile is a parabola.


sy = u yt + a y t2 ……..(6)
2 Ans: (4 Marks)
v. It can be seen that equations (3) and (5) involve i. Consider a body projected with velocity initial
only the x components of displacement, velocity ®
velocity u , at an angle q of projection from point
and acceleration while equations (4) and (6)
involve only the y components of these O in the co-ordinate system of the XY-plane, as
quantities. shown in figure.
vi. Thus, the motion along the x direction of the
object is completely controlled by the x
components of velocity and acceleration while
that along the y direction is completely
controlled by the y components of these
quantities.
vii. This shows that the two sets of equations are
independent of each other and can be solved
independently.

15. Explain relative velocity between two objects


moving in a plane. (3 Marks) r
ii. The initial velocity u can be resolved into two
Ans:
r r rectangular components:
i. If v A and v B be the velocities of two bodies then
u x = u cos q (Horizontal component)
relative velocity of A with respect to B is given
r ® ® u y = u sin q (Vertical component)
by, v AB = v A - v B .
ii. Similarly, the velocity of B with respect to A is iii. The horizontal component remains constant
throughout the motion due to the absence of any
r ® ®
given by, v BA = v B - v A . force acting in that direction, while the vertical
component changes according to,
iii. Thus, the magnitudes of the two relative
velocities are equal and their directions are vy = u y + a yt
opposite.
iv. For a number of objects A, B, C, D---Y, Z, with a y = -g and u y = u sin q
moving with respect to the other. The velocity iv. Thus, the components of velocity of the
of A relative to Z can be given as, projectile at time t are given by,
r ® ® ® ® ®
v x = u x = u cos q
v AZ = v AB + v BC + v CD + .... + v XY + v Yz
The order of subscripts is: v y = u y - gt = u sin q - gt
A ® B ® C ® D - -- ® Z v. Similarly, the components of displacements of
the projectile in the horizontal and vertical
directions at time t are given by,
8
Motion in a Plane
s x = (u cos q)t .....(1) ®
ii. The initial velocity u can be resolved into two
1 rectangular components:
s y = (u sin q)t - gt 2 .....(2)
2 u x = u cos q (Horizontal component)
vi. As the projectile starts from x = 0, we can use
u y = u sin q (Vertical component)
s x = x and s y = y
iii. Thus, the horizontal component remains constant
Substituting s x = x in equation (1), throughout the motion due to the absence of any
force acting in that direction, while the vertical
x = (u cos q)t
component changes according to, v y = u y + a y t
x
\ t= .....(3) with a y = -g and u y = u sin q
(u cos q)
iv. The components of velocity of the projectile at
Substituting, s y = y in equation (2), time t are given by, v x = u x = u cos q
1 v y = u y - gt = u sin q - gt .....(1)
y = (u sin q)t - gt 2 .....(4)
2
v. At maximum height,
Substituting equation (3) in equation (4), we
have, v y = 0, t = t A = time of ascent = time taken to

2
reach maximum height,
æ x ö 1æ x ö
y = u sin q ç ÷- ç ÷ g \ 0 = u sin q - gt A .....[From (1)]
è u cos q ø 2 è u cos q ø
u sin q = gt A
æ g ö 2
\ y = x(tan q) - ç 2 2 ÷
x .....(5)
è 2u cos q ø u sin q
tA = .....(2)
Equation (5) represents the path of the projectile. g

vii. If we put tan q = A and g/2u 2 cos 2 q = B then This is time of ascent of projectile.
equation (5) can be written as y = Ax - Bx2 vi. The total time in air i.e., time of flight T is given
where A and B are constants. This is equation as,
of parabola. Hence, path of projectile is a T = 2tA .......[From (2)]
parabola.
2u sin q
= .....(3)
18. Obtain an expression for the time of flight of a g
projectile. (4 Marks) Equation (3) represents time of flight of projectile.
Ans: Expression for time of flight:

i. Consider a body projected with velocity u , at


® 19. Define Horizontal range of projectile:
(1 Mark)
an angle q of projection from point O in the co-
Ans: The maximum horizontal distance travelled by
ordinate system of the X-Y plane, as shown in
the projectile is called the horizontal range (R)
figure.
of the projectile.

*20. Derive the formula for the range and


maximum height achieved by a projectile
thrown from the origin with initial velocity u
at an angle q to the horizontal. (4 Marks)
Ans: Expression for range:
®
i. Consider a body projected with velocity u , at

9
Motion in a Plane
an angle q of projection from point O in the particle in this time T is given as,
bavi co-ordinate system of the XY- plane, as R = u x .T
shown in figure.
®
R = u cos q.(2t A )
ii. The initial velocity u can be resolved into two
rectangular components: 2u sin q
\ R = u cos q. .....[From (3)]
g

u 2 (2sin q.cos q)
\ R=
g

u 2 sin 2q
\ R= .... [Q sin 2q = 2sin q.cos q]
g
This is required expression for horizontal range
of the projectile.
Expression for maximum height of a
u x = u cos q (Horizontal component) projectile:
i. The maximum height H reached by the projectile
u y = u sin q (Vertical component) is the distance travelled along the vertical (y)
direction in time tA.
iii. The horizontal component remains constant
throughout the motion due to the absence of any ii. Substituting sy = H and t = ta in equation (1),
force acting in that direction, while the vertical we have,
component changes according to
1
vy = u y + a yt H = (u sin q)t A - gtA 2
2

with a y = -g and u y = u sin q 2


æ u sin q ö 1 æ u sin q ö
\ H = u sin q ç ÷ - gç ÷ ....[From(2)]
iv. Thus, the components of velocity of the è g ø 2 è g ø
projectile at time t are given by,
2
v x = u x = u cos q u 2 sin 2 q u y
\ H= =
2g 2g
v y = u y - gt = u sin q - gt
This equation represents maximum height of
v. Similarly, the components of displacements of projectile.
the projectile in the horizontal and vertical
directions at time t are given by,
21. State the formula for the range achieved by a
s x = (u cos q)t projectile thrown from the origin with initial
velocity u at an angle q to the horizontal. For
1
s y = (u cos q)t - gt 2 ......(1) what angle is the horizontal range maximum?
2
(2 Marks)
vi. At the highest point, the time of ascent of the
Ans: Refer Q.20. form formula for the range achieved
u sin q by the projectile.
projectile is given as, t A = g ......(2)
For maximum horizontal range, sin 2q = 1
vii. The total time in air i.e., time of flight is given \ 2q = 90o
as,
or q = 45o
2u sin q
T = 2t A = .....(3)
g u2
\ R = R max = for q = 45o
viii. The total horizontal distance travelled by the g

10
Motion in a Plane
22. Explain the term uniform circular motion. one revolution or to travel a distance of 2pr .
(3 Marks) Thus, T = 2pr /v
Ans:
Dis tan ce 2pr
i. The motion of a body along the circumference \ Speed, v = = .......(1)
Time T
of a circle with constant speed is called uniform
circular motion. ii. During circular motion of a point object, the
ii. The magnitude of velocity remains constant and position vector of the object from centre of the
its direction is tangential to its circular path. circle is the radius vector ® r . Its magnitude is
iii. The acceleration is of constant magnitude and it radius r and it is directed away from the centre
is perpendicular to the tangential velocity. It is to the particle, i.e., away from the centre of the
always directed towards the centre of the circular circle.
path. This acceleration is called centripetal
iii. As the particle performs UCM, this radius vector
acceleration.
describes equal angles in equal intervals of time.
iv. The centripetal force provides the necessary
iv. The angular speed gives the angle described by
centripetal acceleration.
the radius vector.
v. Examples of U.C.M:
iv. During one complete revolution, the angle
a. Motion of the earth around the sun. described is 2p and the time taken is period T.
b. Motion of the moon around the earth.
Hence, the angular speed w is given as,
c. Revolution of electron around the nucleus
of atom. æ 2pr ö
Angle 2p çè T ÷ø ....[From (1)]
w= = =
23. What is meant by period of revolution of time T r
U.C.M. Obtain an expression for the period vi. The unit of angular speed is radian/second.
of revolution of a particle performing uniform
circular motion. (2 Marks)
25. Derive expression for centripetal an
Ans: The time taken by a particle performing uniform acceleration of a particle performing uniform
circular motion to complete one revolution is circular motion. (4 Marks)
called as period of revolution. It is denoted by
Ans: Expression for centripetal acceleration by
T.
calculus method:
Expression for time period:
i. Suppose a particle is performing U.C.M in
During period T, particle covers a distance equal anticlockwise direction.
to circumference 2pr of circle with uniform
The co-ordinate axes are chosen as shown in
speed v. the figure.
Distancecoveredin onerevolution Let,
\ Time period = Linear velocity A = initial position of the particle which lies on
positive X-axis
2pr P = instantaneous position after time t
\ T=
v
q = angle made by radius vector
w = constant angular speed
24. For a particle performing uniform circular
®
motion, derive an expression for angular r = instantaneous position vector at time t
speed and state its unit. (3 Marks)
ii. From the figure,
Ans:
®
i. Consider an object of mass m, moving with a r = ˆi x + ˆj y
uniform speed v, along a circle of radius r. Let
T be the time period of revolution of the object, where, î and ĵ are unit vectors along X-axis
i.e., the time taken by the object to complete and Y-axis respectively.

11
Motion in a Plane
Negative sign shows that direction of
acceleration is opposite to the direction of
position vector. Equation (3) is the centripetal
acceleration.
vii. Magnitude of centripetal acceleration is given
by,

a = -w2 r
viii. The force providing this acceleration should also
be along the same direction, hence centripetal.
® ® ®
iii. Also, x = r cos q and y = r sin q \ F = m a = - m w2 r
® This is the expression for the centripetal force
\ r = [r iˆ cos q + r ˆj sin q] on a particle undergoing uniform circular
But q = wt motion.

® mv 2
\ r = [r ˆi cos wt + r ˆj sin wt] ......(1) ix. Magnitude of F = mw2 r = = mwv
r
iv. Velocity of the particle is given as rate of change
of position vector.
*26. What is a conical pendulum? Show that its
®
® dr d ˆ l cos q
\ v= = [r i cos wt + r ˆjsin wt] time period is given by 2p where l is
dt dt g
the
éd ù éd ù
= r ê cos wt ú ˆi + r ê sin wt ú ˆj length of the string, q is the angle that the
ë dt û ë dt û
string makes with the vertical and g is the
®
\ v = - rw iˆ sin wt + rw ˆjcos wt acceleration due to gravity. (4 Marks)
Ans.: A simple pendulum, which is given such a
®
motion that the bob describes a horizontal circle
\ v = rw ( -ˆi sin wt + ˆj cos wt)
and the string making a constant angle with the
v. Furture, instantaneous linear acceleration of the vertical describes a cone, is called a conical
particle at instant t is given by, pendulum.
®
® dv d
a= = [rw (-ˆi sin wt + ˆj cos wt)]
dt dt

éd ù
= rw ê (-iˆ sin wt + ˆj cos wt) ú
ë dt û

éd d ù
= rw ê (- sin wt)iˆ + cos wt)ˆjú
ë dt dt û

= rw (-w ˆi cos wt - w ˆjsin wt)


O : rigid support v : velocity of bob
= - rw2 ( iˆ cos wt - ˆjsin wt)
T : tension in the stringr : radius of horizontal
®
l : length of string circle
\ a = -w(r ˆi cos wt + r ˆjsin wt) ......(2)
h : height of support q : semi vertical angle
vi. From equation (1) and (2),
from bob mg: weight of bob
® ®
a = -w2 r ......(3)
12
Motion in a Plane
i. Consider a bob of mass m tied to one end of a Where l is length of the pendulum and h is the
string of length 'l' and other end is fixed to rigid vertical distance of the horizontal circle from the
support. fixed point O.
ii. Let the bob be displaced from its mean a position
and whirled around a horizontal circle PS of IMPORTANT FORMULAE
radius 'r' with constant angular velocity w , then
the bob performs U.C.M. 1. Basic kinematical formulae:
iii. During the motion, string is inclined to the Total path length
vertical at an angle as shown in the figure above. i. Average speed = Total time interval
iv. In the displaced position, there are two forces
acting on the bob. Total distance x
= =
a. The weight mg acting vertically downwards. Total time t
b. The tension T acting upward along the string. ii. Instantaneous speed:
v. The tension (T) acting in the string can be r r
resolved into two components: r D x dx
v = lim =
D t ® 0 Dt dt
a. T cos q acting vertically upwards.
b. T sin q acting horizontally towards centre of iii. Average velocity:
r r r
the circle. r Displacement x 2 - x1 D x
vi. Since, there is no net force, vertical component v avg = = =
Time int erval t 2 - t1 Dt
T cos q balances the weight and horizontal
component T sin q provides the necessary iv. Acceleration:
r
centripetal force. r Change in velocity dv
a= =
\ T cos q = mg .....(1) Time dt
r r r
mv 2 r v 2 - v1 D v
T cos q = = mrw2 ......(2) v. Average acceleration: a av = =
r t 2 - t1 Dt
vii. Dividing equation (2) by (1), r r
r D v dv
vi. Instantaneous acceleration: a inst = lim =
v2 Dt ®0 Dt dt
tan q = ......(3)
rg 2. Kinematic Equations of linear motion:
Therefore, the angle made by the string with the
r r r r r 1r 2
i. v = u + at ii. s = ut + at
-1 æ v ö
2 2
vertical is q = tan ç ÷ r r
è rg ø rr r (v + u)
iii. v 2 = u 2 + 2a.s iv. s = t
2pr 2
viii. Since we know v = 3. Relative velocity of a body A with respect to
T
B:
4p2 r 2 r r r
tan q = .....[From (3)] i. v AB = v A - v B
\ T 2 rg
ii. v = v 2x + v 2y (in magnitude)
r l sin q
T = 2p = 2p 4. Velocity of projectile:
g tan q g tan q
i. u x = u cos q (along horizontal)
..... (Q r = l sin q)
ii. u y = u sin q (along vertical)
l tan q h
T = 2p = 2p ..... (Q h = l cos q) 5. Horizontal distance covered by projectile:
g g
x = (u cos q)t
13
Motion in a Plane
6. Vertical distance of projectile: 14. Angular speed in uniform circular motion:

1 v
y = (u sin q)t - gt 2 w=
2 r
7. Equation of trajectory: 15. Centripetal acceleration:
r r
i. a = -w2 r
gx 2
y = x(tan q) - 2
2u cos2 q ii. a = w2 r (in magnitude)
16. Centripetal force:
u 2 sin 2 q
8. Maximum height : H =
2g mv 2
F = mw2 r = = mwv
r
2u sin q
9. Time of flight: T = g
17. Time period of a conical pendulum:

l cos q h
u sin q T = 2p = 2p
10. Time of ascent = Time of descent = g g g

u 2 sin 2q
11. Horizontal range: R =
g

u2
12. Maximum horizontal range: R max =
g
13. Time period in uniform circular motion:

2pr
T=
v

14
Motion in a Plane

COMPETITION WINDOW
MOTION IN A PLANE

Motion along a straight line projectile motion Circular Motion

MOTION ALONG A STRAIGHT LINE


Introduction
Kinematics
Study of motion of objects without taking into account the factor which cause the motion (i.e. nature of force).
Motion
[ If a body changes its position with time, it is said to be moving else it is at rest. Motion is always relative to the
observer. There is no meaning of rest or motion without the viewer.
[ To locate the position of a particle we need a reference frame. A commonly used reference frame is cartesian
coordinate system or coordinate system. The reference frame is chosen according to problems. If frame is not
mention, then ground is taken as reference frame.
[ The coordinates (x, y, z) of the particle specify the position of the particle with respect to origin of that frame. If
all the three coordinates of the particle remain unchanged as time passes it means the particle is at rest w.r.t. this
frame.
[ If particle moves along a straight line then motin is called one dimensional motion (1-D) or linear motion. In
1-D at least one co-ordinate should change with time.
[ If particle moves along a curved path or a zig-zag path in a particular plane then motion is called two
dimensional motion (2-D). In 2-D at least two co-ordinates must change with time.
[ If particle moves in space such that we can't define motion of plane, then motion is called 3-D. In 3-D all
three co-ordinates must change with time.
DISTANCE AND DISPLACEMENT C
Total length of path covered by the particle, in definite time interval is called
distance whereas the minimum distance between the initial and final points is A B
called displacement.
Let a body moves from A to B via C. The length of path ACB is called the distance travelled by the body and
length of line AB is called displacement of body.
GOLDEN KEY POINTS
• Distance is a scalar while displacement is a vector. Direction of displacement is from initial point to final point.
• Distance depends on path while displacement is independent of path but depends only on final and initial
position.
• For a moving body, distance can't have zero or negative values but displacement may be +ive, –ive or zero.
• If motion is in straight line without change in direction then
distance = |displacement|= (magnitude of displacement)
Otherwise, distance > |displacement|
• Magnitude of displacement may be equal or less than distance but never greater than distance.
distance ³ |displacement|

15
Motion in a Plane

Displacement in terms of position vector Y


Let a body is displaced from A(x1, y1, z1) to B(x2, y2, z2) then its A(x1, y1, z1)
® s
displacement is given by vector AB . rA rB B(x2, y2, z2)
r r r r r r
From DOAB rA + s = rB or s = rB - rA O X
r r
rB = x 2 $i + y 2 $j + z 2 k$ and rA = x1$i + y1$j + z1k$
Z
r
s = (x 2 - x1 )$i + (y 2 - y1 ) $j + (z 2 - z1 )k$
Thus, Displacement = Change in position vector
Ex. A particle goes along a quadrant from A to B of a circle radius 10m as shown in fig. Find the direction and
magnitude of displacement and distance along path shown AB. ^j
® ® ®
Sol. Displacement AB = OB- OA = 10$j - 10$i C
–OA B

®
|AB| = 102 + 102 = 10 2 m

OB
q OB

–O
^i

A
OA 10
From DOBC tanq = = =1 Þ q = 45° O A
OB 10

®
Angle between displacement vector OC and x-axis = 90° + 45° = 135°

1 1
Distance of path AB = (circumference) = (2pR) m = (5p) m
4 4
Ex. A point P consider at contact point of a wheel on ground which rolls on ground without sliping then value of
displacement of point P when wheel completes half of rotation - [If radius of wheel is 1 m]
2
r) P
r
2
) +(2
(p
2r
Sol. Displacement = p2 + 4 P
pr

DO IT YOURSELF
Q.1 A particle starts from the origin, goes along the X-axis to the point (20m, 0) and then returns along the same line
to the point (–20m, 0). Find the distance and displacement of the particle during the trip.
B
Q.2 A person moves on semicircular track of radius 40.0 m. If he starts at one end of the track
d
and reaches at the other end. Find the distance covered and megnitude of displacement of
the person. A

Q.3 A man has to go 50 m due north, 40 m due east and 20 m due south to reach a cafe from his home.
(A) What distance he has to walk to reach the cafe.
(B) What is his displacement from his home to the cafe.

Q.4 A car moves from O to D along the path OABCD shown in fig. 8km A
O
What is distance travelled and net displacement. 4km
D
Ans. (1) 60 m, 20 m and direction along negative x-axis. 1km
r C 4km B
(2) Distance = pR = (40p) m, Displacement d = 2r = 80 m from A to B
16
Motion in a Plane
(3) (A) 110 m
3
(B) 50 m, q = tan–1 = 37° (4) Distance = 17 km, Displacement = 5 km, E 37° S
4
SPEED AND VELOCITY

Speed :- Speed means how fast object is moving. This is the distance travelled by the particle per unit time.

Dis tan ce
Speed = Speed is related to distance thus it is a scalar quantity. Its SI unit is m/s.
Time
Uniform speed : An object is said to be moving with a uniform speed, if it covers equal distances in equal
intervals of time, howsoever small these intervals may be. The uniform speed is shown by straight line in
distance-time graph.
For example, suppose a train travels 1000 m. in 60 s The train is said to be moving with uniform speed,
if it travels 500 m. in 30 s, 250 m. in 15 s, 125 m. in 7.5 s and so on.
Non Uniform speed :
An object is said to be moving with a variable speed if it covers equal distances in unequal intervals of time
or unequal distances in equal intervals of time, howsoever small these intervals may be. In this motion acceleration
is must.
For example, suppose a train travels first 1000 m. in 60 s, next 1000 m. in 120 s. and next 1000 m. in 50
s, then the train is moving with variable speed.
Average speed :- If a particle covers a particular distance in a particular time with variable speed, then average
speed of the particle is equal to that constant speed which covers equal distance in equal time.

Total travelled distance


Average speed of a trip, vav =
Total timetaken

[ Let Ds be the distance travelled in the time interval t to t + Dt. The average speed in this time interval is
Ds
vav =
Dt
[ If a particle moves with speeds v1, v2, v3........... vn in successive time intervals t1, t2, t3 ......tn
then Total distance = v1t1 + v2t2 + v3t3 +.......+ vntn

v1 t1 + v 2 t 2 + ..... + v n t n
Average speed, v av = t1 + t2 + ..... + t n

[ If a particle travels distances s1, s2, s3 etc. with speeds v1, v2, v3 etc. respectively, then total travelled distance s
= s1 + s2 + s3 +.......... + sn

1 s2 3 s sn s No sign is needed for distance or speed.


Total time taken = v + v + v +.....+ v They are always positive quantities.
1 2 3 n

s1 + s 2 + s 3......+s n
Average speed of the trip, vav =
s s s
( 1 + 2 +......+ n )
v1 v2 vn

17
Motion in a Plane
GOLDEN KEY POINTS
• If s1 = s2 = s3 = ..........................= sn

n n 1 1 1
then v av = Þ = + + ....... +
1 1 1 1 v av v1 v 2 vn
+ + + ....... +
v1 v 2 v 3 vn

thus vav = Harmonic mean of speeds


• If t1 = t2 = t3 = ..............= tn

v1 + v 2 + ...... + v n
then , v av = thus vav = Arthmatic mean of speeds
n
Instantaneous speed
The speed at a particular instant is defined as instantaneous speed (or speed) while average speed is defined for
a time interval.

If Dt approaches zero, average speed becomes instantaneous speed. lim Ds = ds


v = Dt®0
Dt dt
i.e. instantaneous speed is the time derivative of distance.
Velocity : Speed of a particle in a particular direction is called velocity thus velocity = speed + direction
displacement
velocity =
time

Uniform Velocity :
A body is said to move with uniform velocity, if it covers equal displacements in equal intervals of time, howsoever,
small these intervals may be.
When a body is moving with uniform velocity, then the magnitude and direction of the velocity of the body
remains same at all points of its path.
Non-uniform Velocity :
The particle is said to have non-uniform motion if it covers unequal displacements in equal intervals of time,
howsoever, small these time intervals may be. In this type of motion velocity does not remain constant.
Average Velocity

For small time interval between t and t + Dt, change in position is D rr then average velocity in Dt time interval is,
r
r Dr
v av =
Dt
r r
Let a particle is at a point A at time t1 and B at time t2. Position vectors of A and B are r1 and r2 . The

r r
®
b g
displacement in this time interval is the vector AB = r2 - r1 . The average velocity in this time interval is,
Y
r displacement vector A
v av =
time int erval
r1
r2 B
® r r O X
r AB r -r
v av = = 2 1
t 2 - t1 t 2 - t1 Z

18
Motion in a Plane

® ® ® r r
here AB = OB - OA = r2 - r1 = change in position vector..
x2 B

Displacement
From fig., the average velocity between points A and B is Dx
A q C
x1
®
x 2 - x1 Dx Dt
v av = =
t2 - t1 Dt t1 t2
Time
Graph (A)
Dx
If time interval is small i.e. t2 – t1 = Dt and x 2 - x 1 = D x , then Vav = = tan q from graph (A)
Dt

Average velocity is equal to slope of straight line joining two points on displacement time graph.
Instantaneous velocity
The velocity of the object at a given instant of time or at a given position during motion is called instantaneous
velocity. Velocity is the rate of change of position vector or change in position per unit time.
If Dt ® 0, then average velocity becomes instantaneous velocity instantaneous velocity,

®
Lt D x = d x x2 B

Displacement
|v|= or instantaneous velocity at point P,
Dt ® 0 Dt dt Point P x
A C
x1
® Dtt
D
| v | = tana (slope of tangent at point P, graph B) t1 t2
Time
Graph (B)
r r
If particle moves in x-y plane such that, r = xiˆ + yjˆ then vr = dx ˆi + dy ˆj , v = v x ˆi + v yˆj
dt dt

r
|v|= v 2x + v 2y and tanq = vy/vx where q = angle of instantaneous velocity with x-axis.

GOLDEN KEY POINTS


• Magnitude of instantaneous velocity is the instantaneous speed.
• Speed is related to distance and it is a scalar while velocity is related to displacement and it is a vector.
• For a moving body speed can't have zero or negative values but velocity can have.
• When a particle moves with constant velocity, its average velocity, its instantaneous velocity and its speed all
are equal.
• Average speed and magnitude of average velocity are not equal.
• Uniform speed may be equal to uniform velocity in a linear motion because velocity = speed + direction.
• Moving body with uniform speed may have variable velocity. e.g. in uniform circular motion speed is constant
but velocity is non-uniform due to change in direction.

19
Motion in a Plane

Ex. If a particle travels the first half distance with speed v1 and second half distance with speed v2.Find its average
speed during journey.

A s O s B
s+s 2s 2v1v 2 v1 v2
Sol. vavg. = = =
t1 + t 2 s
+
s v1 + v 2 t1 = sv1 t 2= sv2
v1 v 2

Ex. If a particle travels with speed v1 during first half time interval and with v2 speed during second half time interval.
Find its average speed during its journey. s1 s2

Sol. s1 = v1t and s2 = v2t, Total distance = s1 + s2 = (v1 + v2)t t t


A v1 v2 B
O
total time = t + t = 2t vavg. =
s1 + s 2
=
b =
g
v1 + v 2 t v1 + v 2
t+t 2t 2
Note :- here vavg is arithmatic mean of two speeds.
Ex. A car travels a distance A to B at a speed of 40 km/h and returns to A at a speed of 30 km/h.
(i) What is the average speed for the whole journey? (ii) What is the average velocity?
s s
Sol. (i) Let AB = s, time taken to go from A to B, t1 = and time taken to go from B to A, t2 =
40 30

s s (3 + 4) s 7s
\ total time taken = t1 + t2 = + = = Total distance travelled = s + s = 2s
40 30 120 120

total distance travelled 2s 120 ´ 2


\ Average speed = = = = 34.3 km/h.
total time taken 7s 7
120
(ii) Total displacement = zero, since the car returns to the original position.
total ´ displacement 0
Therefore, average velocity = = = 0
time taken 2t

2v1 v 2 2 ´ 40 ´ 30
Alternate method :- v = = = 34.3km / h
v1 + v 2 40 + 30
Ex. A man walks on a straight r oad from his home to a market 2.5 km a way with a speed of
5 km/h. Finding the market closed, he instantly turns and walks back with a speed of 7.5 km/h.
What is the (a) magnitude of average velocity and (b) average speed of the man, over the interval of time
(i) 0 to 30 min. (ii) 0 to 50 min (iii) 0 to 40 min.
distance 2.5 1
Sol. Time taken by man to go from his home to market, t1 = = = h
speed 5 2

2.5 1
Time taken by man to go from market to his home, t 2 = = h
7.5 3

1 1 5
\ Total time taken = t1 + t2 = + = h = 50 min.
2 3 6

® displacement distance
(a) Average velocity v av = time
(b) Average speed vav=
time

20
Motion in a Plane

® 2.5 2.5
(i) 0 to 30 min v av = = 5 km/h towards market v av = = 5 km / h
1 1
2 2
(ii) 0 to 50 min Total distance travelled = 2.5 + 2.5 = 5 km.
®
5
Total displacement = zero so v av. = 0 v av = = 6 km / h
5/6
(iii) 0 to 40 min Distance moved in 30 min (from home to market) = 2.5 km.
10
Distance moved in 10 min (from market to home) with speed 7.5 km/h = 7.5 × = 1.25 km
60
So displacement = 2.5 – 1.25 = 1.25 km (towards market)
Distance travelled = 2.5 + 1.25 = 3.75 km

® 1.25 3.75
v av = v av =
40 40
60 60
= 1.875 km/h. (towards market) = 5.625 km/h.
DO IT YOURSELF
Q.1 A man walks for 1 minute at a speed of 1m/s and then runs for 2 min. at 3m/s. What is the average speed of the
man ?
Q.2 A bird flies due north at velocity 20 ms–1 for 15 s it rests for 5 s and then flies due south at velocity 24ms –1 for 10s.
For the whole trip find the average speed and magnitude of average velocity.
Q.3 A table clock has its minute hand 4.5 cm long. Find the average velocity of the tip of minute hand
(a) between 12.00 a.m. to 12.30 a.m. and
(b) between 12.00 a.m. to 12.30 p.m.
both have directions 12 mark to 6 mark on the clock panel.
Q.4 The distance travelled by a particle in time t is given by s = (2.5 m/s2) t2. Find (a) the average speed of the
particle during the time 0 to 5.0s and (b) the instantaneous speed at t = 5.0 s.
Q.5 In 1 second a particle goes from point A to point B, moving in a semicircle
1m

of radius 1m. Find the magnitude of the average velocity. A O B

Q.6 Straight distance between a hotel and a railway station is 10 km, but circular route is followed by a taxi covering
23 km in 28 minute. What is average speed and magnitude of average velocity, Are they equal ?
Ans. (1) 2.3 m/s (2)18 ms–1, 2 ms–1
(3) (5 ´ 10–3 cm/s) , 2 ´ 10–4 cm/s (4) (a) 12.5 m/s (b) 25 m/s
(5) 2 m s–1 (6) 49.3 kmh–1, 21.4 kmh-1
RELATIVE MOTION
There is no meaning of motion without reference or observer. If reference is not mention then we take the
ground as a reference of motion. Velocity or displacement of the particle w.r.t. ground is called actual velocity or
actual displacement of the body. If we describe the motion of a particle w.r.t. a object which is also moving w.r.t.

21
Motion in a Plane
r
ground then velocity of particle w.r.t. ground is its actual velocity ( v act ) and velocity of particle w.r.t. moving
r
b g
object is its relative velocity v rel. and the velocity of moving object (w.r.t. ground) is the reference velocity

bvr g
ref. S S'
P
r ps
In the figure let S is ground frame and S' is frame of moving object.
rps'
r
Position of particle P relative to frame S is rPS while position of frame S' relative
r r r r rs's
to frame S is rS 'S at a moment. According to vector D law rPS = rPS' + rS'S

differentiate the eq. w.r.t. time,


r r r r
drPS drPS' drS' S r dr
= + [ but v = ]
dt dt dt dt
r r r
So that v PS = v PS ' + v S 'S
r r r r r r
i.e. v actual = v relative + v reference or v rel = v act - v ref

Ex. Two buses A and B are moving in the same direction with speed 20 m/s and 15 m/s respectively. Find the relative
velocity of A w.r.t. B and relative velocity of B w.r.t A.
r r
b g
Sol. Let their direction of motion of bus A is $i then v A = 20m / s $i and v B = 15m / s $i b g
r r r
(a) Relative velocity of A w.r.t. B = v AB = v A - v B = (actual velocity of A) – (velocity of B)

= (20 m/s) $i – (15 m/s ) $i = 5m/s $i

i.e. A is moving with speed 5 m/s w.r.t B in $i direction.


r r r
(b) Relative velocity of B w.r.t. A = v BA = v B - v A = (Actual velocity of B) – (velocity of A)

= (15 m/s) $i – (20 m/s) $i = (–5m/s) $i = (5 m/s) (– $i )

i.e. B moving in – $i direction w.r.t. A, at speed 5 m/s

Ex. A person moves due east at speed 6m/s and feels the wind is blowing to south at speed 6m/s.
(a) Find actual velocity of wind blow.
(b) If person doubles his velocity then N
find the relative velocity of wind blow w.r.t. man.
r r r r r r ^
vm = 6 i
Sol. (a) v act = v rel + v ref v w = v wm + v m = -6$j + 6$i W E
45°
r
v w = 6$i - 6$j
^
vwm = -6 i vw
r S
|v|= 6 2 m/s and it blowing to S – E

r N
(ii) Person doubles its velocity then v m = 12$i
but actual wind velocity remain unchanged.
–6 ^i
r r r W E
v wm = v w - v m = (6$i - 6$j ) - 12$i 45°

r v wm
v wm = -6$i - 6$j
–6 ^j S
22
Motion in a Plane

Now relative velocity of wind is 6 2m/s to S – W..

Ex. In a reference frame a man A is moving with velocity (3$i - 4$j ) ms–1 and another man B is moving with velocity

($i + $j ) ms–1 relative to A. Find the actual velocity of B.


r r r
Sol. v actual = v rel + v ref = [($i + $j ) + (3$i - 4 $j )] ms–1 = (4$i - 3$j ) ms–1

RELATIVE VELOCITY OF RAIN W.R.T. THE MOVING MAN :


®
A man walking towards west with velocity v®m , represented by OA . Let the rain be falling vertically downwards
®
with velocity vr , represented by OB as shown in figure. The relative velocity of rain w.r.t. man v rm = v r - v m ,
® ® ®

®
will be represented by diagonal OD of rectangle OBDC.

\ vrm = v 2r + v 2m + 2v r v m cos 90° = v 2r + v 2m

q
If q is the angle which ®
v rm makes A vm O –vm C
W E
q
with the vertical direction then
vr vrm

BD vm æ vm ö
tan q = = vr
or q = tan–1 ç v ÷ D
OB è r ø B

Here angle q is from vertical towards west and is written as q, west of vertical.
Note : In the above case if the man wants to protect himself from the rain, he should hold his umbrella
in the direction of relative velocity of rain w.r.t. man i.e. the umbrella should be hold making an angle q

æ -1 v m ö
ç = tan ÷
è v r ø west of vertical.

Ex. A man is moving due east with a speed 1 km/hr and rain is falling vertically with a speed 3 km/hr. At what
angle from vertical the man has to hold his umbrella to keep the rain away. Also find the speed of rain drops w.r.t.
man.

1
Sol. tanq = , q = 30° vm
3
eastward or ^i
r r
v M = (1km hr -1 )$i v R = ( 3 kmhr -1 )(- $j ) v RM q
vR
r r r r – vm downward or -i^
v RM = v R - v M = (- 3 $j - $i ) |v RM|= ( 3 )2 + 12 = 2 km hr
–1

Angle of relative velocity with vertical is 30° so that man has to tilt his umbrella at 30° with vertical towards the
east.
Ex. A girl standing on a road has to hold her umbrella at 30° with the vertical to keep the rain away. She throws
the umbrella and starts runing at 10 km/h. She finds that raindrops are hitting her head vertically. Find the
speed of raindrops with respect to (a) the road (b) the moving girl.
Sol. Suppose the velocity of rain with respect to girl = vRG

23
Motion in a Plane
The velocity of rain with resp ect to the ground = v Rg

30 °
The velocity of girl with respect to ground = vGg = 10 km/h
® ® ® ® ® ®
v RG = v R - v G .........(i) v Rg = v R - v g .........(ii)

30°
® ® ®
v Gg = v G - v g .........(iii) O

® ® ® ® ® v Rg
vRG

30
On adding eqn. (i) and eqn. (iii) v RG + v Gg = v R - v g = v Rg

°
A B
AB 10 10 10 vGg
v Rg = = = 20 km / hr
(a) By triangle AOB – sin 30° = OB = v ; sin 30° 1
Rg 2

v RG
(b) Now, taking – = cot 30° Þ v RG = 3 or v RG = 10 3 km / h
v Gg 10

DO IT YOURSELF
Q.1 Two cars are moving in the same direction with same speed 30 km/hr. They are separated by a distance of
5 km. What is the speed of a car moving in the opposite direction if it meets these two cars at an interval of
4 minutes.
Q.2 A police van moving on a high way with a speed of 30 km/hr. fires a bullet at a thief car which is speeding away
in the same direction with a speed of 190 km/hr. If the muzzle speed of the bullet is 150 m/s, find speed of the
bullet the theif's car will Observe.
Q.3 Delhi is at a distance of 200 km. from Ambala. Car A set out from Ambala at a speed of 30 km/hr. and car B set
out at the same time from Delhi at a speed of 20 km/hr. When they will meet each other ? What is the distance
of that meeting point from Ambala?
Q.4 A man 'A' moves in north direction with speed 10 m/s and another man B moves in
E–30°–N with 10 m/s. Find the relative velocity of B w.r.t. A.
Q.5 A and B are moving with the same speed 10 m/s in the direction E–30°–N and E–30°–S respectively. Find the
relative velocity of A w.r.t. B.
Q.6 Two body A and B are 10 km apart such that B is in south of A. A and B start moving with same speed
20 km/hr eastward and northward respectively then find. N
(a) Relative velocity of A w.r.t. B.
W E
(b) Minimum separation attained during motion
(c) Time lapse, from starting to attain minimum separation. S
Ans. (1) 45 km/hr. (2) 380 km/hr. (3) After 4 hr. at 120 km from the Ambala

km 1
(4) 5 3 $i - 5 $j , E– 30° - S (5) In north direction 10m/s (6) 20 2 B, 10 ´ km, 15 min.
hr 2

SWIMMING INTO THE RIVER


r
A man can swim with velocity v , i.e. it is the velocity of man w.r.t. still water..
r r r r
If water is also flowing with velocity v R then velocity of man relative to ground vm = v + vR
(i) If the swimming is in the direction of flow of water or along the downstream then

v
vm = v + vR vR

(ii) If the swimming is in the direction opposite to the flow of water or along the upstream then
24
Motion in a Plane

v
vm = v - vR vR

r r
(iii) If man is crossing the river i.e. v and v R not collinear then use the vector algebra

vR
r r r
vm = v + vR
v vm
Crosing the river by shortest path :
If man wants to cross the river such that his "displacement B
should be minimum" it means he wants to reach just opposite point across
the river. vR
d vm
man should start swimming at an angle q with the perpendicular to the
v q vcosq= vm
flow of river towards upstream.
r r r vsinq= vR A
So that its resultant velocity v m = (v + v R ) . It is in the direction of
(for minimum displacement)
displacement AB.

vR
To reach at B v sin q = vR Þ sin q =
v

d d
component of velocity of along AB is = vcos q so time taken T = =
v cos q 2 2
v - vR

Crossing the river in minimum time


B C
To cross the river in minimum time the velocity along
vR
AB (v cos q) should be maximum. d
v vm
q
It is possible if q = 0, i.e. swimming should start perpendicular to water current.
Due to effect of river velocity man will reach at point C along resultant velocity, A
(for minimum time)
i.e. his displacement will not be minimum but time taken to cross the river will
be minimum,

d
t min =
v

In time tmin swimmer travels distance BC along the river with speed of river vR \ BC = tmin vR

d
distance travelled along river flow = drift of man = tmin vR = vR
v

Ex. A river 400 m wide is flowing at a rate of 2.0 ms–1. A boat is sailing at a speed of 10ms–1 w.r.t. water, in a direction
perpendicular to the river.
(a) Find the time taken by the boat to reach the opposite bank.
(b) How far from the point directly opposite to the starting point does the boat reach the opposite bank.
25
Motion in a Plane
B C
AB 400
Sol. (a) t = = s = 40s vR
vb 10 d vb vm
q
(b) Drifting of boat along the river = BC = t ´ vR = 40 ´ 2 m = 80m A

Ex. A boat moves along the flow of river between two fixed points A and B. It takes t 1 time when going downstream
and takes t2 time when going upstream between these two points. What time it will take in still water to cover the
distance equal to AB.

AB AB AB AB
Sol. t1 = , t2 = or vb + vR = and vb – vR =
vb + vR vb - v R t1 t2

Ft +t I F 2t t I = AB = time taken by the boat to cover AB


Þ 2vb =
AB
t1
+
AB
t2
= AB GH t t JK
1

1 2
2
or GH t + t JK v
1 2

1 2 b

Ex. A man can swim in still water at a speed of 3 km/hr. He wants to, cross a 500 m wide river flowing at
2 km/hour. He keeps himself always at an angle of 1200 with the river flow while swimming.
(a) Find the time he takes to cross the river. (b) At what point on the opposite bank will he arrive.
B C
1
Sol. Width of river AB = d = 500 m = km. Y
2

d vm v
vm = 3 km/hr. Velocity of man in still water
q X
30

vr = 2 km/hr. Velocity of river


°

120°
v = resultant velocity of man in flowing river. A vr

r 1 1 vy v
v = v x ˆi + v y ˆj Now, vx = vr – vm sin30° Þ 2 - 3 × = km/hr
2 2

3 3 q
vy = vm cos30° = km/hr Displacement along Y-axis, d = vy × t
2
vx

d 1/2 1 1 1 1
or t = = , t= hr Displacement along X-axis, BC = vx × t = × = km
v y 3 3/2 3 3 2 3 3 6 3

Q. A man can swim at a speed 2ms–1 in still water. He starts swimming in a river at an angle 150° with direction of flow
of water and reaches at the exactly opposite point on the opposite bank.
(a) Find the speed of flowing water.

(b) If width of river is 1 km then calculate the time taken also. Ans. (a) 3 ms–1, (b) 1000s.

26
Motion in a Plane
ACCELERATION
Uniform and non-uniform motion
If the velocity of particle remains constant with time, it is called uniform motion or motion with uniform velocity.
If the velocity changes with time, it is said to be accelerated or non-uniform motion.
Acceleration
The acceleration is rate of change of velocity or change in velocity per unit time interval.
Velocity is a vector quantity hence a change in its magnitude or in direction or in both, will change the velocity
(or non uniform motion)
Uniform acceleration :
An object is said to be moving with a uniform acceleration if its velocity changes by equal amounts in equal
intervals of time.
Variable acceleration :
An object is said to be moving with a variable acceleration if its velocity changes by unequal amounts in
equal intervals of time.
Average Acceleration :
When an object is moving with a variable acceleration, then the average acceleration of the object for the
given motion is defined as the ratio of the total change in velocity of the object during motion to the total
time taken i.e.,

total change in velocity v


Average Acceleration = B
total time taken

® ®
Suppose the velocity of a particle is v 1 at time t1 and v 2 at time t2 . q
A C
t
® ® ®
Then, Change in velocity = v 2 - v1 = Dv

® ® ®
®
v 2 - v1
Elapsed time in changing the velocity = t2 – t1 = Dt Thus , a av = = Dv
t 2 - t1 Dt

®
BC
Þ a av = = tan q = the slope of chord of v – t graph is average acceleration.
AC
Ex. An athlete takes 2 second to reach the maximum speed of 18 km/h from rest. What is the magnitude of his
average acc.?
Sol. Here, Initial velocity u = 0,

5
v = (vmax) = 18 km/h = 18 ´ = 5 m/s , t1 = 0 s, t2 = 2 s
18

v -u Dv 5.0
aav = t - t = so a av = = 2.5 m/s2
2 1 Dt 2

Ex. A car starts from rest and acquires velocity equal to 10 m/s after 5 sec. Find the acceleration of the car.
Sol. Here, u = 0 and v = 10 m/s , t = 5 s

27
Motion in a Plane

v-u (10 - 0)m / s


Using , a = , we have a = = 2 m/s2
t 5 s
Ex. A car moving with a velocity of 20 ms–1 is brought to rest in 5 seconds by applying brakes. Calculate the
retardation of the car.

v-u
Sol. Here, u = 20 ms–1 , v = 0, t = 5 sec. Using a = ,
t

(0 - 20)
we have a = = – 4 m/s2
5
–ve acceleration is known as retardation. Thus, retardation of the car = 4 ms–2 .

Ex. Intial velocity of a car in 5 m/s due east. After 10 sec. velocity of car becomes 5 m/s due north. Calculate
magnitude and dirrection of averge occ. of car. ®
v–v
®
®
2 1
v2
r r r r r
Sol. v1 = 5iˆ ; v 2 = 5jˆ charge in velocity, Dv = v 2 - v1
r
= -5iˆ + 5ˆj
®
|Dv|= 5 2 m / s -v1
®
v1

r
r | Dv| 5 2 1
Dt = 10 s thus |aav |= = = m / s2
Dt 10 2
Direction of averge acc. is same as direction of change in velocity. Thus it is N-W

Instantaneous Acceleration :
The acceleration of the object at a given instant of time or at a given point of motion, is called its instantaneous
acceleration.
® ®
Suppose the velocit y of a particle at time t 1 = t is v 1 = v and becomes ®
v 2 = v + Dv at time
® ®

®
®
Dv
t2 = t + Dt, Then, a av = B
Dt v2

If Dt approaches to zero then the rate of change of velocity Dv


v1 A q
will be instantaneous acceleration. Instantaneous acceleration
Dt
C
r r t1 t2
r Dv dv
a inst = Lim = Time
D t ® 0 Dt dt

Instantaneous acceleration at a point is equal to slope of tangent at that point on velocity time graph in the
graph shown above this point is A.

æ ®ö ®
® ® d çdx÷ d2 x
® dx a= =
v= , therefore dt çç dt ÷÷ dt2
dt è ø

Thus, instantaneous acceleration of an object is equal to the second time derivative of the position of the
object at the given instant.

Ex. The velocity of a particle moving in the x direction varies as v = a x where a is a constant. Assuming that at
the moment t = 0 the particle was located at the point x = 0. Find the acceleration.

28
Motion in a Plane
dv d 1 dx 1 a2
Sol. a= =a x = a . x -1/2 = a. .a x Þ a=
dt dt 2 dt 2 x 2

Ex. The velocity of any particle is related with its displacement As; x = v + 1 , Calculate acceleration at
x = 5 m.

Sol. x=
2 2
( )
v + 1, x = v + 1, v = x - 1 Therefore a = dv = d x2 - 1 = 2x dx = 2x v = 2x x 2 - 1
( ) ( )
dt dt dt

a = 2 × 5 (25 - 1 ) = 240 m/s


2
at x = 5 m,

Ex. Initial velocity of a particle is v0 and its acceleration is a = kt , where k is a constant. calculate velocity and
displacement of particle at time t.

dv ds k k
Sol. (i) Given that , a = kt Þ = kt (ii) v = = v 0 + t2 Þ ds = v 0 + t2 )dt
dt dt 2 2

v t s t k t 2
Þ dv = k t dt Þ ò v0
dv = k ò t dt
0
Þ ò0 ds = ò0 v 0dt + 2 ò0 t dt

k 2 k t3 k 3
Þ v – v0 = k.t2/2 Þ v = v0 + t Þ S = v0t + Þ S = v0 t + t
2 23 6

GRAPHICAL METHOD :
1. Displacement time graph : - In displacement time graph, slope of straight line or tangent at any point is equal
to velocity of particle :
S S S
st. q2
c on q2
v=
q1
(a) q (b) (c)
t q1 t t

v = tanq = constant Q q1 < q2 \ tanq1 < tanq2 Q q1 > q2 \ tan q1 > tanq2
v 1 < v 2 velocity of particle increases v1 > v 2 velocity of particle
with time. decreases with time.
Thus motion is acceleration Thus motion is retardation
2. Velocity time graph : - v
(a) In velocity time graph. Slope of straight line is equal to acceleration of particle.
v max v max vmax
a1 = tan q1 = a2 = tan( p - q2 ) = - tan q2 = - a2
t1 t2 a1
q1 q2
Acceleration of particle = vmax/t1 t
Retardation of particle = vmax/t2 t1 t2

(b) In velocity time graph area bounded by the curve and time axis is equal v
to displacement
S1
S1 = disp. in forward direction
S2
t
S2 = disp. in backward direction
thus Total displacement = S1 – S2
29
Motion in a Plane
a
Total distance = S1 + S2
3. Acceleration time graph - In a-t graph, area bounded by the
curve and time axis is equal to change in velocity (Dv) of the particle
Dv
during that time interval.
t

GOLDEN KEY POINTS


• Slopes of v-t or s-t graphs can never be infinite at any point, because infinite slope of v-t graph means infinite
acceleration. Similarly, infinite slope of s-t graph means infinite velocity. Hence, the following graphs are not possible.
v s

t v
• At one time, two values of velocity or displacement are not possible Hence, the following graphs are not acceptable.
v v s
v1 v1 s1

v2 v2 s2

t0
t t0
t
t0 t

Ex. A car accelerates from rest at a constant rate a for some time, after which it decelerates at a constant rate b, to come
to rest. If the total time elapsed is t evaluate (a) the maximum velocity attained and (b) the total distance travelled.
Sol. (a) Let the car accelerates for time t1 and decelerates for time t2 then
t = t1 + t2 ......(i) vmax A
and corresponding velocity-time graph will be as shown in. figure
B
O t1 t2 t
v max v max
From the graph a = slope of line AB = t or t1 = .....(ii)
1 a

v max v max
and b = – slope of line OB = t or t2 = ....(iii)
2 b

v max v max æa + bö ab t
From Eqs. (i),(ii) and (iii) + = t or vmax ç ÷=t or vmax =
a b è ab ø a +b

1 æ abt ö
2
1 1 ab t
(b) Total distance = area under v-t graph = × t × vmax = ×t× = ç ÷
2 2 a +b 2 è a +b ø

Note: This problem can also be solved by using equations of motion (v = u + at, etc.).

s(m)
Ex. Velocity-time graph of a particle moving in a straight line is shown.
Plot the corresponding displacement-time graph of the particle. 20

Sol. Displacement = area under velocity-time graph. 10

t(s)
O 2 4 6 8
30
Motion in a Plane

1
Hence, s OA = × 2 × 10 = 10 m, sAB = 2 × 10 = 20 m
2

æ 10 + 20 ö s(m)
or s OAB = 10+20 = 30 m, sBC = 2 ´ ç ÷ = 30 m 80
è 2 ø
60

or s OABC = 30+30 = 60 m, 40
20
t(s)
æ 20 + 0 ö O 2 4 6 8
and sCD = 2 ´ ç ÷ = 20m or sOABCD= 60 + 20 = 80 m
è 2 ø

Between 0 to 2 s and 4 to 6 s motion is accelerated, hence displacement-time graph is a parabola. Between


2 to 4 s motion is uniform, so displacement-time will be a straight line. Between 6 to 8 s motion is decelerated
hence displacement-time graph is again a parabola but inverted in shape. At the end of 8 s velocity is zero,
therefore, slope of displacement-time graph should be zero. The corresponding graph is shown in figure.

DO IT YOURSELF
Q.1 Velocity time graph for a particle moving in a line is given.
2
Calculate the displacement of the particle and distance travelled 4
v (m/s) s
in first 4 seconds. 0 2 6
–2

Q.2 The velocity time graph of a body moving in a straight line is as follows. 2
Find the magnitude of displacement of body and distance travelled by
1
body during first 6 seconds.
v m/s

0
1 2 3 4 5 6t
–1
–2

Q.3 A particle moves in a straight line from origin. Its velocity


4
time curve is shown. Find the distance of particle at t = 8 s
2
from the origin and the distance travelled by the particle
v m/s

during first 8 seconds. 0


1 2 3 4 5 6 7 8
–2 t(s)
–4

Q.4 A particle starts with an initial velocity 2.5 m/s along the positive x-axis and it accelerates uniformly at the rate
0.5 m/s2. (a) Find the distance travelled by it in the first two seconds. (b) How much time does it take to reach
the velocity 7.5 m/s ? (c) How much distance will it cover before reaching the velocity 7.5 m/s ?
Ans. (1) 0, 4 m (2) 4m, 6m (3) 6m, 18m (4) 6.0 m, 10 s, 50 m

31
Motion in a Plane
EQUATIONS OF MOTION
r
r r dv r
If a particle moves with acceleration a , then by definition a = or dvr = adt
dt
®
v t
r r r
Let at starting (t = 0) initial velocity of the u and at time t its final velocity = vr then ò dv = ò adt
® 0
f t u
r r r v ® t
®

If acceleration is constant (uniformally accelerating motion) ò


i
ò
dv = a dt or
0
ëév ûù ® = a ëét ûù 0
u

or r r r or r r r ...(1)
v - u = at v = u + at
scalar form of equation (1) is v = u + at ...(i)
Now by definition of velocity, eq . (1) reduces to
n

z z
®
r r
bur + atr gdt Þ rs = LMNutr + 12 atr OPQ
s t t
r ds ds r r r 2 r r 21r
v= or = u + at or ds = Þ s = ut + at ...(2)
dt dt 0 2
0 0

1 2
scalar form of equation (2) is s = ut + at ...(ii)
2
Now substituting the value of t from eqn. (i) to eqn. (ii)

s=u
bv - ug + 1 a LM v - u OP 2

or 2as = 2uv – 2u2 + v2 + u2 – 2uv Þ v2 = u2 + 2as ...(iii)


a 2 N a Q
rr
vector form of equation (iii) v 2 = u 2 + 2a. s ...(3)
These three equation are called equations of motion and are applicable only and only when acceleration is
constant.
distance travalled by the body in nth second

1 2 1 1 2 1 a
an - u ( n - 1) - a ( n - 1)
2
S nth = S n - S n -1 = un + = un + an - un + u - an2 + an -
2 2 2 2 2

a
S nth = u + (2n - 1) ...(iv)
2

r r
r a
vector form of equation (iv) Snth = u + ( 2n - 1) ...(4)
2
VERTICAL MOTION UNDER GRAVITY
If air resistance is neglected and a body is freely moving along vertical line near the earth surface then an
acceleration acts downward which is 9.8m/s2 or 980 cm/s2 or 32 ft/s2
Freely falling bodies from a height h above the ground
Taking initial position as origin and direction of motion (i.e. downward direction) positive y axis,
as body is just released/dropped u = 0 u = 0, t = 0

acceleration along +Y axis a = g


Use equations of motion to describe the motion, i.e. v
y
1 2
v = u + at, y = ut + at , v2 = u2 + 2ay
2
Let the body acquires velocity v(downward) after falling a distance h in time t, then
v = gt .....(1) or t = v/g

32
Motion in a Plane

1 2 2h
h= gt ....(2) or t=
2 g

v2
v2 = 2gh ....(3) or v= 2gh or h =
2g
(v–t graph) (h–t graph)
a
(a–t graph) v h
hµt
2

g
aµt°
v µt
tanq=g
q
t t t
Body is projected vertically upward : Take initial position as origin and direction of motion
(i.e. vertically upward) as positive y-axis. y v=0
v = 0 at maximum height, at t = T,
a = – g (because directed downward)
y=0 u = 0, t = 0
Put the values in equation of motion v = u + at Þ 0 = u – gT or u = gT

1 2 1 2 1 1 1
s = ut + at Þ h = ut – gt or [hmax = uT – gT2 ] or [hmax = (gT)T – gT2 = gT2 ]
2 2 2 2 2
v2 = u2 + 2as Þ v2 = u2 – 2gh or 0 = u2 – 2g hmax or u2 = 2g hmax Again u = gT

1 2h max
and hmax = gT2 or T = and u2 = 2g hmax or u = 2g h max
2 g

After attaining maximum height body turns and come back at ground. During complete flight acceleration is constant,
a = –g. y v=0

Time taken during up flight and down flight are equal h


u
u 2u
Time for one side T = and total flight time = 2T = g
g
At each equal height from ground speed of body will be same either going up or coming down.
GRAPHS
v
a +u
+ 2T
T
velocity

0 t 0
time time

-g

S
2hmax
hmax
displacement

hmax
travelled
distance

t
T 2T t
time T 2T
time

33
Motion in a Plane
Ex. A driver takes 0.20 s to apply the brakes after he sees a need for it. This is called the reaction time of the driver.
If he is driving a car at a speed of 54 km/h and the brakes cause a deceleration of 6.0 m/s 2, find the distance
travelled by the car after he seeds the need to put the brakes on
Sol. Distance covered by the car during the application of brakes by driver -

5
u = 54 km/h = 54 × m/s = 15 m/s
18

s1 = ut or s1 = 15 x 0.2 = 3.0 meter After applying the brakes;


v=0 u = 15 m/s, a = 6 m/s2 s2 = ?

225
Using v2 = u2 - 2as 0 = (15)2 – 2 × 6 × s2 Þ 12 s2 = 225 Þ s2 = = 18.75 metre .
12

Distance travelled by the car after driver sees the need for it s = s1 + s2 = 3 + 18.75 = 21.75 metre.
Ex. A passenger is standing d distance away from a bus. The bus begins to move with constant acceleration a. To
catch the bus, the passenger runs at a constant speed u towards the bus. What must be the minimum speed of
the passenger so that he may catch the bus.
Sol. Let the passenger catch the bus after time t.

1 2
The distance travelled by the bus, s1 = 0 + at ...(1)
2
and the distance travelled by the passenger s2 = ut + 0 ...(2)
Now the passenger will catch the bus if d + s 1 = s2 ...(3)

1 2
Substituting the values of s1 and s2 from eqn. (1) and eqn. (2) in (3) d + at = ut
2

1 2 [u ± u 2 - 2ad ]
i.e. at – ut + d = 0 or t=
2 a

So the passenger will catch the bus if t is real, i.e., u2 ³ 2 ad or u ³ 2 ad

So the minimum speed of passenger for catching the bus is 2 ad .

Ex. A body is freely dropped from a height h above the ground. Find the ratio of distances fallen in first one second,
first two seconds, first three seconds, also find the ratio of distances fallen in 1 st second, in 2nd second, in
3rd second etc.

1 2 1 2
Sol. From second equation of motion, i.e. h = gt (h = ut + gt and u = 0)
2 2

1 1 1
h1 : h2 : h3 ...... = g(1)2 : g(2)2 : g(3)2 = 12 : 22 : 32 ........ = 1: 4 : 9 :..... .
2 2 2
Now from the of distance travelled in nth second

1 1
Sn = u + a (2n –1) here u = 0, a = g Sn = g (2n – 1)
2 2

1 1 1
or S1 : S2 : S3 ........ = g (2 ´ 1 – 1) : g (2 ´ 2 – 1) : g(2 ´ 3 – 1) = 1 : 3 : 5 ........
2 2 2

34
Motion in a Plane
Ex. A rocket is fired vertically up from the ground with a resultant vertical acceleration of 10m/s 2. The fuel is
finished in 1 minute and it continues to move up.
(a) What is the maximum height reached?
(b) After finishing fuel, calculate the time for which it continues its upwards motion.
(Take g = 10 m/s2)
Sol. (a) The distance travelled by the rocket during burning interval (1minute= 60s) in which resultant
acceleration is vertically upwards and 10 m/s2 will be
h1 = 0 × 60 + (1/2) × 10 × 602 = 18000 m = 18 km and velocity acquired by it will be
v = 0 + 10 × 60 = 600 m/s
Now after 1 minute the rocket moves vertically up with initial velocity of 600 m/s and acceleration due to
gravity opposes its motion. So, it will go to a height h2 from this point, till its velocity becomes zero such that
\ 0 = (600)2 –2gh2 or h2 = 18000 m = 18 km [g = 10 ms–2]
So the maximum height reached by the rocket from the ground, H = h1 +h2 = 18 + 18 = 36 km
(b) As after burning of fuel the initial velocity 600m/s and gravity opposes the motion of rocket, so from 1 st
equation of motion time taken by it till it velocity v =0
0 = 600 –gt Þ t = 60 s
Ex. A ball is thrown upwards from the top of a tower 40 m high with a
+ve
velocity of 10 m/s, find the time when it strikes the ground u=10m/s
(g = 10 m/s2) a=–10m/s2

Sol. In the problem u = + 10 m/s, a = – 10 m/s2 and s = –40m


h
(at the point where ball strikes the ground)

1 2
Substituting in s = ut + at Þ – 40 = 10t – 5t2
2
or 5t2 – 10t – 40 = 0 or t2 – 2t – 8 = 0
Solving this we have t = 4 s and – 2s. Taking the positive value t = 4s.

Ex. A particle moves in a straight line with a uniform acceleration a. Initial velocity of the particle is zero. Find the
average velocity of the particle in first 's' distance.

1 2 s2 1 s as
Sol. Q s = at \ 2
= as Average velocity = =
2 t 2 t 2
DO IT YOURSELF
Q.1 If a body travels half of total path in the last second of its fall from rest, then find: (a) the time of fall and (b) the
height of its fall. (g = 9.8 m/s2)
Q.2 A ball is projected vertically up wards with a velocity of 100 m/s. Find the speed of the ball at half the maximum
height. (g = 10 m/s2)
Q.3 A man standing on the edge of a cliff throws a stone straight up with initial speed u and then throws another
stone straight down with the same initial speed and from the same position.Find the ratio of the speed the stones
would have attained when they hit the ground at the base of the cliff.
Q. 4 A ball is projected vertically up with an initial speed of 20 m/s and acceleration due to gravity is 10 m/s 2 .
(a) How long does it take to reach the highest point? (b) How high does it rise above the point of projection?
(c) How long will it take for the ball to reach a point 10 m above the point of projection?

35
Motion in a Plane

Q.5 A pebble is thrown vertically upwards from a bridge with an initial velocity of 4.9 m/s. It strikes the water after
2 s. If acceleration due to gravity is 9.8m/s2 (a) what is the height of the bridge? (b) with what velocity does the
pebble strike the water ?
Q.6 A body is released from a height and falls freely towards the earth, exactly 1second later another body is
released. What is the distance between the two bodies 2 second after the release of the second body.
(g = 9.8m/s2)
Q.7 The retardation of a particle moving in a straight line is proportional to its displacement (proportionality constant
being unity). Find the total distance covered by the particle till it comes to rest. Given that velocity of particle is
v0 at zero displacement.

Ans. (1) (a) 3.414 seconds (b) 57 m (2) 50 2 m/s (3) 1:1

(4) (a) t= 2 s, (b) h = 20 m, (c) 3.41s (5) (a) 9.8m, (b) 14.7 m/s (6) 24.5 m (7) v0

VERTICAL MOTION UNDER GRAVITY IN PRESENCE OF AIR RESISTANCE


A body is thrown vertical upward from ground with velocity U. A constant air resistance (R) acts on this body
during the motion, which is opposite to motion of the body.
(a) Upward Motion - In upward motion , net force on the body is mg + R (downward). Thus,
R U U2
Net acceleration , a1 = g + Time of ascending t1 = maximum height, h =
m R æ Rö
g+ 2ç g + ÷
m è mø
(b) Downward Motion : In downward motion, net force on the body is mg-R (upward), thus
R
Net acceleration a2 = g - Time of decending (from III eq. of motion)
m
1 R
h = 0+ (g - )t22
2 m
R
U2 1 R U g-
t2 = t1 m < 1 GRAVITY
Þ = (g - ) t22 Thus =
R 2 m 2
t2 R
2(g + ) 2 æRö g+
m g -ç ÷ m
èmø

36
Motion in a Plane

S.N. Different Case s v-t graph s-t graph Important Points

1. Uniform motion v s (i) Slope of s – t graph =


v = constant
v = constant (ii) In s – t graph s = 0 at
s = vt
t t=0

2. Uniformly accelerated (i) u = 0, i.e. v = 0at t = 0


motion with u = 0 at v s (ii) u = 0, i.e., slope of s - t
t = 0 1 graph at t = u, should
s = at2
v = at 2 be zero
t t
(iii) a or slope of v - t
graph is constant

3. Uniformly accelerated (i) u ¹ 0, i.e., v or slope


motion with u ¹ 0 at t = 0 v s of s – t graph at t = 0
is not zero
u v = u + at 1 2
s = ut + at (ii) v or slope of s – t graph
t 2
t gradually goes on
increasing.

4. Uniformly accelerated v s (i) s = s0 at t = 0


motion with u ¹ 0
and s = s0 at t = 0
u 1 2
v = u + at s = s0+ ut+ at
2
t t

5. Uniformly retarded (i) Slope of s – t graph at


motion till velocity v s t = 0 gives u
becomes zero 1 2 (ii) Slope of s – t graph at
u s = ut – at t = t0 becomes zero
v = u + at 2
(iii) In this case u can't
t t be zero.
to to

6. Uniformly retarded (i) At time t = to, v = 0 or


then accelerated in v s slope of s – t graph is zero
opposite direction (ii) In s – t graph slope or
velocity first decreases
u then increases with
to
t t opposite sign.
o to

37
Motion in a Plane

SOLVED EXAMPLES
Ex.1 A ball is thrown vertically upwards with a velocity of 20 ms–1 from the top of a tower. The height of the
tower is 25 m from the ground.
(a) How high will the ball rise ? and
(b) How long will it be before the ball hits the ground ? (Take g = 10 ms–2)
Sol. Let vertically upwards be positive direction
(a) Now u = +20 m/s, a = –10 ms–2 , v = 0
If ball rises to height h from the point of launch, then by using v2 = u2 + 2as
0 = (20)2 + 2 (–10) h or h = 20 m
1 2
(b) S = ut + at here S = –25 m, u = +20 m/s, a = –10 m/s2
2

1
– 25 = + 20t + (–10) t2 or t2 – 4t – 5 = 0
2
or (t – 5) (t + 1) = 0 or t = 5s, –1s
time can not be negative so that t = 5s

Ex.2 A body has to travel 120 km distance. It travels first 30 km with uniform speed 30 km/h. Its average speed
is 60 km/hr. What should be its speed in next 90 km distance ?
total dis tan ce 120
Sol. Average speed = Þ 60 = Þ t = 2h
total time t

30
time taken to cover first 30 km = = 1 h,time taken to cover next 90 km = 1 h
30

90
So that speed in this duration = = 90 km/h
1

Ex.3 For shown situation in which interval is the average speed greatest ?
Dx
Sol. 3, because v av = ; Dx in 3 is maximum
Dt

Ex.4 A particle starts motion from rest and moves along a straight line. Its acceleration-time graph is shown.
Find out speed of particle at t = 2s and at t = 3s.
Sol. Velocity = area under a – t curve

1
at t = 2s, v = 1 × 1 + × 1 × 1 = 1.5 ms–1
2

1
at t = 3s, v = 1.5 – × 1 × 1 = 1 ms–1
2
dv
Ex.5 The motion of a body is given by = 6 – 3v where v is the velocity in ms–1 and t is in second. Calculate
dt
the terminal speed of the body.
dv
Sol. At terminal speed = 0 so 6 – 3vt = 0 Þ vt = 2 ms–1
dt

38
Motion in a Plane
Ex.6 A particle is moving along x - axis with acceleration a = a 0 (1 – t/T) where a0 and T are constants. The
particle at t = 0 has zero velocity. Calculate the average velocity between t = 0 and the instant when a = 0.

F tI
z z FG1 - t IJ 2
dv t
dt Þ v = a G t - 2T J
v t
Sol. (a)
dt
= a0(1 – ) Þ
T 0
dv =
0
a0
H TK H K 0

Ft - t I F I
Q
dx
dt
= v so z z
dx = v dt Þ x = z t

0
a0
2

GH 2T JK dt Þ x = a GH t2 - 6t T JK Now
0
2 3

a=0 Þt=T

FT 2
T3 I
displacement
a0 GH 2 -
6T JK a0T
average velocity = = =
time T 3

Ex.7 A train, travelling at 20 km/hr is approaching a platform. A bird is sitting on a pole on the plateform. When
the train is at a distance of 2 km from pole, brakes are applied which produce a uniform deceleration in
it. At that instant the bird flies towards the train at 60 km/hr and after touching the nearest point on the
train flies back to the pole and then flies towards the train and continues repeating itself. Calculate how
much distance will the bird have flown before the train stops ?
Sol. For retardation of train
v2 = u2 + 2aS Þ 0 = (20)2 + 2(a)(2) Þ a = –100 km/hr2
Time required to stop the train
1
v = u + at Þ 0 = 20 – 100t Þ t = hr..
5

Dis tance 1
For Bird speed = Þ SB = vB × T = 60 × = 12 km.
time 5
Ex.8 A truck starts from rest with an acceleration of 1.5 metre/sec2 while a car 225 metre behind starts from
rest with an acceleration of 2 metre/sec2. Find the time in which car overtake to the truck.
Sol. Relative acceleration = 2 – 1.5 = 0.5 m/s2
Relative distance = 225 metre, Relative initial velocity = 0

FG 1IJ
Let time taken to catch the truck by the car is 't' then by s = ut + 2 at2 H K
1
Þ 225 = 0 + × 0.5 × t2 Þ t2 = 900 Þ t = 30s
2
Ex.9 A particle starts with an initial velocity and passes successively over the two halves of a given distance
with acceleration a1 and a2 respectively show that the final velocity is the same as if the whole distance is
FG a 1 + a2 IJ
covered with a uniform acceleration H 2
. K
Sol. In first case

FG a 1 IJ
+ a2
v12 = u2 + 2a1S and v22 = v12 + 2a2S Þ v22 = u2 + 2a1S + 2a2S Þ v22 = u2 + 2 H 2 K(2S)

FG a 1 + a2 IJ
In second case v2 = u2 + 2 H 2 K (2S) Þ v = v2

Ex.10 The acceleration versus time graph of a particle moving along a straight line is shown in the figure. Draw
39
Motion in a Plane

the velocity–time graph. (Assume at t = 0, v = 0.)

Sol.

Ex.11 A car, starting from rest, accelerates at the rate f through a distance S, then continues at constant speed
f
for time t and then come to rest with retardation if the total distance travelled is 15S then calculate the
2
value of S in term of f and t.

1
Sol. for time t1 vm = ft1 and S = f t12
2

f
for time t3 0 = vm – t Þ t2 = 2t1
2 2

1 FG f IJ t FG f IJ e4t j = f t
2 2
= 2S Therefore S + vmt + 2S = 15S Þ vmt = 12S Þ ft1t = 12S
S3 =
H 2K =
H 4K
2
1 1
2 2

F 2S I
Þf G J
1
2
t = 12 S Þ 2Sf =
144S2
ÞS=
ft 2
HfK t2 72

Ex.12 A man in a lift asending with an upward acceleration a throws a ball vertically upwards with a velocity v
and catches it after t1 second. Afterwards when the lift is desending with the same acceleration a acting
downwards, the man again throws the ball vertically upwards with the same velocity and catches it after
t2 second. Find the velocity and acceleration.

2v 2v
Sol. For upward motion of lift t1 = For downward motion of lift t2 = g – a
g+a

2v 2v gt1t 2 2 FG g t t IJ Þ a = FG t
1 2 2 - t1 IJ
Þ g+a=
t1
& g – a = t2
Þ v = t1 + t 2 Þ g + a = t1 Ht +t K
1 2 Ht 2 + t1 K g

Ex.13 A particle starts from rest. Its acceleration at t = 0 is 5 ms–2 which varies

with time as shown in figure. Calculate maximum speed of the particle.


1
Sol. vmax = Area under a - t curve = × 5 × 6 =15 ms–1
2

40
Motion in a Plane

Ex.14 A particle is moving along X-axis. It's

velocity time graph is given. Draw it


acceleration - time and displacement-time

curves.

Sol.

Ex.15 A particle is projected vertically upwards from a point X on the ground. It takes a time t 1 to reach a point
A at a height h above the ground, it continues to move and takes a time t2 in remaining path to reach the
ground. Find value of h.
Sol. Time of flight of particle

t = t1 + t2 =
2u
or u=g
bt 1 + t2 g
g 2
1 2 g 1 g
Now h = ut1 – gt1 = (t1 + t2)t1 – gt12 = t1t2
2 2 2 2
Ex.16 A lift performs first part of its ascent motion with uniform acceleration ‘f’ and the remainder with uniform
retardation ‘2f’. Prove graphically that, if ‘h’ is the height traversed by lift and time ‘t’, is total time of
1 2
ascent then h = ft .
3
Sol. Let OC = t1 and CB = t2 Clearly t = t1 + t2
1 1
h = height attained = Area of DOAB = AC × OB = × v × t
2 2

1
\ h= vt ............(i)
2

v v v v v v 3v 2
Again f = t Þ t1 = and 2f = t Þ t2 = Q t = t1 + t2 = + = Þ v= ft
1 f 2 2f f 2f 2f 3

1 2 1 1
Putting this value of v in (i) h = × ft × t = ft2 i.e. h = ft2.
2 3 3 3
Ex.17 A particle is moving on a circular path of radius r with constant speed v. It describes angle of 120° at the
centre, calculate its –
(a) Magnitude of displacement
(b) Magnitude of change in velocity
(c) Magnitude of average acceleration
q
Sol. (a) Displacement = 2r sin = 2r sin60° = 3r
2
q
(b) Change in velocity = 2vsin = 2vsin60° = 3v
2
Change in velocity 3v 3v 3 3v 2
(c) Average acceleration = = = =
timetaken (length of arc) speed 2pr 3v b g 2pr
41
Motion in a Plane
Ex.18 A man can swim with a speed 4 km/hr in still water.

(a) How long does he takes to cross a river 1 km wide if the river flows steadily at 3 km/hr and makes
his strokes normal to the river current ?

(b) How far down the river does he go when he reaches the other bank ?

AB
Sol. (a) t= v
m

Here AB is width of river and Vm is speed of man in perpendicular

direction to the river.

1
t= hr = 15 min.
4

(b) The distance travelled along the river during its crossing is

1
BC = vR × t = 3 × = 0.75 km
4

Ex.19 A man can swim at a speed 2 ms–1 in still water. He swims at angle 150° with direction of water flow and
reaches at the exactly opposite point on the other bank.
(a) Find out speed of flowing water and
(b) If width of river is 2 km then calculate time taken to cross the river.
Sol. (a) To reach at just opposite point

3 –1
VSR sin 60° = VR Þ 2 × = VR Þ VR = 3 = 1.732 ms
2

width 2000
(b) t= = = 2000 s
VSR cos60° 1

2

Ex.20 A man crosses a river in a boat. If he cross the river in minimum time he takes 10 minutes with a
horizontal drift 120 m. If he crosses the river taking shortest path in 12.5 minutes then find width of the
river, velocity of the boat w.r.t. water and speed of flow of river.

Sol. For shortest time

x x 120 1 d
t1 = Þu= = = ms–1 and t1 = Þ d = 600v
u t1 10 ´ 60 5 v

For shortest path


d 600v 16 2 1
t2 = 2 2 Þ 12.5 × 60 = Þ v = v2 –
v -u 2 25 25
e j
v2 - 1
5

1 1
Þv= ms–1 Þ d = 600 × = 200m
3 3
42
Motion in a Plane
Ex.21 The water of a 0.5 km wide river is flowing with a velocity of 4 km/h. A boatman standing on one of the
bank of the river wishes to take his boat to a point on the opposite bank exactly infront of his present
position. He can row his boat with a velocity of 8 km/h. relative to the water. In which direction should he
row boat ? Find time will be taken to cross the river.

4 1
Sol. From figure sinq = = , i.e., q = 30° so the boatman should row his boat
8 2
at an angle 30° with AB or 30° + 90° = 120° with the flow of water.
From figure (AE)2 = (AD)2 – (DE)2
If v be the velocity of boatman in direction AB, then

v2 = (8)2 – (4)2 = 48 Þ v = 48 = 6.9 km/h.

dis tan ce AB 0.5 km 0.5 ´ 60


Time taken in crossing the river = = 6.9 km / h = min. = 4.3 min,
speed in this direction 6.9

43
Motion in a Plane

NCERT BASED QUESTION


1. Can Earth be regarded as a point object when it is revolving around the Sun?

2. Can displacement be greater than the distance traversed by an object?

3. Give an example where displacement of a particle is equal to the distance covered by it.

4. Is it possible that the displacement may be zero but not the distance?

5. The instantaneous speed is always equal to the magnitude of instantaneous velocity. Why ?

6. What does the speedometer of a car measure ?


7. A drunkard walking in a narrow lane takes 5 steps forward and 3 steps backward, followed again by 5 steps
forward and 3 steps backward, and so on. Each step is 1 m long and requires 1 s. Plot the x-t graph of his motion.
Determine graphically and otherwise how long the drunkard takes to fall in a pit 13 m away from the start.
8. A jet airplane travelling at the speed of 500 km h–1 ejects its products of combustion at the speed of
1500 km h–1 relative to the jet plane. What is the speed of the latter with respect to an observer on the ground?
9. A car moving along a straight highway with speed of 126 km h–1 is brought to a stop within a distance of
200 m. What is the retardation of the car (assumed uniform), and how long does it take for the car to stop?
10. A player throws a ball upwards with an initial speed of 29.4 m s–1 .
(a) What is the direction of acceleration during the upward motion of the ball ?
(b) What are the velocity and acceleration of the ball at the highest point of its ?
(c) Choose the x = 0 m and t = 0 s to be the location and time of the ball at its highest point, vertically downward
direction to be the positive direction of x-axis, and give the signs of position, velocity and acceleration of the ball
during its upward, and downward motion.
(d) To what height does the ball rise and after how long does the ball return to the player's
hands ? (Take g = 9.8 m s–2 and neglect air resistance).
11. Suggest a suitable physical situation for each of the following graphs :
x v

A
B t t

(a) (b)
Fig.
12. The speed-time graph of a particle moving along a fixed direction is shown in Fig. Obtain the distance traversed
by the particle between (a) t = 0 s to 10 s, (b) t = 2 s to 6 s.
Speed (ms )
–1

12

0 5 10 t
Fig.
What is the average speed of the particle over the intervals in (a) and (b) ?
44
Motion in a Plane
13. A car is moving along a straight line, say OP in Fig. It moves from O to P in 18s and returns from P to Q in 6.0s.
What are the average velocity and average speed of the car in going (a) from O to P ? and (b) from O to P and
y
back to Q ?

x
O Q P
240m
400m
14. The position of an object moving along x-axis is given by x = a + bt2 where a = 8.5 m, b = 2.5 m s–2 and t is
measured in seconds. What is its velocity at t = 0 and t = 2.0 s. What is the average velocity between t = 2.0 and
t = 4.0 s ?
15. A ball is thrown vertically upwards with a velocity of 20 m s–1 from the top of a multistorey building. The height
of the point from where the ball is thrown is 25.0 m from the ground. (a) How high will the ball rise? and (b) how
long will it be before the ball hits the ground? Take g = 10 m s–2.
16. Stopping distance of vehicles : When brakes are applied to a moving vehicle, the distance it travels before
stopping is called stopping distance. It is an important factor for road safety and depends on the initial velocity
(v0) and the braking capacity, or deceleration, –a that is caused by the braking. Derive an expression for stopping
distance of a vehicle in terms of v0 and a.
17. Two parallel rail tracks run north-south. Train A moves north with a speed of 54 km h–1, and train B moves south
with a speed of 90 km h–1. What is the
(a) velocity of B with respect to A ?,
(b) velocity of ground with respect to B ?, and
(c) velocity of a monkey running on the roof of the train A against its motion (with a velocity of 18 km–1 with
respect to the train A) as observed by a man standing on the ground ?
18. The position-time (x-t) graphs for two children A and B returning from their school O to their homes P and Q
respectively are shown in fig. Choose correct entries in the brackets below :
(a) (A/B) lives closer to the school than (B/A)

(b) (A/B) starts from the school earlier than (B/A)


(c) (A/B) walks faster than (B/A)
(d) A and B reach home at the (same / different) time
(e) (A/B) overtakes (B/A) on the road (once/ twice).
19 Two trains A and B of length 400 m each are moving on two parallel tracks with a uniform speed of 72 km h–1
in the same direction, with A ahead of B. The driver of B decides to overtake A and accelerates by 1 ms –2. If
after 50 s, the guard of B just brushes past the driver of A, what was the original distance between them?
20. Two towns A and B are connected by a regular bus service with a bus service with a bus leaving in either
direction every T minutes. A man cycling with a speed of 20 km h–1 in the his motion, and every 6 min in the
opposite direction. What is the period T of the bus service and with what speed (assumed constant)
do the buses run on the road ?
21. Look at the graphs (a) to (d) fig. carefully and state, with reasons, which of these cannot possibly represent one-
dimensional motion of a particle.

45
Motion in a Plane

NCERT BASED SOLUTION


1 to 6 are conceptual
7. Drunkard moves 5 m forward & 3 m backward so fall in pit after 37 sec. after 13 m ahead.
x(m)
14
12
10
8
6
4
2

5 8 10 13 1516 2021 2425 2930 32 35 37 40 t(s)

x-t graph
r r
8. Vpa = 500 km/hr VgasG = ?
r
VgasP = – 1500 km/hr
r r r
VgasP = VgasG – VPG

9. By v2 = a2 + 2as
10. (a) During upward motion acceleration a = g is dowanward.
(b) At heighest point a = g & v = 0
(c) Consider upward direction – ve & downward + ve

u2 u
(d) H = ,t=
2g g

11. (a) The given x-t graph shows that initially a body was at rest. Then, its velocity increases with time and attains
an instantaneous constant value. The velocity then reduces to zero with an increase in time. Then, its velocity
increases with time in the opposite direction and acquires a constant value. A similar physical situation arises
when a football (initially kept at rest) is kicked and gets rebound from a rigid wall so that its speed gets reduced.
Then, it passes from the player who has kicked it and ultimately gets stopped after sometime.
(b) In the given v-tgraph, the sign of velocity changes and its magnitude decreases with a passage of time. A
similar situation arises when a ball is dropped on the hard floor from a height. It strikes the floor with some velocity
and upon rebound, its velocity decreases by a factor. This continues till the velocity of the ball eventually becomes
zero.

12. Distance travelled by the particle = Area under the given graph

Dis tan ce
Average speed =
Time
Displacement
13 (a) Average velocity =
Time interval
+360 m
n= = 20 m s–1
18s

Path length = 360 m


Average speed = = 20 m s–1
Time interval 18s

Thus, in this case the average speed is equal to the magnitude of the average velocity.
46
Motion in a Plane
(b) In this case,

Displacement = +240 m
Average velocity = = +10 m s–1
Time interval (18 + 6.0)s
Path length OP + PQ = 360 + 120m
Average speed = = = 20 m s–1
Time interval t 24s

dx d
14. In notation of differential calculus, the velocity is v = = (a + bt 2 ) = 2b t = 5.0 t ms -1
dt dt
At t = 0 s, v = 0 m s–1 and at t = 2.0 s, v = 10 m s–1
a + 16b - a - 4b
Average velocity = = 6.0 b = 6.0 × 2.5 = 15 ms–1
2.0
15. (a) Let us take the y-axis in the vertically upward direction with zero at the ground, as shown in Fig.
Now v0 = + 20 m s–1,
a = – g = –10 m s–2,v = 0 m s–1
If the ball rises to height y from the point of launch, then using the equation
v 2 = v 20 2a y - y 0
we get
0 = (20)2 + 2(–10)(y – y)
Solving, we get, (y – y0) = 20 m.
(b)
B
(y–y0)
A 25m

y
a = –10 m/s2
25m

C 0
Fig.
The total time taken can also be calculated by noting the coordinates of initial and final positions of the ball with
respect to the origin chosen and using equation

1 2
y = y0 + v0 t at
2
Now y0 = 25 m, y = 0 m,
v0 = 20 m s , –1
a = –10 m s–2, t=?

1
0 = 25 + 20 t + ( ) (–10) t2
2
Or, 5t2 – 20t – 25 = 0
Solving this quadratic equation for t, we get t = 5s
16. Let the distance travelled by the vehicle before it stops be ds.

Then, using equation of motion v 2 = v 02 + 2ax , and noting that v = 0, we have the stopping distance.
47
Motion in a Plane
Thus, the stopping distance is proportional to the square of the initial velocity. Doubling the initial velocity increases
the stopping distance by a factor of 4 (for the same deceleration).
For the car of a particular make, the braking distance was found to be 10 m, 20 m, 34 m and 50 m corresponding
to velocities of 11, 15, 20 and 25 m/s which are nearly consistent with the above formula.
Stopping distance is an important factor considered in setting speed limits, for example, in school zones.
17. Choose the positive direction of x-axis to be from south to north. Then,
vA = + 54 km h–1 = 15 m s–1
vB = –90 km h–1 = –25 m s–1
Relative velocity of B with respect to vBA = vB – vA= –40 m s–1 , i.e. the train B appears to A to move with a
speed of 40 m s–1 from north to south.
In (c), let the velocity of the monkey with respect to ground be vM. Relative velocity of the monkey with respect
to A,
vMA = vM – vA = –18 km h–1 = –5 ms–1.
Therefore, vM = (15 – 5) m s–1 = 10 m s–1 X
Q
P R
18. (a) A lives closer to the school than B (as P is close to O than Q)
A
(b) A starts earlier than B (A starts when time t = 0) B

(c) B walks faster than A (slope of B is greater than slope of A) O


t1 E

(d) A and B reach home at the same time t1 (as shown by dotted lines in the graph)
(e) B overtakes A on the road once (as graphs for A and B intersect only at one point R.)

19. Let s = distance between the trains initiallyVelocity of train B w.r.t. A is zero

1 1
acceleration of train B w.r.t. A a = 1ms–2 than s = ut + at2 =0+ × (50)2 = 1250m
2 2
20. Let v = speed of buses playing between A and B
The relative speed of bus going from A to B w.r.t. cyclist = (v – 20) km h–1
and relative speed of bus going from B to A w.r.t. cyclist = (v + 20) km h–1
in time T, distance covered by bus = vT
vT
when bus and cyclist are in same direction then = 18 min ........(1)
v - 20

vT
when bus and cyclist are in opposite direction then = 6 min ........(2)
v + 20

v + 20 18
dividing eq (1) by eq (2) = =3 Þ v = 40 km h–1
v - 20 6

40 ´ T 40
putting value v in eq (1) = 18 min Þ T = 18 min Þ T = 9 min
40 - 20 20
21. None of these graphs can represent one dimensional motion of the particle.
in fig (a) : A particle can not have two position at the same time.
in fig. (b) : A particle can not have both +ve and –ve velocity at the same time.
in fig. (c) : A particle can not have negative speed
in fig. (d) : Total path length can never decrease.

48
Motion in a Plane
PROJECTILE MOTION

When a body is projected such that velocity of projection is not parallel to the force, then body moves along a
curved path. This motion is called two dimensional motion. If force on the body is constant then curved path of
the body is parabolic. This motion is called projectile motion.
[ It is an example of two dimensional motion.
[ It is an example of motion with constant or uniform acceleration. Thus equations of motion can be used to solve
the projectile motion.
[ A particle thrown in the space which moves under the effect of gravity only is called as "projectile". The motion
of this particle is referred as projectile motion.
[ If a constant acceleration is given to particle in oblique direction with initial velocity, the resultant path is
parabolic. Let X-axis is along the ground and Y-axis is along the vertical then path of projectile projected at q
from the ground is as shown.
vy = u sin q – gt

v
ay = – g

vy = 0 vx= u cos q

u a
uy = u sin q

P vx = u cos q
H

q
X
(0, 0) ux = u cos q ax = 0 (R, 0)

GROUND TO GROUND PROJECTION


Projectile motion can be considered as two mutually perpendicular motions, which are independent of each
other. i.e. Projectile motion = Horizontal motion + Vertical motion
Horizontal Motion
Initial velocity in horizontal direction = u cosq = ux
force or acceleration along horizontal direction = a x = 0. (Neglect air resistance)
Therefore, Horizontal velocity remains unchanged.
[ At any instant horizontal velocity ux = u cosq
[ At time t, x co-ordinate or displacement along X-direction is
x = uxt or x = (u cosq)t
Vertical Motion : It is motion under the effect of gravity so that as particle moves upwards its vertical speed
decreases.
Initial velocity in vertical direction = u sinq = uy
Acceleration along vertical direction = ay = – g
[ At time t, vertical speed vy = uy – gt = u sinq – gt
[ In time t, displacement in vertical direction or "height" of the particle above the ground

49
Motion in a Plane

1 1
y = uyt – gt2= usinqt – gt2
2 2

®
Net Motion : Net initial velocity = u = u xˆi + u y ˆj = u cos qˆi + u sin qˆj

direction of u is at angle q from the ground


®
Net acceleration = a = a xˆi + a y ˆj = – g $j direction of g is along down wards

1
Coordinate of particle at time t : (x, y) x = uxt and y = uyt – gt2
2

Net displacement in t time = x 2 + y2


Equation of Trajectory
Along horizontal direction x = uxt or x = ucosqt
1 2 1 2
Along vertical direction y = uyt – gt or y = usinqt – gt
2 2
On eliminating t from these two equations
2
æ x ö 1 æ x ö
y = (u sin q) ç ÷ - gç ÷
è u cos q ø 2 è u cos q ø

1 x2
Þ y = x tan q - g 2 This is the equation of parabola :
2 u cos2 q

é gx ù é xù
y = x tan q ê1 - y = x tan q ê1 -
2u sin q cos q úû
Again or
R úû
2
ë ë

Velocity of particle at time t


®
v = v x ˆi + v yˆj = u xˆi + (u y - gt)ˆj = u cos qˆi + (u sin q - gt)ˆj
r =
Magnitude of velocity |v| v 2x + v 2y = u 2 - 2u y gt + g2 t 2

Þ r
|v|= u 2 + g2 t2 - 2ugt sin q
If angle of velocity vr from ground is a, then

vy u y - gt
tan a = =
vx ux

u sin q - gt gt
Þ tana = = tan q -
u cos q u cos q
Maximum height attained H
At maximum height vertical velocity becomes zero. At this instant y coordinate is its maximum height.
Q v 2y = u 2y - 2gy \ 0 = u 2y - 2gH { Q vy = 0, y = H }

50
Motion in a Plane

u2y u2 sin2 q
H= =
2g 2g
Time of flight (T)
At time T particle will be at ground again, i.e. displacement along Y-axis becomes zero.

1 1
Q y = uy t - gt 2 \ 0 = uy T - gT2 Time of flight is the time for
2 2 which projectile remains in air.

2u y 2u sin q
or T= =
g g

T u y u sin q
Assending time = desending time = = =
2 g g

T
at time particle attains maximum height of its trajectory..
2
Horizontal range or Range (R)
It is the displacement of particle along X-direction during its complete flight.

2u y 2u x u y
Q x = uxt \ R = uxT = ux ; R=
g g

2(u cos q)(u sin q) u2 sin 2q


R= Þ R= (Q 2sinq cosq = sin2q)
g g

Maximum horizontal range (Rmax)


If value of q is increased from q = 0° to 90°, then range increases from q = 0° to 45° but it decreases after 45°.
Thus range is maximum at q = 45°
u2 sin 2(45o ) u2 sin 90o
For maximum range q =45° and R max = =
g g

u2
Þ R max =
g

Kinetic energy of projectile


Let a body is projected with velocity u at angle q.
1
Thus initial kinetic energy of projectile, K 0 = mu 2
2
Since velocity of projectile at maximum height is ucosq.
1
Thus, kinetic energy at highest point , K = m(u cos q)2 = K 0 cos2 q
2

Change in momentum of projectile


When particle again returns to ground at B point, its y coordinate is zero and its magnitude
of velocity is u at angle q with ground. Total angular change = 2q

51
Motion in a Plane
®
Intial velocity u i = u cos qˆi + u sin qˆj
A v = u cos q
Y u
®
Final velocity u f = u cos qˆi - u sin qˆj H
r q
Total change in its velocity, | Dv|= 2u sin q O X
q
r r
Total change in momentum, | DP|= m| Dv|= 2mu sin q u
y T'
Comparison of two projectiles of equal range
When two projectiles are thrown with equal speeds at u
T H'
angle q and 90° - q then their ranges are equal but maximum °-q u
90
height attained are different and time of flighs are also different. H
q x
O
2
u sin 2q R
At angle q, R =
g

u 2 sin 2(90 - q) u2 sin(180 - q) u2 sin2q


At angle (90 – q), R ' = = =
g g g
Thus, R' = R
Maximum heights of projectiles
u 2 sin2 q u 2 sin2 (90 - q) u 2 cos2 q
H= and H' = =
2g 2g 2g

H sin2 q
[ = = tan2 q
H ' cos2 q

u 4 sin2 q cos 2 q R 2
[ HH ' = = Þ
4g2 16 R = 4 HH '

u 2 sin2 q u 2 cos2 q u2
[ H + H' = + Þ H + H' =
2g 2g 2g
Time of flights of projectiles
2u sin q 2u sin(90 - q) 2u cos q
T= ; T' = =
g g g
T sin q
[ = = tan q
T ' cos q
4u2 sin q cos q 2
[ TT ' = = R Þ TT' µ R
g2 g

Relative path of two projectiles


Two projectiles are thrown from ground with different velocities at different angles. Since both projectiles have
equal acceleration so their relative acceleration is zero. Thus path of one projectile w.r.t. other is straight line and
motion of one projectile w.r.t. other is uniform.
[ If u1 cosq1 = u2 cosq2 then relative path is vertical line.
[ If u1 sinq1 = u2 sinq2 then relative path is horizontal line.

Projection from a moving system


Consider a man who throws a ball from a moving trolley. Let the velocity of ball relative to man be u
® ® ® ® ® ®
v ball, trolley = v ball - v trolley i.e. v ball = v ball, trolley + v trolley

52
Motion in a Plane
u u

q q

v v

Horizontal component = u cosq + v Horizontal component = u cos q – v


Vertical component = u sin q Vertical component = u sin q

Consider a man who throws a ball from a moving balloon -

u u
v
q q

Horizontal component = u cosq Horizontal component = u cosq


Vertical component = u sinq + v Vertical component = u sin q – v

Ex. A football player kicks a ball at an angle of 30° with an initial speed of 20 m/s. Assuming that the ball travels in
a vertical plane, calculate (a) the time at which the ball reaches the highest point (b) the maximum height
reached (c) the horizontal range of the ball (d) the time for which the ball is in the air. (g = 10 m/s 2)
T u sin q 20
´ sin 30o = 2 ´ = 1s
1
Sol. (a) Time taken by the ball to reach the highest point t = = =
2 g 10 2

u2 sin2 q (20)2 ´ sin2 30o


(b) The maximum height = = = 5m
2g 2 ´ 10

u2 sin 2q (20)2 ´ sin 2 ´ 30o


(c) The horizontal range = = = 36.64m
g 10

2u sin q 2 ´ 20 ´ sin´ 30o


(d) The time of flight = = = 2s
g 10

Ex. A cricketer can throw a ball to a maximum horizontal distance of 100 m. With the same speed how high above
the ground can the cricketer throw the ball.
Sol. Let u be the velocity of projection of the ball. The ball will cover maximum horizontal distance when angle of
u2
projection with horizontal, q = 45°. Then Rmax = = 100 m
g
u2 R max
If ball is projected vertically upwards (q = 90 from ground) then H attains maximum value, Hmax
0 = =
2g 2
R max 100
\ the height to which cricketer can through the ball is = = = 50 m.
2 2

Ex. A ball is thrown at angle q and another ball is thrown at angle (90° – q) with the horizontal direction from the
same point with velocity 40 ms–1. The second ball reaches 50m higher than the first ball. Find their individual
53
Motion in a Plane
heights. g = 10 ms–2.
Sol. For the first ball, angle of projection = q, velocity of projection, u = 40 ms–1.
Let h be the maximum height attained by it.
u2 sin2 q (40)2 sin2 q
As maximum height = \ h= .....(i)
2g 2 ´ 10
For second ball, Angle of projection = (90° – q). velocity of projection, u = 40 ms–1
Maximum height reached = (h + 50) m

u2 sin2 (90o - q) (40)2 cos2 q


\ h + 50 = = .....(i)
2g 2 ´ 10

By adding (i) and (ii),

(40)2 (40)2
2h + 50 = ´ (sin2 q + cos2 q) = = 80 Þ 2h = 80 – 50 = 30 Þ h = 15 m
2 ´ 10 2 ´ 10
Height of the first ball, h = 15 m & Height of the second ball,= h + 50 = 15 + 50 = 65 m

Ex. Two bodies are thrown with the same initial speed at angles a and (90° – a) with the horizontal. What will be the
ratio of (i) maximum heights attained by them and (ii) of horizontal ranges ?

u2 u2 sin2 q
Sol. Horizontal range R = sin 2q and maximum height H =
g 2g

u2 u2 sin2 a
Case (i) when q = a ; R1 = sin 2a and H1 =
g 2g

u2 sin 2(90o - a ) u2 sin(180o - 2a) u2 sin 2a


Case (ii) when q = (90° – a); R2 = = =
g g g

u2 sin 2 (90o - a ) u2 cos 2 a H1 sin2 a R1


and H2 = = \ = = tan2 a and =1
g g
2
H2 cos a R2

Ex. The range of a particle when launched at an angle of 15° with the horizontal is 1.5 km. What is the range of the
projectile when launched at an angle 45° to the horizontal.

u2 sin 2 ´ 15o u2 1 u2
Sol. R = = 1.5 or ´ = 1.5 or = 3 km
g g 2 g

2
u2 2
o = u sin 90o = u = 3km
Horizontal range for angle of projection 45° is R' = ´ sin 2 ´ 45
g g g

Ex. A ball of mass m is thrown vertically up. Another ball of mass 2m is thrown at an angle q with the vertical. Both
of them stay in air for the same period of time. What is the ratio of the height attained by the two balls.

2u1
Sol. For the ball thrown vertically upwards, the time taken by the ball to come back is T1 = .
g

For the ball projected at an angle q with their vertical, the time of flight is T2 = 2u2 cosq/g
54
Motion in a Plane

2u1 2u 2 cos q
Since time of flights for the balls is same, so = or u1 = u2 cosq.
g g

u12 u22 h1 u12 u 22 cos 2 q


Q h1 = and h2 = cos2 q \ = 2 2
= 2 2
=1
2g 2g h2 u 2 cos q u 2 cos q

Ex. A body is projected with the velocity u1 from the point A as shown in Figure. At the same time another body is
u1
projected vertically upwards with the velocity u2 from the point B. What should be the value of u for both the
2

bodies to collide.
Sol. The two bodies will collide, if they reach at a point acquiring the same vertical distance in the same time.
1 1
y = u1 sin 60° ´ t - gt 2 = u 2 t - gt 2
2 2
u1 u2
3 u1 2 A 60° B
or u1 ´ t = u2 t or =
2 u2 3

Ex. A particle thrown over a triangle from one end of a horizontal base falls on the other end of the base after grazing
the vertex. If a and b are the base angle of triangle and angle of projection is q, then prove that
tan q = tan a + tan b .
Sol. From triangle y = x tan a
u y
y = (R – x) tan b q
a b
y y yR
tan a + tan b = + = x R-x
x R-x x(R - x)

é xù yR
Q y = x tan q ê1 - Þ tan q = \ tan q = tan a + tan b
ë R úû x(R - x)

Ex. A hunter aims his gun and fires a bullet directly towards a monkey sitting on a distant tree. At the instant the
bullet leaves the barrel of the gun, the monkey drops from the tree freely. Will the bullet hit the monkey
Sol. Let the horizontal distance to the tree be x and original height of the monkey be H.
Then tan q = (H/x). If there was no gravity the bullet would reach height H in the time t taken by it to travel the
horizontal distance x i.e.,
x = u cosq × t and H = usinq × t
However, because of gravity, the bullet has an acceleration ‘g’ vertically downwards; so in time t the bullet will
1 1
reach a height y = ( u sin q ) ´ t- gt² = H - gt²
2 2

55
Motion in a Plane

1
This is lower than H by 2
gt2 which is exactly the amount the monkey falls in
this time. So the bullet will hit the monkey regardless of the initial velocity of the
bullet so long as it is great enough to travel the horizontal distance to the tree
before hitting the ground. However, for large u lesser will be the time of motion;M
so the monkey is hit near its initial position and for small u it is hit just before it
reaches the ground. 1
gt2
2
Note :Bullet will hit the monkey if and only if H
usinq u
1
y > 0, i.e., H- gt² > 0 M
2
q ucosq
1 1 x² x
or H > gt², i.e., H> g
2 2 u² cos ² q

x g g
or u> or u> (x² + H²) = (u 0 )
cos q 2H 2H
If u < u0 the bullet will hit the ground before reaching the monkey.

GOLDEN KEY POINTS

• At maximum height, vy = 0 and vx = ux = ucosq so that at maximum height v = v 2x + v 2y = u cos q


• At maximum height angle between velocity and acceleration is 90°

• Magnitude of velocity at height 'h' .


1 1
By energy conservation law ; mu2 + 0 = mv 2 + mgh or v= u 2 - 2gh
2 2
y
2u y u2y 2u x u y
• T= , H= , R=
g 2g g
H
T and H depends only upon initial vertical speed uy
If two projectiles thrown in different direction, have
equal time of flight then their intial vertical speeds O x
are same so that their maximum height altained is
also same. If HA = HB then (uy)A = (uy)B and TA = TB
• For situation shown in figure for q = 45°
y
u2 u2 sin2 45 u2
here R max = and H= = u
g 2g 4g H
45° x
\ R max = 4H = 4× (maximum height attained)
O
• When R=H Rmax

u2 (2 sin q cos q) u2 sin2 q R


R= and H= Þ = 4 cot q =1
g 2g H
Þ 4 cot q = 1 Þ tanq 4 Þ q = tan–1 (4) = 76°

56
Motion in a Plane
PROJECTILE THROWN PARALLEL TO THE HORIZONTAL
Consider a projectile thrown from point O at some height h (0, 0) u +x
O vx
from the ground with a velocity u in horizontal direction.
vy q
Now we shall deal the characteristics of projectile motion v
H
with the help of horizontal and vertical direction i.e.
Horizontal direction : Vertical direction :
(i) Initial velocity ux = u Initial velocity uy = 0 –y

(ii) Acceleration ax = 0 Acceleration ay = – g (downward)

Trajectory Equation
The path traced by projectile is called the trajectory. After time t,

1
x = ut and y=- gt 2 negative sign indicates the direction of vertical-displacement in downwards direction.
2

1 x2 x
so y= - g 2 (Q t= )
2 u u
This is called trajectory equation i.e. equation of a parabola
Velocity at a general point P(x, y)

v= v 2x + v 2y

Horizontal velocity of the projectile after time t is vx = u (remains constant)


Velocity of projectile in vertical direction after time t is

vy = 0 – (g)t = – gt (downward) \ v= u 2 + g2 t 2

vy gt
and tan q = or tan q = - (negative sign indicates clockwise direction)
vx u

Displacement :- The displacement of the particle is expressed by

® ® 1
s = xiˆ + yjˆ Where | s |= x2 + y2 = (ut)iˆ - ( gt2 )ˆj
2

Time of flight :- Time taken by the projectile reach to the ground.

1
From equation of motion for vertical direction. h = uy t + gt 2
2

1 2h
At highest point uy = 0 Þ h= gT 2 Þ T=
2 g

Horizontal range
Distance covered by the projectile along the horizontal direction between the point of projection to the point on

2h
the ground. R = uxt = u
g

57
Motion in a Plane

Velocity after falling height h :- Along vertical direction vy2 = 02 + 2(h)(g) vy = 2gh

OBLIQUE PROECTION FROM HEIGHT H u


[ Projection from a height at an angle q above horizontal :
ux = u cos q uy = + u sin q q
x = (u cos q) t ay = – g
X
1
– H = (u sin q) t – gt²
2 –Y
H ay = – g ¯
gt² – (2u sin q) t – 2H = 0

(2u sin q )
2
+2u sin q + + 8Hg
t=
2g

2 R
u sin q æ u sin q ö 2H
or t= + ç ÷ +
g è g ø g

[ Projection from a height at an angle q below horizontal :


ux = u cos q uy = – u sin q
x = (u cos q) t ay = – g q
u
1 X
– H = (- u sin q) t – gt²
2
gt² + (2u sin q) t – 2H = 0 –Y
H ay = – g ¯

(2u sin q )
2
-2u sin q + + 8Hg
t=
2g

2
u sin q æ u sin q ö 2H
or t=- + ç ÷ + R
g è g ø g

Ex. A projectile is fired horizontally with a velocity of 98 ms –1 from the top of a hill 490m high. Find
(i) the time taken to reach the ground (ii) the distance of the target from the hill and (iii) the velocity with which
the projectile hits the ground. (g = 9.8 m/s2)
Sol. (i) The projectile is fired from the top O of a hill with velocity u = 98 ms –1 along the horizontal OX. It reaches the
target P in vertical distance, OA = y = 490 m u = 98 ms–1
O X
1
As y = gt2
2

1 y = 490 m
\ 490 = × 9.8 t2
2
vx
or t = 100 = 10 s A
x b
(ii) Distance of the target from the hill is
vy v
AP = x = horizontal velocity × time = 98 × 10 = 980m.
(iii) The horizontal components of velocity v of the projectile at point P is vx = u = 98 ms–1
vy = uy + gt = 0 + 9.8 × 10 = 98 ms–1 and vertical component

\ v= v 2x + v 2y = 982 + 982 = 98 2 = 138.59 ms–1 .


Now if the resultant velocity v makes angle b with the horizontal, then

58
Motion in a Plane
vy 98
tan b = = =1 or b = 45°
vu 98

Ex. Two tall buildings face each other and are at a distance of 180m from each other. With what velocity must a ball
be thrown horizontally from a window 55m above the ground in one building, so it that enters a window 10m
above the ground in the second building.
( g = 10 ms– 2)
Sol. in figure P and Q are two tall buildings which are 180m apart. W1 and W2 are the two windows in P and Q
respectively. Vertical downward distance to be covered by the ball
= Height of W1 – Height W2 P Q
= 55 – 10 =45m
W1
Initial vertical velocity of ball, uy = 0
1
Q y = uyt + gt2

55 m
2

1 W2
\ 45 = 0 + × 10t2 or t = 3 s. 180m
2 10 m

Horizontal distan ce 180


Required horizontal velocity = = = 60ms -1
Time 3
Ex. Two paper screeens A and B are separated by a distance of 100m. A bullet pierces A and then B. The hole in B
is 10 cm below the hole in A. If the bullet is travelling horizontally at the time of hitting the screen A, calculate
the velocity of the bullet when it hits the screen A.
Sol. Assume that the bullet hits the screen A with velocity u and pierces the screen B after time t.
u Q
100
\ Horizontal distance, PQ = ut = 100m or t = P
u
0.1m
2
1 æ 100 ö
1
Vertical distance, QR = gt = 0.1 or g ç
2 è u ø
÷ = 0.1
2
2 A B
100m

1 10000 9.8 ´ (100)2


g´ = 0.1 Þ u2 = = 49 ´ (100)2 Þ u = 700 ms–1
2 u2 2 ´ 0.1

2H
Alternative method : R = u R = 100m, H = 0.1 m, g = 9.8 m/s2
g

2 ´ 0.1 1 u
100 = u = u = Þ u =7 × 100 = 700 m/s
9.8 49 7

Ex. A ball rolls off top of a stair way with a horizontal velocity u. If each step has h height and b width, the ball will
just hit the edge of nth step. Find the value of n.
Sol. If the ball hits the nth step, the horizontal and vertical distances traversed are nb and nh respectively. Let t be the
time taken by the ball for these horizontal and vertical displacement. Velocity along horizontal direction = u
(remains constant) and initial vertical velocity = zero.

59
Motion in a Plane

1
\ nb = ut and nh = 0 + gt2 u
2 h 1

2 2
1 æ nb ö 2hu2 b
Elminating t from the equation nh = g ç ÷ Þ n= th
n step
2 è u ø gb2

R
Ex. A relief aeroplane is flying at a constant height of 1960 m with speed 600 km/h above the ground towards a point
directly over a person struggling in flood water. At what angle of sight with the verticale should the pilot release
a survival kit if it is to reach the person in water? (g = 9.8m/s2)

5 500
Sol. Plane is flying at a speed = 600 × = m/s horizontally (at a height 1960m)
18 3
A

2h 2 ´ 1960 q
time taken by the kit to reach the ground t = = = 20s
h
g 9.8
x P
500 10, 000
in this time the kit will move horizontally by x = ut= × 20 = m
3 3

x 10,000 10
So the angle of sight tan f = = = = 1.7 ; 3 or f = 60°
h 3 ´ 1960 5.88
Ex. A body falling freely from a given height H hits an inclined plane in his path at a height ‘h’. As a result of this
æhö
impact the direction of the velocity of the body becomes horizontal. For what value of ç H ÷ the body will take
è ø
maximum time to reach the ground ?

2 (H - h)
Sol. Time taken by the body to strike the inclined plane t1 =
g

After impact the velocity becomes horizontal


H–h

2h H
so time taken to reach the ground t2 =
g
h

2
Total time of motion t = t1 + t2 = [ h + (H - h)] g

æ dt ö d 2
For t to be maximum ç dh ÷ = 0 i.e., [h1/ 2 + (H - h)1 / 2 ] =0
è ø dh g

1 –1/2 1 é 2 ù
or h + (H – h)1/2 (–1) = 0 êas ¹ 0ú
2 2 êë g úû

60
Motion in a Plane

h 1
or h = H – h, i.e., =
H 2

Ex. A ball is thrown from the top of a tower with an initial velocity of 10 m/s at an angle of 30° above the horizontal.
It hits the ground at a distance of 17.3 m from the base of tower. Calculate the height of the tower. (g = 10 m/
s2 )
Sol. The angle of projection of the ball is q0 (= 30º) and the
Y
velocity of projection is u (= 10 m/s). Resolving u in
u
horizontal and vertical components, we have uy
horizontal component, 30°
ux
x
ux = u cos q0 = 10 cos 30° = 8.65 m/s
and vertical component (upwards),
uy = u sin 30º = 5.0 m/s h
Tower
If the ball hits the ground after t seconds of projection,
then the horizontal range is
R = ux × t = 8.65 t meter
R=17.3m
R 17.3m
\ t= = = 2.0s
8.65 8.65m / s
If h be the height of the tower, then h = u’y t + 1/2 gt2,
where uy’ is the vertical component (downward) of the velocity of the ball
Here uy ‘ = – uy = – 5.0 m/s and t = 2.0 s
\ h = (– 5.0) × 2.0 + 1/2 × 10 × (2.0)2 = – 10 + 20 = 10 meter.

61
Motion in a Plane

SOLVED EXAMPLE
r
Ex.1 {b g e j}
A body is projected up such that its position vector varies with time as r = 3t $i + 4t - 5t 2 $j m here t is in
seconds. Find the time and x co-ordinate of particle when its y co-ordinate is zero.
4
Sol. If y co-ordinate of particle = 4t – 5t2 = 0 Þ t = s = 0.8s
5
4 12
At that time x = 3 × = m = 2.4 m
5 5

Ex.2 A particle of mass m is projected with velocity v at an angle q with the horizontal. Find its angular
momentum about the point of projection when it is at the highest point of its trajectory.
Sol. At the highest point, particle has only horizontal velcoity vx = vcosq
Length of the perpendicular to the line of action of horizontal velocity
from 'O' is the maximum height, where
2 2
v sin q
Hmax =
2g
Angular momentum J = mvx × Hmax.
m(v cos q)v 2 sin2 q mv 3 sin 2 q cos q
= =
2g 2g

Ex.3 A fighter plane flying horizontally at an altitude of 1.5 km with speed 720 km/h passes directly overhead
an anti-aircraft gun. At what angle from the vertical should the gun be fired for the shell with muzzle
velocity 600 ms–1 to hit the plane ? At what minimum altitude should the pilot fly the plane to avoid being
hit at this angle of projection ? (take g=10ms–2)
Sol. To hit the plane, horizontal speed of plane = horizontal speed of shell

5 1 FG IJ
1
720 ×
18
= 600 sinq Here q is angle of shell with vertical or
3 H K
= sinq or q = sin–1 3

For this angle, maximum height attained by the shell


u 2 cos 2 q 1 8
hmax = , Q sinq = \ cosq =
2g 3 3

hmax =
(600)2 e 8 /3j = 16 km
20
So that to avoid the being hit, minimum height of the plane should be 16km

1 2
Ex.4 A projectile follows the trajectory y = x – x . Determine its range.
2

Comparing with y = x 1 - R ,
FG x IJ æ xö
Sol. H K y = x ç1 - ÷ Þ
è 2ø
R = 2 unit

Ex.5 The motion of a body is described by equations x = 2t and y = 5t2. Calculate its initial speed.
dx dy
Sol. ux = = 2 & uy = = 10 t, At t = 0, v = u 2x + u 2y = 22 + 02 = 2
dt dt

62
Motion in a Plane
Ex.6 A stone is projected from the ground with a velocity of 25 m/s. Two second later, it just clears a wall 5m
high. Find –
(a) The angle of projection of the stone.
(b) The greatest height reaches
(c) How far beyond the wall the stone hits the ground.
Neglect air resistance. Assume g = 10 m/sec2.
Sol. Horizontal component of initial velocity = 25 cosq
vertical component of initial velocity = 25 sinq
(a) Here u = 25 sinq, g = –10 m/s2, h = 5 metre, t = 2 s
1 2 1
Using h = ut + gt , we have 5 = (25 sinq) 2 – × 10 × 4
2 2

1
or 25 = 50sinq, sinq = , q = 30°
2
25
(b) v2 = u2 + 2gh Þ v = 0, u = 25sin30° = , g = –10 m/s2
2
FG 25IJ 2
25 ´ 25
\ 0= H 2K – 2 × 10 × h Þ h=
4 ´ 20
= 7.8 m

2u sin q 2 ´ 25
(c) Total time of flight = = = 2.5 s
g 2 ´ 10
The time taken to clear the wall is two second, hence time in air after clearing the wall = (2.5 – 2)
= 0.5 sec.
Horizontal distance travelled during the interval = (2.5 – 2) = 0.5 s
3
= ucosq × t = 25 cos30° × 0.5 = 25 × × 0.5 = 10.8 m
2

Ex.7 A particle is projected at angle q with speed u from the ground. What is magnitude of change in velocity
and magnitude of change in speed when particle is at the top most point of the path ?
Sol. Velocity at projection point = u cosq î + u sinq ĵ Þ Velocity at maximum point = u cosq î
change in velocity = ( u cos qˆi + u sin qˆj ) – ( u cos qˆj ) = -u sin qˆj
speed at maximum point = u cosq Þ change in speed = u – u cosq

Ex.8 A particle is projected at angle q with speed u from the ground. Calculate its speed when particle is
moving at 90° angle with its initial velocity.
r
Sol. When particle is moving at 90° from the u then velocity
r
v is at 90–q from the ground, as shown. at point A and B, q u 90 – q v
horizontal speed must be same
q
u cos q
u cos q = v sin q Þ v = sin q = u cot q

Ex.9 A boy stands at 78.4 m away from a building and throws

a ball which just enters a window at maximum height 39.2m

above the ground. Calculate the velocity of projection of the ball.


63
Motion in a Plane

u 2 sin 2 q
Sol. Maximum height Hmax = = 39.2 m ............. (1)
2g

u 2 sin 2q 2u 2 sin q cos q


Range R = = = 2 × 78.4 ............. (2)
g g

Dividing (1) by (2) we get tanq = 1 Þ q = 45°

u 2 sin 90°
Substituting in (2) , we get = 2 × 78.4 Þ u= 2 ´ 78.4 ´ 9.8 = 39.2 m/s
g

Ex.10 A mass slides down a very smooth curved track

as shown. At what horizontal distance from the


end of the track does it hit the ground ?

1
Sol. Change in P.E. = Change in K.E. mg [3–1] = mv2 Þ v = 2 g
2

vertical component of velocity at leaving end = vsin 30° = g

1 1 2 1 2 2 2
Q Sy = uyt + a t2 \ –1 = gt– gt Þ gt – g t – 1 = 0 Þ t2 – g t – =0
2 y 2 2 g

1 3 3 +1
Þ t= + =
g g g

Horizontal displacement = Horizontal velocity × time

LM 3 +1 OP
PQ = e3 + 3 j m = 4.73 m.
3
Þ
x = 2 g cos30° × t x= 2 g ×
2
×
MN g

Ex.11 A particle starts from origin at t = 0 with a velocity 5.0 $i m/s and move in x-y plane under action of a force
which produces a constant acceleration of (3.0 $i + 2.0 $j ) m/s2. (a) What is the y-coordinate of the
particle at the instant its x-coordinate is 84 m? (b) What is the speed of the particle at this time?
Sol. (a) u = 5.0 î m/s, a = (3.0iˆ + 2.0j)
ˆ m / s2

1 2
at time t, x-coordinate of particle x = uxt + at
2 x

1 28
84 = 5t + × 3t2 Þ 3t2 + 10t – 168 = 0 Þ 3t2 + 28t – 18t – 168 = 0 Þ t = 6s, - s
2 3
so time t = 6s
1 2 1
y coordinate at t = 6s y = uyt + at =0+ × 2 × (6)2 = 36m
2 y 2
(b) velocity of particle at time t
r
v = (u x + a x t)iˆ + (u y + a y t)ˆj = (5 + 3 × 6) î + (0 + 2 × 6) ĵ = (23iˆ + 12j)
ˆ

speed = (23)2 + (2)2 = 25.94 ms


–1

64
Motion in a Plane
Ex.12 The celling of a long hall is 25 m high. What is the maximum horizontal distance that a ball thrown with a
speed of 40 ms–1 can go without hitting the celling of the hall?
u 2 sin2 q 25 ´ 2 ´ 10 5
Sol. hmax = = 25 Þ sin2q = =
2g 40 ´ 40 16 40ms
–1

25m
5 11 q
Þ sinq = Þ cosq =
4 4

u 2 2 sin q cos q ( ) ´ 2 11
2
5
Rmax = = 40 ´ ´ = 148.3 m
g 10 4 4

Ex.13 Two particles A and B are projected from the ground simultaneously in vA=20 m/s vB=10 m/s
the directions shown in the figure with initial velocities vA = 20 m/s
and vB =10 m/s respectively. They collide after 0.5 s. Find out
(i) the angle q and (ii) the distance x.
Sol. Both particle will collide if they are at same height in same time.
yA = yB [Let upward be positive direction]
1 2 1
or (uy)At – gt = (uy)Bt – gt2
2 2

1
or (uy)A = (uy)B or (vA sin q) = vB or 20 sin q = 10 or sinq = Þ q = 30°
2
In 0.5s horizontal distance covered by A is x. or x = (ux)At
3
x = (20 cos30°)0.5 or 10 × = 5 3m
2
Ex.14 Two bodies are thrown simultaneously from the same point, one straight up and the other at an angle of q
= 30° from horizontal, the initial velocity of each body is equal to v0 = 20 ms–1. Neglect air resistance, find
the distance between the bodies at t = 1.2 s.
1 2 1
Sol. y = (20 × 1.2 – gt ) – (20 sin 30° × 1.2 – gt2) = 24 – 12 = 12 m
2 2

3
x = 20 cos30° × 1.2 = 24 × Þ 12 3 m
2
30°
distance between two bodies = (12)2 + (12 3 )2 = 24 m

Ex.15 Two particles are projected from the two towers


simultaneously, as shown in the figure. What should
be value of ‘d’ for their collision.
Sol. Their is no relative acceleration of B w.r.t. A.

y BA
So time of collision t = Where yBA = vertical displacement of B w.r.t. A = 10 m. &
dv i
y BA

10
(vy)BA = vertical velocity of B w.r.t. A = 0 – (–10 2 sin45°) = 10 m/s Þ t = = 1s
10

d = horizontal distance travelled by B w.r.t. A = (vx)BA × t = (10+10 2 cos45°) × 1 = 20 m.


65
Motion in a Plane

NCERT BASED QUESTION


1. What in meant by projectile motion ? Give examples.

2. A ball is projected with a velocity u making an angle q with the horizontal. Find the maximum height, the time of
flight and the horizontal range.

3. Find the equation of the trajectory of a projectile fired in the horizontal direction. What is the velocity of the
projectile at any point on the trajectory?

4. At what points on the projectile trajectory is the speed : (a) minimum (b) maximum ?

5. At what points on the projectile trajectory is the : (a) potential energy maximum (b) kinetic energy minimum and
(c) total energy maximum ?

6. A projectile is projected at an angle of 15° to the horizontal with speed v. If another projectile is projected with
the same speed, then at what angle with the horizontal it must be projected so as to have the same range.

7. Discuss the effect of air resistance on projectile motion.

8. A projectile is thrown horizontally with a velocity 'v' from the top of a tower of height 'h'. Calculate the velocity
of the projectile when it strikes the ground.

9. Two tall buildings are 200 m apart. With what speed must a ball be thrown horizontally from a window 540 m
above the ground in one building so as to enter a 50 m high window in the other building ?

10. A bullet is fired at angle of 60° above the horizontal with a speed of 98 3 m/s. Find the velocity of the bullet
after 6s.

11. Galileo, in his book Two new sciences, stated that “for elevations which exceed or fall short of 45° by equal
amouts, the ranges are equal”. Prove this statement.

12. A hiker stands on the edge of a cliff 490 m above the ground and throws a stone horizontally with an initial speed
of 15 m s–1. Neglecting air resistance, find the time taken by the stone to reach the ground, and the speed with
which it hits the ground. (Take g = 9.8 m s–2 ).

13. A cricket ball is thrown at a speed of 28 m s-1 in a direction 30° above the horizontal. Calculate (a) the maximum
height, (b) the time taken by the ball to return to the same level, and (c) the distance from the thrower to the point
where the ball returns to the same level.

14. The ceiling of a long hall is 25 m high. What is the maximum horizontal distance that a ball thrown with a speed
of 40 m s–1 can go without hitting the ceiling of the hall ?

15. A cricketer can throw a ball to a maximum horizontal distance of 100 m. How much high above the ground can
the cricketer throw the same ball ?

16. A bullet fired at an angle of 30° with the horizontal hits the ground 3.0 km away. By adjusting its angle of
projection, can one hope to hit a target 5.0 km away ? Assume the muzzle speed to be fixed, and neglect air
resistance.

66
Motion in a Plane

17. A fighter plane flying horizontally at an altitude of 1.5 km with speed 720 km/h passes directly overhead an anti-
aircraft gun. At what angle from the vertical should the gun be fired for the shell with muzzle speed 600 m s–1 to
hit the plane ? At what minimum altitude should the pilot fly the plane to avoid being hit ?
(Take g = 10 m s–2 ).

18 (a) Show that for a projectile the angle between the velocity and the x-axis as a function of time is given by

æ v - gt ö
q(t) = tan -1 ç 0y ÷
è v ox ø

(b) Shows that the projection angle q0 for a projectile launched from the origin is given by

æ 4h ö
q0 = tan -1 ç m ÷
è R ø

where the symbols have their usual meaning.

67
Motion in a Plane

HINT & SOLUTION


DO IT YOUR SELF : Q1 to Q10 are theory and formula based Question
11. For a projectile launched with velocity vo at an angle qo, the range is given by

v 20 sin 2 q°
R=
g

Now, for angles, (45° + a) and (45° – a), 2q is (90° + 2a) and (90° – 2a), respectively. The values of sin (90°
+ 2a) and sin (90° – 2a) are the same, equal to that of cos2a. Therefore, ranges are equal for elevations which
exceed or fall short of 45° by equal amounts a.
12. We choose the origin of the x-,and y-axis at the edge of the cliff and t = 0 s at the instant the stone is thrown.
Choose the positive direction of x-axis to be along the initial velocity and the positive direction of y-axis to be the
vertically upward direction. The x-, and y-components of the motion can be treated independently. The equations
of motion are :
x (t) = xo + vox t
y (t) = yo + voy t + (1/2) ay t2
Here, xo = yo = 0, voy = 0, ay = –g = –9.8 m s–2,
vox = 15 m s–1.
The stone hits the ground when y(t) = –490 m.
–490 m = –(1/2)(9.8) t2.
This gives t =10 s.
The velocity components are vx = vax and
vy = voy – g t
so that when the stone hits the ground :
vox = 15 m
voy = 0 – (9.8) (10) = –98 m s–1
Therefore, the speed of the stone is

v 2x + v 2y = 152 + 982 = 99ms-1

13. (a) The maximum height is given by

v 20sin2 q (28 ) ( sin30 )


2 2

hm = = m
2g 2 ( 9.8 )

14 ´ 14
= = 10.0 m
2 ´ 9.8
(b) The time taken to return to the same level is
Tf = (2 vo sinqo)/g = (2 × 28 × sin 30°)/9.8
= 28/9.8 s = 2.9 s
(c) The distance from the thrower to the point where the ball returns to the same level is

v 20 sin 2 q° ( 28 ) sin 60°


2

R= = = 69m
g 9.8
68
Motion in a Plane
14. Speed of the ball, u = 40 m/s
Maximum height, h = 25 m
In projectile motion, the maximum height reached by a body projected at an angle q, is given by the relation:

u 2 sin2 q
h=
2g

(40)2 sin2 q
25 =
2 ´ 9.8

sin2q = 0.30625
sinq = 0.5534
\ q = sin–1 (0.5534) = 33.60°

u 2 sin 2q
Horizontal range, R =
g

(40)2 ´ sin (2 ´ 33.60 ) 1600 ´ sin 67.2 1600 ´ 0.922


= = = = 150.53 m
9.8 9.8 9.8

15. Maximum horizontal distance, R = 100 m


The cricketer will only be able to throw the ball to the maximum horizontal distance when the angle of projection
is 45°, i.e., q = 45°.
The horizontal range for a projection velocity n, is given by the relation :
u 2 sin 2q
R=
g

u2
100 = sin90°
g

u2
= 100 .........(i)
g
The ball will achieve the maximum height when it is thrown vertically upward. For such motion, the final velocity
n is zero at the maximum height H.
Acceleration, a = –g
Using the third equation of motion :
n2 – u2 = – 2gH

1 u2 1
H= ´ = ´ 100 = 50 m
2 g 2

16. Answer: No
Range, R = 3 km
Angle of projection, q = 30°
Acceleration due to gravity, g = 9.8 m/s2
Horizontal range for the projection velocity u0, is given by the relation :

u 20 sin2q
R=
g

69
Motion in a Plane
2
u
3= 0
sin 60°
g

u20
2 3 .......(i)
g

The maximum range (Rmax) is achieved by the bullet when it is fired at an angle of 45° with the horizontal, that
is,

u20
R max = .......(ii)
g

On comparing equations (i) and (ii), we get :

Rmax = 3 3 = 2 × 1.732 = 3.45 km


Hence, the bullet will not hit a target 5 km away.
17. Height of the fighter plane = 1.5 km = 1500 m
Speed of the fighter plane, n = 720 km/h = 200 m/s
Let q be the angle with the vertical so that the shell hits the plane. The situation is shown in the given figure.
–1
Muzzle velocity of the gun, u = 600 m/s 200 ms

Time taken by the shell to hit the plane = t


Horizontal distance travelled by the shell = uxt 1500 m
uy q u
Distance travelled by the plane =nt
The shell hits the plane, Hence, these two distances must ux

be equal.
uxt = nt
u sin q = n

n 200 1
sin q = = = = 0.33
u 600 3
q = sin–1(0.33)
= 19.5°
In order to avoid being hit by the shell, the pilot must fly the plane at an altitude (H) higher than the maximum
height achieved by the shell.

u 2 sin2 (90 - q)
\ H=
2g

(600)2 cos2 q
=
2g

360000 ´ cos2 19.5


=
2 ´ 10
= 18000 × (0.943)2
= 16006.482 m
» 16 km

70
Motion in a Plane
18. Let v0x and v0y respectively be the initial components of the velocity of the projectile along horizontal (x) and
vertical (y) directions.
Let vx and vy respectively be the horizontal and vertical components of velocity at a point P.
y
Time taken by the projectile to reach point P = t
Applying the first equation of motion along the vertical vy
a
v0
and horizontal directions, we get: voy= v0sinq 0 P vx
q0
x
vy = v0y – gt vox= v0cosq0

And vx = v0x

y n n 0 y - gt
\ tan a = n = n 0x
x

æ n0y - gt ö
a = q(t) = tan -1 = ç ÷
è n0x ø

v 20 sin2 q
Maximum vertical height, h m = .......(i)
2g

v 20 sin 2q
Horizontal range, R = .......(ii)
g

Solving equations (i) and (ii), we get :

hm sin2 q
=
R 2 sin2q

sin q ´ sin q
= =
2 ´ 2 sin q cos q

1 sin q 1
= = tan q
4 cos q 4

æ 4h ö
tan q = ç m ÷
è R ø

æ 4h ö
q = tan -1 ç m ÷
è R ø

71
Motion in a Plane

CIRCULAR MOTION

When a particle moves in a plane such that its distance from a fixed point remains constant then its motion is
called as circular motion with respect to that fixed point.
That fixed point is called centre and the distance is called radius of circular path.

RADIUS VECTOR
The vector joining the centre of the circle and the center of the particle performing circular motion is called
radius vector. It has constant magnitude and variable direction.

KINEMATICS OF CIRCULAR MOTION


Angular Displacement
Angle traced by position vector of a particle moving w.r.t. some fixed
point is called angular displacement. Q
fixed
Dq = angular displacement point Dq
Arc Arc (PQ) r
P
Angle = Þ Dq =
Radius r

GOLDEN KEY POINTS


r
• Small Angular displacement dq is a vector quantity, but large angular displacement q is scalar quantity..
® ® ® ® ® ® ® ®
dq1 + dq2 = dq2 + dq1 But q1 + q2 ¹ q2 + q1

• Its direction is perpendicular to plane of rotation and given by right hand screw rule.
• It is dimensionless and has S.I. unit is "Radian" while other units are degree or revolution.
2p radian = 360° =1 revolution

Frequency (n)
Number of revolutions describes by particle per second is its frequency. Its unit is revolutions per second (r.p.s.)
or revolutions per minute (r.p.m.)
Time Period (T)

1
It is time taken by particle to complete one revolution. T =
n
Angular Velocity (w)
It is defined as the rate of change of angular displacement of moving particle .

Angle traced Dq dq
w= = Lim =
Time taken Dt ®0 Dt dt

GOLDEN KEY POINTS


• It is an axial vector quantity.
• Its direction is same as that of angular displacement i.e. perpendicular to the plane of rotation and along the axis
according to right hand screw rule.
• Its unit is radian/second.

72
Motion in a Plane
Relation between linear and Angular velocity Q
r
Arc Ds
Angle = or Dq = or Ds = rDq
Radius r Dq Ds

r
Ds rDq ds dq
\ = if Dt ® 0 then =r v = wr P
Dt Dt dt dt
® ® ® ®
v = w´ r (direction of v is according to right hand thumb rule)

Average Angular Velocity (wav )

total angle of rotation q2 - q1 Dq 2p


wav = = t -t = = = 2pn
total time taken 2 1 Dt T

where q1 and q2 are angular position of the particle at instant t1 and t2.

Instantaneous Angular Velocity


r
Dq dq r dq
The angular velocity at some particular instant w = Lim = or w=
Dt ®0 Dt dt dt

Relative Angular Velocity


Relative angular velocity of a particle 'A' w.r.t. other moving particle 'B' is the
angular velocity of the position vector of 'A' w.r.t. 'B'. That means it is the rate
at which position vector of 'A' w.r.t. 'B' rotates at that instant

( v AB ) ^ Relative velocity of A w.r.t. B perpendicular to line AB


wAB = rAB
=
seperation between A and B

v A sin q1 + v B sin q2
here (vAB)^ = vA sin q1 + vB sin q2 \ wAB =
r
Angular Acceleration (a)
r
Dw dw r dw
Lim
Rate of change of angular velocity is called angular acceleration. a = Dt ®0 = or a =
Dt dt dt
Its an axial vector quantity. It direction is along the axis according to right hand screw rule.
Unit ® rad/s2
Relation between Angular and Linear Acceleration vr = wr ´ rr
( vr is a tangential vector, wr is a axial vector and rr is a radial vector.) These three vectors are mutually
perpendicular.
r r r axis of rotation
r dv d r r dw r r dr
but a= = (w ´ r ) = ´ r + w´ w
dt dt dt dt
r r v
r r r r r dw r dr r
or a = a ´ r + w´ v ( = a and =v)
dt dt r
r r r r r r r r r
or a = aT + aC ( a T = a ´ r is tangential acc. and a C = w´ v is centripetal acceleration.)
r r r r r
a = aT + aC ( a T and a C are two component of net linear acceleration.)

73
Motion in a Plane
Tangential Acceleration
r
a T = ar ´ rr , its direction is parallel to velocity..
r r r
r
v = w´ r and a T = ar ´ rr
r r
as wr and ar both are parallel and along the axis so that v and a T are also parallel and along the tangential
direction.
Magnitude of tangential acceleration, aT = a r sin 90° = ar
( ar is axial, rr is radial so that ar ^ rr )
r r
As a T is along the direction of motion (in the direction of vr ) so that a T is responsible for change in speed of the
particle. Its magnitude is rate of change of speed of the particle. If particle is moving on a circular path with
constant speed then tangential acceleration is zero.

axis of rotation
Centripetal acceleration w
r r r r r r r r r
aC = w ´ v Þ a C = w ´ (w ´ r) (Q vr = w ´r) r
O P
Let rr is in î direction and wr is in ĵ direction
r ˆj ´ (ˆj ´ ˆi) ˆ
then direction of a C is along or ĵ ´ ( -k) or – î
r
opposite direction of r i.e., from P to O and it is centripetal direction.

v2 r v2
Magnitude of centripetal acceleration, a C = wv = = w2 r aC = ( -ˆr)
r r

GOLDEN KEY POINTS


• Centripetal acceleration is always perpendicular to the velocity or displacement at each point. So that work
done by centripetal force is always zero.
• When a force acts always perpendicular to the direction of velocity then path decribed by the particle is
circular.

Equations of Motion
If a particle moves along a circle with constant angular acceleration, then following equations of motion can be
used to calculate instantaneous angular displacement and angular velocity
1
(i) w = w0 + at (ii) q = w0t + at2 (iii) w2 = w02 + 2aq
2

æ w0 + w ö a
(iv) q= ç ÷t (v) qn = w0 + (2n – 1)
è 2 ø 2
where, w0 = Initial angular velocity, w = Instantaneous angular velocity
a = Angular acceleration,q = Angular displacement at time t, qn = Angular displacement in nth second

Ex. A particle is revolving in a circular path completes first one third of circumference in 2 s, while next one third in
1 s. Calculate the average angular velocity of particle.
2p 2p
Sol. q1 = and q2 = total time T = 2 + 1 = 3 s
3 3

2p 2p 4p
q1 + q2 + 4p
\ < wav > =
T = 3 3 = 3 =
9
rad/s
74 3 3
Motion in a Plane

1
Ex. The angular displacement of a particle is given by q = wot + at2 , where wo and a are constant and wo = 1 rad/s, a
2
= 1.5 rad/s2. Find the angular velocity at time t = 2s.

dq
Sol. w = = w0 + at = 4 rad/s
dt

Ex. Two moving particles P and Q are 10 m apart at any instant.


Velocity of P is 8 m/s at 30°, from line joining the P and Q and velocity of
Q is 6m/s at 30°.Calculate the angular velocity of P w.r.t. Q

8 sin 30o - ( -6 sin 30o )


Sol. wPQ = = 0.7 rad/s.
10

Ex. A particle is moving parallel to x-axis as shown in figure such that at all instant
the y-axis component of its position vector is constant and is equal to 'b'. Find
the angular velocity of the particle about the origin.

v sin q v
Sol. \ wPO = 2
= sin q
b b
sin q

Ex. Two points of a rod move with velocity 3v and v seperated by a distantce 'r'. Calculate the angular velocitry of
the rod w.r.t. its end.

3v - v 2v
Sol. \ w = =
r r

Ex. The angular velocity of a particle is given by w =1.5 t – 3t2 +2, Find the time when its angular acceleration
becomes zero.

dw
Sol. a = = 1.5 – 6 t = 0 or t = 0.25 s.
dt

Ex. A disc starts from rest and on the application of a torque, it gains an angular acceleration given
by a = 3t – t2 . Calculate the angular velocity after 2 s.

w t
dw 3t 2 t3 10
ò ò
2
Sol. = 3t-t2 Þ dw = (3t - t )dt Þw= - Þ at t =2 s, w= rad/s
dt 0 0 2 3 3

75
Motion in a Plane
CENTRIPETAL FORCE
When a particle moves along a curved path, a force is required to change the direction of motion of the particle
The force which is required to change the direction of motion is called centripetal force.

mv 2
Fc = Q v=rw \ Fc = mw2 r
r

[ Direction of centripetal force is always perpendicular to the velocity of particle.


[ It can change the direction of motion but it can not change speed of the particle.
[ Work done by centripetal force is always zero (whether particle completes the loop or not)
[ If net force on the particle is perpendicular to its velocity than particle moves along a circular path with
constant speed. This motion is called uniform circular motion
NOTE :
Since circular motion is an acceleration motion, thus a pseudo force acts on the particle which is opposite to
centripetal acceleration (away from the centre of circle). This pseudo force is called centrifugal force.

mv 2
Centrifugal force =
r

UNIFORM CIRCULAR MOTION


When a particle moves in a circle at a constant speed then the motion is said to be a uniform circular motion.
In such motion, position vector keep changing. v2
r
Speed is constant, so that a T = 0 v1

r r r r v r2
Acceleration of particle a = a C = w ´ v or a = w v ( but w = ) r1
r
O
v2
\ a= = w2 r = centripetal acceleration
r
due to centripetal acceleration as a result the velocity of the particle keeps on changing the direction i.e. the
particle is accelerated.

GOLDEN KEY POINTS


r r r r
• Position vector (r) is always perpendicular to the velocity vector (v) i.e. r. v = 0
r r
• velocity vector is always perpendicular to the acceleration. v. a = 0
• for circular motion force towards centre (Centripetal force) must act so that direction of vr keeps on changing
which forces the particle to describe a circular path.
• The work done by centripetal force is always zero.
ac
• Kinetic Energy = constant
• Q r \
|v| = constant so tangential acc. a T=0 FT = 0 aT = 0
• Important difference between the projectile motion and uniform circular motion :
In projectile motion, both the magnitude and the direction of acceleration (g) remain constant, while in uniform
circular motion the magnitude remains constant but the direction continuously changes.

76
Motion in a Plane
Ex. A body of mass 2 kg lying on a smooth surface is attached to a string 3 m long and then rotated in a horizontal
circle making 60 rev/min. Calculate the centripetal acceleration.

2p
Sol. w = 60 ´ = 2p rad/s \ ac = w2 r = 118.4 m/s2
60

Ex. A stone of mass 0.1 kg tied to one end of a string 1m long is revolved in a horizontal circle at the rate of

10
rev/s. Calculate the tension in the string.
p

2
æ 10 ö
Sol. In horizontal circular motion tension T = mw r = (0.10) ç ´ 2 p ÷ ´ 1 = 40 N
2
è p ø

Ex. Two balls of equal mass are attached to a string at distances 1m and 2m from one end as shown.The string with
the masses is then moved in a horizontal circle with constant speed. What is the ratio of the tension T 1 and T2 ?
Sol. at T1 ¬¾ P
®T
2

T1 - T2 = mw2 and T2 = mw2 O


T1 P T
2 Q
T1 3
\ T1 = T2 + mw2 = 3 mw2 \ =
T2 2

Ex. One meter long string can bear maximum of 0.5 kg mass. A mass of 0.05 kg is tied to one of its end and rotated
in a horizontal circle, calculate the maximum number of revolution so that string does not brakes (rev/min.)
Sol. m w2 r = mbg = 0.5 × 9.8 \ w= 98 = 2pn Þ n = 1.576 rev/s = 94.5 rev/min.

CIRCULAR MOTION IN HORIZONTAL PLANE


Conical Pendulum
A conical pendulum consits of a body attached to a string of length L, such that it can revolved a horizontal circle
with uniform speed. The string traces out a cone in the space.
forces acting on the bob are :
(i) tension in string = T (ii) weight of bob = mg

mv 2
T sin q = T cos q = mg
r

v2
tan q = \ v= rg tan q
rg

2pr 2pr
\ Time period = Þ (Time Period) T=
v rg tan q

r L cos q OP r
= 2p = 2p (Q in D OSP,, = sin q or = SP = L )
g tan q g SP sin q

77
Motion in a Plane
NON- UNIFORM CIRCULAR MOTION
When a particle moving in a circle and if the speed of particle increases or decreses then the motion is non-
uniform circular motion.
r
In non- uniform circular motion |v|¹ cons tan t w ¹ constant
In non-uniform circular motion particle has two acceleration :
(a) Tangential acceleration
dv ds
aT = = rate of change of speed; v = = speed; s = arc-length
dt dt
Tangential force FT = maT
v2
(b) Centripetal acceleration ac = = w2 r
r

mv 2
Centripetal force Fc = = mw2 r
r
r r r
Net acceleration of the particle a = ac + a T Þ a = a2C + a2T
r r r
and net force F = Fc + FT

FT æ FT ö -1 a T
If 'q ' is the angle made by net force ( F ) with Fc , then tanq = F \ q = tan–1 ç ÷ = tan
c è Fc ø aC

[ In both uniform and non- uniform circular motion fc is perpendicular to velocity. So work done by centripetal
force will be zero in both the cases.
[ In uniform circular motion Ft = 0 as aT = 0
But in non- uniform circular motion FT ¹ 0. Thus there will be work done by tangential force in this case.

Ex. A road makes a 90° bend with a radius of 190 m. A car enters the bend moving at 20 m/s. Finding this
too fast, the driver decelerates at 0.92 m/s². Determine the acceleration of the car when its speed rounding
the bend has dropped to 15 m/s.
Sol. Since it is rounding a curve, the car has a radial acceleration associated with its changing direction, in addition
to the tangential deceleration that changes its speed. We are given that a T = –0.92 m/s², since the car
is slowing down, the tangential acceleration is directed opposite the velocity. a a r

2 2
v (15)
The radial acceleration is ar = = = 1.2 m / s²
r 190
2 1/2 q
Magnitude of net acceleration. a = a2r + a2T = [(1.2)² + (0.92) ] = 1.5 m/s² aT

æa ö æ 1.2 ö
and points at an angle q = tan-1 ç r ÷ = tan-1 ç = 53°
è aT ø è 0.92 ÷ø
relative to the tangent line to the circle.
Ex. The kinetic energy of a particle moving along a circle of radius r depends on distance covered s as
2
K = As where A is a constant. Find the force acting on the particle as a function of s.
1
Sol. According to given problem mv2 = As2
2

78
Motion in a Plane

2A
or v=s ...(i)
m

v2 2As2
So ar = = ...(ii)
r mr

dv dv ds dv
Further more as a T = = . = v ...(iii)
dt ds dt ds

n æ 2A ö dv 2A
from eq . (1), v = s ç ÷ Þ = ...(iv)
è m ø ds m

n n n é 2A ù é 2A ù 2As
Substitue values from eq . (i) and eq . (iv) in eq . (iii) a T = ês ú ê ú=
ëê m ûú ëê m ûú m

2 2 2
é 2As 2 ù 2
é 2As ù 2As ésù ésù
So a = a 2
+ a =
2 ê ú + ê m ú = 1+ ê ú so F = ma = 2As 1+ê ú
r T
êë mr úû ë û m ër û ër û

Ex. A particle of mass m is moving in a circular path of constant radius r such that its centripetal
2 2
acceleration is varying with time t as ac = k rt , where k is a constant. Determine the power delivered to
particle by the forces acting on it.
v2 v2
Sol. If v is instaneous velocity, centripetal accelration a c = \ = k2 rt2 Þ v = krt
r r
In circular motion work done by centritpetal force is always zero & work is done only by tangential force.

dv d
Q Tangent acceleration a T = = (krt) = kr
dt dt

\ Tangential force FT = ma T = mkr


2 2
Power P = FTv = (mkr) (krt) = mk r t

79
Motion in a Plane

MH BOARD QUESTIONNAIRE EXERCISE - I

*1. An aeroplane has a run of 500 m to take off from the runway. It starts from rest and moves with constant
acceleration to cover the runway in 30 sec. What is the velocity of the aeroplane at the take off ?
[Ans: 120 km/hr]
*2. A car moving along a straight road with a speed of 120 km/hr, is brought to rest by applying brakes. The
car covers a distance of 100 m before it stops. Calculate (i) the average retardation of the car (ii) time
taken by the car to come to rest. [Ans: 50/9 m/sec2, 6 sec]
*3. A car travels at a speed of 50 km/hr for 30 minutes, at 30 km/hr for next 15 minutes and then 70 km/hr
for next 45 minutes. What is the average speed of the car? [Ans: 56.66 km/hr]
*4. A velocity-time graph is shown in the adjoining figure.

Determine:
(i) initial speed of the car (ii) maximum speed attained by the car (iii) part of the graph showing zero
acceleration (iv) part of the graph showing constant retardation (v) distance travelled by the car in first 6
sec. [Ans: (i) 0 (ii) 20 m/sec (iii) AB (iv) BC (v) 90 m]
*5. A man throws a ball to maximum horizontal distance of 80 meters. Calculate the maximum height reached.
[Ans: 20 m]
*6. A particle is projected with speed v0 at angle q to the horizontal on an inclined surface making an angle
f(f < q) to the horizontal. Find the range of the projectile along the inclined surface.

2v02 cos q sin(q - f)


[Ans: R = ]
g cos 2 f
*7. A metro train runs from station A to B to C. It takes 4 minutes in travelling from station A to station B.
The train halts at station B for 20 s. Then it starts from station B and reaches station C in next 3 minutes.
At the start, the train accelerates for 10 sec to reach the constant speed of 72 km/hr. The train moving at
the constant speed is brought to rest in 10 sec. at next station. (i) Plot the velocity- time graph for the train
travelling from the station A to B to C. (ii) Calculate the distance between the stations A, B and C.
[Ans: AB = 4.6 km, BC =3.4 km]
*8. A train is moving eastward at 10 m/sec. A waiter is walking eastward at 1.2m/sec; and a fly is flying
toward the north across the waiter’s tray at 2 m/s. What is the velocity of the fly relative to Earth
[Ans: 11.4 m/s, 10° due north of east]
*9. A car moves in a circle at the constant speed of 50 m/s and completes one revolution in 40 s. Determine
the magnitude of acceleration of the car. [Ans: 7.85 m s-2 ]
*10. A particle moves in a circle with constant speed of 15 m/s. The radius of the circle is 2 m. Determine the
centripetal acceleration of the particle. [Ans: 112.5 m s-2 ]
*11. A projectile is thrown at an angle of 30° to the horizontal. What should be the range of initial velocity (u)
so that its range will be between 40m and 50 m? Assume g = 10 m s-2. [Ans: 21.49 £ u £ 24.03ms-2 ]

80
Motion in a Plane
MHT-CET QUESTIONNAIRE EXERCISE - II
*1. For uniform acceleration in rectilinear motion (A) must be zero
which of the following is not correct? (B) may or may not be zero
(A) Velocity-time graph is linear (C) cannot be zero
(B) Acceleration is the slope of velocity time (D) depends upon the particle
graph. 10. If the distance covered is zero, the displacemnt :–
(C) The area under the velocity-time graph (A) must be zero
equals displacement.
(B) may or may not be zero
(D) Velocity-time graph is nonlinear.
(C) cannot be zero
*2. If three particles A, B and C are having velocities
r r r (D) depends upon the particle
v A , v B and v C , which of the following formula
11. An athlete completes half a round of a circular track
gives the relative velocity of A with respect to
of radius R, then the displacement and distance
B?
r r r r r covered by the athlete are :–
(A) v A + v B (B) v A +
vC + v B
r r r r (A) 2R and pR (B) pR and 2R
(C) v A - v B (D) v C +
vA
*3. An object thrown from a moving bus is an (C) R and 2pR (D) 2pR and R
example of 12. The location of a particle is changed. What can we
(A) Uniform circular motion say about the displacement and distance covered
(B) Rectilinear motion by the particle :–
(C) Projectile motion (A) Both cannot be zero
(D) Motion in one dimension (B) One of the two may be zero
*4. For a particle having a uniform circular motion, (C) Both must be zero
which of the following is constant (D) If one is positive, the other is negative and vice-
(A) Speed (B) Acceleration versa
(C) Velocity (D) Displacement 13. A person moves on a rectangular path of length 3
*5. The bob of a conical pendulum undergoes. m & breadth 4 m, then find the ratio of distance &
(A) Rectilinear motion in horizontal plane displacement when it reaches at diagonally opposite
(B) Uniform motion in a horizontal circle point :-
(C) Uniform motion in a vertical circle (A) 7/5 (B) 1 (C) 5/7 (D) None
(D) Rectilinear motion in vertical circle 14. A ball is thrown upward to a heigth of 15 m and
return back to thrower. Find the distance travelled
6. An athlete completes one round of a circular track
by the ball :-
of radius R in 40 seconds. What will be his
(A) 0 (B) 30 m (C) 60 m (D) 15 m
displacement at the end of 2 minute 20 second? 15. Velocity of a particle (starting at t = 0) varies with
(A) Zero (B) 2R (C) 2pR (D) 7pR time as v = 4t. Calculate the displacement of particle
7. A body goes to 10 km north and 20 km east. The between t = 1 to t = 5 sec :-
displacement from initial point is :– (A) 12 m (B) 24 m (C) 48 m (D) 96 m
16. A particle starts from origin & moves towards
(A) 22.36km(B) 2 km (C) 5 km (D) 20 km
positive y-axis to the point (0, 20m) & return back
8. The numerical ratio of displacement to distance is
to the point (0, 5m). The ratio of distance travelled
(A) always < 1 (B) always = 1 to the magnitude of displacement is :-
(C) always > 1 (D) £ 1 (A) 4 : 1 (B) 1 : 4 (C) 7 : 1 (D) 1 : 7
9. If displacement of a particle is zero, the distance 17. From the top of a tower, a stone A is thrown upward
& a stone B is thrown downward with the same
covered :–
speed. The velocity of stone A, on colliding with the
81
Motion in a Plane
ground is :-
(A) Greater than the velocity of B (A) 2t(c + b) (B) 2t c2 - b2
(B) Less than the velocity of B
(C) Equal to the velocity of B (C) t c2 + b2 (D) 2t c2 + b2
(D) Both the stone will fall on the earth at the same
25. A particle moves on a circular path with constant
time
speed 15 m/s. Find the change in speed & magnitude
18. Displacement of a particle is given by x = 6t2 – 24t, where
of change in velocity, when particle completes half
t is in second then velocity will be zero at :-
revolution :-
(A) t = 0 (B) t = 1 sec
(C) t = 2 sec (D) t = 4 sec (A) 30 m/s, 30 m/s (B) 0, 30 m/s
19. A person travels along a straight road for the first half (C) 0, 15 m/s (D) 0, 0
time with a velocity v1 and the second half time with a 26. The displacement x of a particle along x-axis is given
velocity v2. Then the mean velocity v is given by :– by x = 3 + 8t + 7t 2 . Obtain its velocity and
acceleration at t = 2 sec :-
v1 + v 2 2 1 1
(A) v = (B) v = v + v (A) 18 m/s, 14 m/s2 (B) 36 m/s, 14 m/s2
2 1 2

v2 (C) 18 m/s, 7 m/s2 (D) 36 m/s, 7 m/s2


(C) v = v1 v 2 (D) v = 27. The velocity of a particle is given by the equation
v1
v = 3t2 + 4 m/s. Find the average acceleration during
20. A train covers the first half of the distance between
the time interval between t = 1s to t = 2s. :-
two stations at a speed of 40 km/h and the other
(A) 3 m/s2 (B) 6 m/s2
half at 60 km/h. Then its average speed is :–
(C) 9 m/s2 (D) 12 m/s2
(A) 50 km/h (B) 48 km/h
28. A 100 m long train crosses a man travelling at
(C) 52 km/h (D) 100 km/h
5 km/hr, in opposite direction, in 7.2 seconds then
21. A particle moves for 20 seconds with velocity
the velocity of train is :–
3 m/s and then moves with velocity 4 m/s for another
20 seconds and finally moves with velocity 5 m/s (A) 40 km/hr (B) 25 km/hr
for next 20 seconds. What is the average velocity (C) 20 km/hr (D) 45 km/hr
of the particle ? 29. A train of 150m length is going towards north
(A) 3 m/s (B) 4 m/s (C) 5 m/s (D) zero direction at a speed of 10 ms–1. A parrot flies at a
22. An object travels 10 km at a speed of 100 m/s and speed of 5ms–1 towards south direction parallel to
another 10 km at 50 m/s. The average speed over the railway track. The time taken by the parrot to
the whole distance is :– cross the train is equal to :–
(A) 75 m/s (B) 55m/s
(A) 12 s (B) 8 s
(C) 66.7m/s (D) 33.3 m/s
(C) 15 s (D) 10 s
23. A car moves along a straight line whose equation
30. A river is flowing from east to west at a speed of
of motion is given by s = 12t + 3t2 – 2t3 where s is
5 m/min. A man on south bank of river, capable of
in metres and t is in seconds. The velocity of the
swimming 10 m/min in still water, wants to swim
car at start will be :–
across the rive in shortes time; he should swim :–
(A) 7 m/s (B) 9 m/s
(A) due north
(C) 12 m/s (D) 16 m/s (B) due north-east
24. The coordinates of a moving particle at time t are (C) due north-east with double the speed of river
given by x = ct and y = bt . The speed of the particle
2 2
(D) none of the above
is given by :–

82
Motion in a Plane
31. Two trains, each 50m long, are travelling in opposite (A) velocity (B) v2
directions with velocity 10 m/s and (C) v3 (D) v
15 m/s. The time of crossing is :–
38. Acceleration is defined as the rate of change of
(A) 2 s (B) 4 s (C) 2 3 s (D) 4 3 s velocity. Unit of rate of change of aceleration is
32. A motorcycle is moving with a velocity 80 km/hr (A) m/s2 (B) m/s3
ahead of a car moving with a velocity of 65 km/hr (C) m/s (D) m2/s3
in the same direction. What is the relative velocity 39. Figure below shows the velocity time graph of a
of the motorcycle with respect to the car :– one dimensional motion.Which of the following
(A) 15 km/hr (B) 20 km/hr characteristic of the particle is represented by the
(C) 25 km/hr (D) 145 km/hr. shaded area?
33. A particle is moving eastward with a velocity of v
5 m/s In 10 s, the velocity changes to 5 m/s
northward. The average acceleration in this time is
(A) zero
O t
1
(B) m/s towards north-west
2
(A) Speed (B) Distance covered
2
(C) Acceleration (D) Momentum
1
(C) m/s2 towards north-east 40. The time elapsed is plotted along the x-axis and the
2
acceleration is plotted along the y-axis.The area
1 between the graph and the x-axis gives :–
(D) m/s2 towards north-west
2
(A) average velocity
34. The velocity of a body depends on time according
(B) distance covered
to the equation v = 20 + 0.1 t 2 . The body is
undergoing : (C) difference in velocities

(A) uniform acceleration (D) difference in accelerations

(B) uniform retardation 41. The instantaneous velocity of a body can be


(C) non-uniform acceleration measured :–
(D) zero acceleration (A) Graphically (B) By speedometer
35. If for a particle position x µ t2 then :– (C) Both of above (D) Vectorially
(A) velocity is constant 42. The area under acceleation-time graph gives :–
(B) acceleration is constant (A) Distance travelled
(C) acceleration is variable (B) Change in acceleration
(D) None of these (C) Force acting
36. Relation between displacement x and time t is (D) Change in velocity
x = 2 – 5t + 6t , the initial acceleration will be :–
2
43. Velocity-time curve for a body projected vertically
(A) –3 ms –2
(B) 12 ms –2
upwards :–
(C) 2 ms –2
(D) –5 ms –2
(A) Parabola
37. In a straight line motion the distance travelled is (B) Ellipse
proportional to the square root of the time taken. (C) Hyperbola
The acceleration of the particle is proportional (D) Straight line
to :
83
Motion in a Plane
44. The displacement-time graph of a moving particle
is shown. The instantaneous velocity of the particle v v
is negative at the point :– (A) (B)
s s
displacement
v v
D (C) (D)
E s s
C F
48. Acceleration-time graph of a body is shown. The
time corresponding velocity-time graph of the same body
is :
(A) D (B) F a
(C) C (D) E
45. Which of the following velocity-time graphs
t
represent uniform motion :–
v v
v v
(A) (B)
(A) (B) t t
v v
t t

(C) (D)
v v
t t

(C) (D) 49. Which graph represents the uniform acceleration


t t
s s
46. Which of the following velocity–time graph shows (A) (B)
a realistic situation for a body in motion :- t
t

s s
v v (C) (D)
(A) (B)
t t
t t 50. Figure shows the distance time (x – t) graph of the
motion of a car. It follows from the graph that the
car is :–

v
(C) v (D) x
t t
t
47. A body starting from rest moves along a straight line (A) At rest (B) In uniform motion
with a constant acceleration. The variation of speed (C) Retarded (D) Accelerated
(v) with distances (s) is represented by the 51. The velocity-time graph of a particle moving along
graph :– straight line is shown in figure. Find the displacement
of the particle in 5 seconds :-

84
Motion in a Plane
v(m/s) of :-
(A) 20 m (B) 40 m
4
(C) 60 m (D) 80 m
4 5 57. If a car at rest accelerates uniformly to a speed of
0
2 3 t(sec) 144 km/h in 20 second, it coveres a distance of :–

–4 (A) 20 m (B) 400 m

(A) 16 m (B) 4 m (C) 1440 m (D) 2980 m


(C) 12 m (D) 10 m 58. A body starts from rest, what is the ratio of the
52. What is the magnitude of average velocity of a
distance travelled by the body during the 4th and
particle over 5 second for shown position-time graph
:- 3rd second ?
x(m) 7 5 7 3
(A) (B) (C) (D)
5 7 3 7
20
59. A body starts from rest is moving under a constant

10 acceleration up to 20 s. If it moves S1 distance in


first 10 s, and S2 distance in next 10 s then S2 will be
0 equal to :
1 2 4 5 t(sec)
(A) S1 (B) 2S1 (C) 3S1 (D) 4S1
(A) 10 m/s (B) 5 m/s
60. If a body starts from rest and travels 120m in the
(C) 4 m/s (D) 0
8th second, then acceleration is :–
53. Length of a minute hand of a clock is 4 cm. Find
the distance and displacement of tip of the minute (A) 16 m/s2 (B) 10 m/s2
hand between 3 P.M. to 4:15 P.M. ? (C) 0.227 m/s2 (D) 0.03 m/s2
(A) 8p cm, 4 2 cm 61. If a train travelling at 72 km/h is to be brought to
(B) 10p cm, 4 2 cm rest in a distance of 200 m, then its retardation should
(C) 10p cm, 2 cm be :–
(D) 8p cm, 2 cm (A) 20 m/s2 (B) 2 m/s2
54. A ball thrown vertically upwards with a speed of
20 m/s from the top of a tower returns to the earth in (C) 10 m/s2 (D) 1 m/s2
6 sec. Find the height of the tower ? 62. If a body starts from rest and travels 120cm in the
(g = 10 m/s2)
6th second then what is the acceleration ?
(A) 40 m (B) 60 m
(C) 80 m (D) 100 m (A) 0.20 m/s2 (B) 0.027 m/s2
55. A car moving with a speed of 40 Km/h can be
stopped by applying brakes in 2 sec. If the same (C) 0.218 m/s2 (D) 0.003 m/s2
car is moving with 80 Km/h. What is the minimum 63. A body starts from rest and has an acceleration
stopping time ?
20cm/s2. What is the distance covered by the body
(A) 1 sec (B) 2 sec
in first 8 second ?
(C) 3 sec (D) 4 sec
(A) 160cm (B) 640cm
56. If a car at rest accelerates uniformly and attains
speed of 54 Km/h in 8 sec, then it covers a distance (C) 1280 cm (D) 1640 cm

85
Motion in a Plane
64. Initially a body is at rest. If its acceleration is (A) 5 (B) 20

5ms–2 then the distance travelled in the 18th second (C) 40 (D) 10
is :– 72. Two particles of masses m1 and m2 are dropped
from heights h1 and h2. They reach the earth after
(A) 86.6 m (B) 87.5 m
times t1 and t2 respectively. Then :–
(C) 88 m (D) 89 m
t1 h1 t1 h2
65. When a train is stoped by applying break it stops (A) = (B) =
t2 h2 t2 h1
after travelling a distance of 50 metres. If speed of
train is doubled and same retarding force is applied t h
2 2
then it stops after travelling a distance of :– (C) t = h (D) none of these
1 1

(A) 50 m (B) 100 m


73. A body is falling from hight 'h' it takes t1 time to
(C) 200 m (D) 400 m reach the ground. The time taken to cover the first
66. A particle of mass 4kg is acted upon by steady force half of height is :–
of 4N. Distance travelled the particle in 4 s and is :– t1 t2
(A) 16 m (B) 2m (A) t2 = (B) t1 =
2 2
(C) 8 m (D) 4m (C) t2 = 3 t1 (D) None of these
67. A body is dropped from a tower with zero velocity, 74. A body is imparted motion from rest to move in a
reaches ground in 4s. The height of the tower is straight line. It is then obstructed by an opposite
about :– force, then :–

(A) 80 m (B) 20 m (A) the body may necessarily change direction


(B) the body is sure to slow down
(C) 160 m (D) 40 m
(C) the body will necessarily continue to move in
68. A ball is thrown upward with a velocity of
the same direction at the same speed
100 m/s. It will reach the ground after :–
(D) none of the above.
(A) 10 s (B) 20 s
75. The maximum range of a gun on horizontal
(C) 5 s (D) 40 s
surface is 16 km. If g = 10 ms– 2 , the muzzle
69. A ball of mass m1 and another ball of mass m2 are velocity of the shell must be :–
dropped from equal height. If times taken by the
(A) 1600 ms– 1 (B) 400 ms– 1
balls are t1 and t2 respectively, then :–
(C) 200 2 ms– 1 (D) 160 10 ms– 1
t2
(A) t1 = (B) t1 = t2
2 76. A body is thrown with a velocity of 9.8 ms – 1 making

t2 an angle of 300 with the horizontal. It will hit the


(C) t1 = 4t2 (D) t1 = ground after how much time :–
4

70. If a ball is thrown vertically upwards with 40 m/s. (A) 3 s (B) 2 s


its velocity after two second will be :– (C) 1.5 s (D) 1 s

(A) 10 m/s (B) 20 m/s 77. If a body A of mass M is thrown with velocity u at
an angle of 300 to the horizontal and another body
(C) 30 m/s (D) 40 m/s
B of the same mass is thrown with the same speed
71. A body projected vertically upwards with a velocity
at an angle of 600 to the horizontal, the ratio of
u returns to the starting point in 4 second. If
horizontal range of A to B will be :–
g = 10 ms–2, the value of u is (ms–1) :–

86
Motion in a Plane
84. Two projectiles of same mass and with same
(A) 1 : 3 (B) 1 : 1
velocity are thrown at an angle 60° and 30° with
(C) 1 : 3 (D) 3 :1 the horizontal, then which quantity will remain same
78. During projectile motion, the quantities that remain (A) time of flight
unchanged are :– (B) range of projectile
(A) force and vertical velocity (C) max height acquired
(B) acceleration and horizontal velocity (D) all of them

(C) kinetic energy and acceleration 85. A projectile of mass m is fired with velocity v from a
point as shown in figure, neglecting air resistance,
(D) acceleration and momentum
what is
79. Three particles A, B and C are projectied from the
the change in momentum when leaving P and
same point with the same initial speeds making
arriving at Q ?
angles 30°, 45° and 60° respectively with the
horizontal. Which of the following statements is
v
correct
(A) A, B and C have unequal ranges
30°
(B) Ranges of A and C are equal and less than that
P Q
of B
(A) 0 (B) mv
(C) Ranges of A and C are equal and greater than
that of B 2mv
(C) 4 mv (D)
3
(D) A, B and C have equal ranges
80. At the top of the trajectory of a projectile, the 86. At the uppermost point of a projectile its velocity
and acceleration are at an angle of :–
acceleration is :–
(A) 180° (B) 90°
(A) maximum (B) minimum
(C) 60° (D) 45°
(C) zero (D) g
87. In the graph shown in fig. the time is plotted along
81. A ball whose kinetic energy is E, is thrown at an
x-axis. Which quantity associated with the projectile
angle of 45° with the horizontal, its kinetic energy
motion is plotted along the y - axis
at the highest point of its flight will be :–
y
E
(A) E (B)
2
E
(C) (D) zero
2
82. A bullet is fired from a gun with velocity 500 ms– 1, x
time (t)
then the maximum range is :–
(A) kinetic energy (B) momentum
(A) 25 × 103 m (B) 12.5 × 103 m
(C) horizontal velocity (D) none of the above
(C) 50 × 102m (D) 25 × 102 m
88. In case of a projectile fired at an angle equally
83. Range of a projectile is R, when the angle of
inclined to the horizontal and vertical with velocity
projection is 300. Then, the value of the other angle
(u). The horizontal range is :–
of projection for the same range, is :–
(A) 45° (B) 60° u2 u2
(A) (B)
g 2g
(C) 50° (D) 40°

87
Motion in a Plane
2u2 u2 R R
(C) (D) 2 (A) R (B) 2R (C) (D)
g g 2 4

89. The speed at the maximum height of a projectile is 96. A ball is projected upwards. Its acceleration at the
highest point is :–
3
times of its inital speed 'u' of projection. Its (A) zero
2
(B) directed upwards
range on the horizontal plane :–
(C) directed downwards
3u 2 u2 3u2 3u2 (D) such as cannot be predicted
(A) (B) (C) (D)
2g 2g 2g g 97. A bullet is fired from a cannon with velocity 500
90. A ball is thrown at an angle q with the horizontal and ms–1. If the angle of projection is 15° and g = 10
the range is maximum. The value of tanq is :– ms–2, then the range is :–
(A) 25 × 103 m (B) 12.5 × 103 m
1
(A) 1 (B) 3 (C) (D) 2 (C) 50 × 102 m (D) 25 × 102 m
3
98. If a projectile is fired at an angle q with the horizontal
91. A student is able to throw a ball vertically to
with velocity u, then the time of flight is given by :–
maximum height of 40 m. The maximum distance
to which the student can throw the ball in the u cosq u sin q
(A) (B)
g g
horizontal direction :–
(A) 40 (B)1/2 m (B) 20(B)1/2m 2u sin q 2u cos q
(C) (D)
(C) 20 m (D) 80 m g g

99. The horizontal range of a projectile is maximum


gx2
92. The equation of a projectile is y = 3x– the when angle of projection is :–
2
p p
angle of projection is :– (A) (B)
3 6
(A) 30° (B) 60°
p p
(C) 45° (D) none (C) (D)
2 4
93. If air resistance is not considered in projectiles, the
100. A particle is fired with velocity u making angle q
horizontal motion takes place with :–
with the horizontal. What is the magnitude of change
(A) Constant velocity
in velocity when it is at the highest point ?
(B) Constant acceleration
(A) u cos q (B) u
(C) Constant retardation
(D) Variable velocity (C) u sin q (D) (u cosq – u)

94. If a projectile is fired at an angle q with the vertical 101. In the above, the change in speed is :–
with velocity u, then maximum height attained is (A) u cos q (B) u
given by :– (C) u sin q (D) (u cosq – u)
u 2 cos q u 2 sin 2 q 102. A projectile is fired at 30°, with momentum p.
(A) (B)
2g 2g Neglecting friction, the change in kinetic energy

u 2 sin 2 q u 2 cos 2 q
when it returns to the ground will be-
(C) (D)
g 2g (A) Zero (B) 30%
95. If R is the maximum horizontal range of a particle, (C) 60% (D) 100%
then the greatest height attained by it is :–

88
Motion in a Plane
103. A ball is thrown with initial energy 100 J at an angle 110. Two stone are projected with the same speed but
q to the horizontal. If its energy at the top becomes making diffrent angles with the horizontal. Their
30 J then angles of projection- ranges are equal. If the angle of projection of one
(A) q = 45° (B) q = 30° p
is and its maximum height is y1 then the
3
1/ 2 maximum height of the other will be :–
–1 æ 3 ö –1 æ 3 ö
(C) q = cos ç 10 ÷ (D) q = cos ç ÷
è ø è 10 ø y1 y1
(A) 3y1 (B) 2y1 (C) (D)
2 3
104. A body is projected at an angle q with horizontal.
Another body is projected with the same velocity 111. A projectile is thrown from a point in a vertical plane
at an angle q with the vertical. The ratio of the such that its horizontal and vertical velocity
component are 9.8 m/s and 19.6 m/s respectively.
time of flights is-
2 its horizontal range is :–
(A) 1 : 1 (B) tan q : 1
(A) 4.9 m (B) 9.8 m
(C) 1 : cot q (D) tan² 1 : q
(C) 19.6 m (D) 39.2 m
105. A body is projected at such an angle that horizontal
112. A projectile is projected with initial velocity
range is three times the greatest height. The angle
of projection is- (6iˆ + 8j)
ˆ m/s. If g = 10 ms–2, then horizontal range

(A) 30° (B) 37° is :–

(C) 45° (D) 53° (A) 4.8 metre (B) 9.6 metre
(C) 19.2 metre (D) 14.0 metre
106. The range of a projectile when fired at 75° with the
113. Three projectile A, B and C are thrown from the
horizontal is 0.5 km. what will be its range when
same point in the same plane. Their trajectories are
fired at 45° with same speed :–
shown in the figure. Then which of the following
(A) 0.5 km. (B) 1.0 km.
statement is true :–
(C) 1.5 km. (D) 2.0 km.
x2
107. The equation of projectile is y = 16x – the
4
horizontal range is :–
(A) 16 m (B) 8 m
O ABC
(C) 64 m (D) 12.8 m
(A) The time of flight is the same for all the three
108. The maximum range of a projectile fired with some
(B) The launch speed is greatest for particle C
initial velocity is found to be 1000 metre The
(C) The horizontal velocity component is greatest for
maximum height (H) reached by this projectile
particle C
is :–
(D) All of the above
(A) 250 metre (B) 500 metre
114. An aeroplane is flying in a horizontal direction at
(C) 1000 metre (D) 2000 metre
600 km/hr at a height of 6 kms and is advancing
109. A body is thrown with some velocity from the
towards a point which is exactly over a target on
ground. Maximum height when it is thrown at 60°
earth. At that instant the pilot releases a ball which
to horizontal is 90 m. What is the height reached
on descending the earth strike the target. The falling
when it is thrown at 30° to the horizontal :–
ball appears-
(A) 90 m (B) 45 m
(A) to the pilot in the aeroplane, as falling along a
(C) 30 m (D) 15 m
parabolic path

89
Motion in a Plane
(B) to a person standing near the target, as falling seconds, then the angular velocity of the particle
exactly vertical after 2 s from its start is :–
(C) to a person standing near the target, as describing (A) 8 rad/s (B) 12 rad/s
a parabolic path (C) 24 rad/s (D) 36 rad/s
(D) to the pilot sitting in the aeroplane, as falling in 122. The angular acceleration of particle moving along
a zigzag path a circular path with uniform speed :–
115. A marble is dropped vertically, Another identical
marble B is projected horizontally from the same (A) uniform but non zero
point at the same instant :– (B) zero
(A) A will reach the ground earlier than B
(C) variable
(B) B will reach the ground earlier than A
(C) Both A and B will reach the ground at the same (D) such as can not be predicted from given
instant information
(D) None of the above
123. The blades of an aeroplane propeller are rotating at
116. An aeroplane moving horizontally with a speed of
180 km/hr. drops a food packet while flying at a the rate of 600 revolutions per minute its angular
height of 490m. The horizintal range of the packet velocity is :–
is :–
(A) 10 p rad/s (B) 20 p rad/s
(A) 180 m (B) 980 m
(C) 500 m (D) 670 m (C) 2p rad/s (D) None of them
117. A bomber is flying horizontally with a constant speed 124. a partical moves in a circle of the radius 25 cm at
of 150 m/s at a height of 78.4 m. The pilot has to
drop a bomb at the enemy target. At what horizontal two revolutions per second. The acceleration of the
distance from the target should he release the bomb? particle in m/sec2 is :–
(A) 0 m (B) 300 m (C) 600 m (D) 1000m (A) p2 (B) 8p2
118. The angular velocity of a wheel is 70 rad/s. If the
(C) 4p2 (D) 2p2
radius of the wheel is 0.5 m, then linear velocity of
125. A particle moves in a circle describing equal angle
the wheel is :–
in equal times, its velocity vector :–
(A) 70 m/s (B) 35 m/s
(A) remains constant
(C) 30 m/s (D) 20 m/s
(B) changes in magnitude
119. Two particles having mass 'M' and 'm' are moving
(C) change in direction
in a circular path having radius R and r. If their time
(D) changes in magnitude and direction
period are same then the ratio of angular velocity
126. If a r and a t represent radial and tangential
will be :–
acceleration, the motion of a particle will be uniform
r R R
(A) (B) (C) 1 (D) circular motion if :–
R r r
(A) ar = 0 and at = 0 (B) ar = 0 but at ¹ 0
120. Angular velocity of minute hand of a clock is :–
(C) ar ¹ 0 but at = 0 (D) ar ¹ 0 and at ¹ 0
p 127. A 500 kg car takes a round turn of radius 50 m with
(A) rad/s (B) 8p rad/s
30
a velocity of 36 km/hr. The centripetal
force is
2p p
(C) rad/s (D) rad/s (A) 250 N (B) 1000N
1800 1800
(C) 750N (D) 1200 N
121. If the equation for the displacement of a
particle moving on a circular path is given by 128. A stone is attached to the end of a string and whirled
(q) = 2t3 + 0.5, where q is in radians and t in in horizontal circle, with uniform speed then :–

90
Motion in a Plane
133. A wheel is of diameter 1m. If it makes 30
(A) its linear and angular momentum are constant
revolutions/sec., then the linear speed of a point on
(B) only linear momentum is constant
its circumference will be :-
(C) its angular momentum is constant but linear
(A) 30p m/s (B) p m/s
momentum is variable
(C) 60p m/s (D) p/2 m/s
(D) both are variable 134. Angular speed of a uniformly circulating body with
129. A car moving with speed 30 m/s on a circular path time period T is :
of radius 500 m. Its speed is increasing at the rate
of 2m/s2. The acceleration of the car is :– 2p
(A) 2pT (B)
T
(A) 9.8 m/s2 (B) 1.8 m/s2
(C) 2 m/s2 (D) 2.7 m/s2
p
130. A stone tied to the end of a string 80 cm long is (C) pT (D)
T
whirled in a horizontal circle with a constant speed.
If the stone makes 14 revolutions in 22 s than the 135. Speed of an object moving in circular path of radius

acceleration of the stone is :– 10 m with angular speed 2 rad/s is :

(A) 5 m/s2 (B) 10 m/s2 (A) 10 m/s (B) 5 m/s


(C) 12.8 m/s2 (D) None of these (C) 20 m/s (D) 30 m/s
131. A 0.5 kg ball moves in a circle of radius 0.4m at a
136. The distance of a particle moving on a circle of radius
speed of 4 ms –1 . The centripetal force on the
12 m measured from a fixed point on the circle and
ball is
measured along the circle is given by
(A) 10N (B) 20N
s = 2t3 (in meters). The radio of its tangential to
(C) 40N (D) 80N
centripetal acceleration at t = 2s is :
132. A car is travelling at 20 m/s on a circular road of
(A) 1 : 1 (B) 1 : 2
radius 100 m. It is increasing in speed at the rate of
(C) 2 : 1 (D) 3 : 1
3 m/s2. Its acceleration is :–
(A) 3 m/s2 (B) 4 m/s2
(C) 5 m/s2 (D) 7 ms–1

91
Motion in a Plane

MHT-CET QUESTIONNAIRE ANSWER KEY EXERCISE - II


Que. 1 2 3 4 5 6 7 8 9 10
Ans. D C C A B B A D B A
Que. 11 12 13 14 15 16 17 18 19 20
Ans. A A A B C C C C A B
Que. 21 22 23 24 25 26 27 28 29 30
Ans. B C C D B B C D D A
Que. 31 32 33 34 35 36 37 38 39 40
Ans. B A B C B B C B B C
Que. 41 42 43 44 45 46 47 48 49 50
Ans. C D D D A B C C A D
Que. 51 52 53 54 55 56 57 58 59 60
Ans. B D B B D C B A C A
Que. 61 62 63 64 65 66 67 68 69 70
Ans. D C B B C C A B B B
Que. 71 72 73 74 75 76 77 78 79 80
Ans. B A A B B D B B B D
Que. 81 82 83 84 85 86 87 88 89 90
Ans. C A B B B B C A A A
Que. 91 92 93 94 95 96 97 98 99 100
Ans. D B A D D C B C D C
Que. 101 102 103 104 105 106 107 108 109 110
Ans. D A D C D B C A C D
Que. 111 112 113 114 115 116 117 118 119 120
Ans. D B D C C C C B C D
Que. 121 122 123 124 125 126 127 128 129 130
Ans. C B B C C C B C D C
Que. 131 132 133 134 135 136
Ans. B C A B C B

92
Motion in a Plane
JEE MAINS QUESTIONNAIRE EXERCISE - III
1. A man walks 30 m. towards north, then 20 m.
6. The relation 3t = 3x + 6 describes the
towards east and in the last 30 2 m. towards south
displacement of a particle in one direction where x
- west. The displacement from origin is :–
is in meters and t in seconds. The displacement,
(A) 10 m. towards west
when velocity is zero, is.
(B) 10 m. towards east
(A) 24 m (B) 12 m
(C) 60 2 m.towards north west
(C) 5 m (D) zero
(D) 60 2 m towards east north 7. Starting from rest ,the acceleration of a particle is
2. A body moves along a curved path of a a = 2(t – 1).The velocity of the particle at t= 5s is :–
quarter circle. Calculate the ratio of distance to (A) 15 m/s (B) 25 m/s
displacement:– (C) 5 m/s (D) None of these
(A) 11 : 7 (B) 7 : 11 8. A scooter going due east at 10 m s–1 turns right
(C) 11 : 2 ´ 7 (D) 7 : 11 2 through an angle of 90°. If the speed of the scooter
3. Three particles P, Q and R situated at point A on the remains unchanged in taking this turn, the change
circular path of radius 10 m. All three particles moves in the velocity of the scooter is :
along different path and reaches at point B shown in (A) 20.0 m s–1 in south-western direction
figure then the ratio of distance traversed by particle P (B) zero
& Q is
(C) 10.0 m s–1 in south-east direction
P (D) 14.14 m s–1 in south-western direction
Q O
A 9. A boy walks to his school at a distance of 6 km
R with a speed of 2.5 km/h, and walks back with a
B constant speed by 4 km/h. His average speed for
trip expressed in km/h is :–
3 1
(A) (B)
4 3
24 40
(A) (B)
13 13
3p p
(C) (D)
4 3 (C) 3 (D) 4.8
4. The displacement of a particle starting from rest (at 10. One car moving on a straight road covers one thirds
t=0) is given by s = 6t2 – t3 The time which the of the distance with 20 km/hr and the rest with
particle will attain zero velocity again, is : 60 km/hr. The average speed is :–
(A) 4s (B) 8s (A) 40 km/hr (B) 80 km/hr
(C) 12s (D) 16s
2
5. A particle moves along the x-axis such that is (C) 46 km/hr (D) 36 km/hr
3
co-ordinate (x) varies with time (t), according to the
expression : x = 2 – 5t + 6t2, where x is in metrres and 11. A body starts from rest and with a uniform
t is in seconds. The initial velocity of the particle is acceleration of 10 ms– 2 for 5 seconds. During the

(A) –5 m/s (B) –3 m/s next 10 seconds it moves with uniform velocity, the
total distance travelled by the body is :–
(C) 6 m/s (D) 2 m/s

93
Motion in a Plane
(A) 100 m (B) 125 m acceleration are respectively
(C) 500 m (D) 625 m (A) b, – 4d (B) –b, 2c
12. A car runs at constant speed on a circular track of (C) b, 2c (D) 2c, – 4d
radius 10 m. taking 6.28 s. on each lap. The average 18. The displacement 'x' of a particle moving along a straight
speed and average velocity on each complete lap is line at time t is given by x = a0 + a1t + a2t2. the
(A) Velocity 10 ms–1, speed 10 ms–1 acceleration of the particle is :
(B) Velocity zero, speed 10 ms–1 (A) a1 (B) a2
(C) Velocity zero, speed zero (C) 2a2 (D) 3a2
(D) Velocity 10 ms–1 speed zero 19. A boat is sailing at a velocity 3iˆ + 4ˆj with respect
13. In 1.0 second a particle goes from point A to point to ground and water in river is flowing with a velocity
B, moving in a semicircle of radius 1.0 m, the magnitude
-3iˆ - 4ˆj . Relative velocity of the boat with respect
of the average velocity is :–
to water is :–
A (A) 8jˆ (B) 5 2 ˆj

O (C) 6iˆ + 8ˆj (D) -6iˆ - 8ˆj


1m
B 20. Two trains each of length 50 m. are running with
constant speeds on parallel tracks. While moving in
(A) 3.14 ms –1
(B) 2.0 ms –1
same direction one over takes the other in 40 seconds
(C) 1.0 ms–1 (D) Zero and while moving in opposite direction one crosses
14. A particle moves along the side AB, BC, CD of a the other in 20 seconds. The speeds of trains will
be :–
square of side 25 m. with a velocity of 15 m/s. Its
(A) 2.25 ms–1 and 2.75 ms–1
average velocity is :–
(B) 2.15 ms–1 and 2.85 ms–1
(A) 15 ms–1 (B) 10 ms–1
(C) 3.75 ms–1 and 1.25 ms–1
(C) 7.5 ms–1 (D) 5 ms–1
(D) 4.25 ms–1 and 0.75 ms–1
15. The velocity-time relation of an electron starting
21. The initial velocity of a particle is u (at t = 0) and
from rest is given by u = kt, where k=2m/s2.
the acceleration is given by f = at. Which of the
The distance traversed in 3 sec is :– following relations is valid ?
(A) 9m (B) 16 m
at 2
(C) 27 m (D) 36 m (A) v = u + at2 (B) v = u +
2
16. The x and y co-ordinates of a particle at any time t
(C) v = u + at (D) v = u
are given by x = 7t + 4t2 and y = 5t where x and
22. A car moving with a velocity of 10 m/s can be
y are in m and t in s. The acceleration of the particle
stopped by the application of a constant force F in a
at 5s is :– distance of 20m. If the velocity of the car is
(A) zero (B) 8 m/s 2
30 m/s. It can be stopped by this force in :–
(C) 20 m/s2 (D) 40 m/s2
20
(A) m (B) 20 m
17. The displacement of a particle is given by 3
y = a + bt + ct2 – dt4 The initial velocity and (C) 60 m (D) 180 m

94
Motion in a Plane
23. A particle travels 10m in first 5 second 10m in next H
(B) At metres from ground
3 second. Assuming constant acceleration what is 4
the distance travelled in next 2 second : 3H
(A) 8.3 m (B) 9.3 m (C) At metres from the ground
4
(C) 10.3 m (D) None of these H
® ® ® (D) At metres from the ground
24. The initial velocity of a particle, u = 4 i + 3 j . 6
It is moving with uniform acceleration, 30. A particle is thrown vertically upward. Its velocity
® ® ®
a = 0.4 i + 0.3 j . Its velocity after 10 second is – at half of the height is 10m/s. The maximum height
(A) 3 unit (B) 4 unit attained by is (g=10 ms–2) :
(C) 5 unit (D) 10 unit (A) 8m (B) 20m
25. A body falls freely from rest. It covers as much
(C) 10m (D) 16m
distance in the last second of its motion as covered
in the first three second. The body has fallen for a 31. A stone falls from a balloon that is descending at a
time of :– uniform rate of 12 ms–1. The displacement of the
(A) 3s (B) 5s (C) 7s (D) 9s stone from the point of release after 10 sec is :–
26. The water drops fall at regular intervals from a tap (A) 490 m (B) 510 m
5 m above the ground. The third drop is leaving the
(C) 610 m (D) 725 m
tap at the instant the first drop touches the ground.
How far above the ground is the second drop at 32. A body starts falling from height 'h' and travels
that instant ? distance h/2 during last second of motion then time
(A) 1.25 m (B) 2.50 m
of travel is :– (In second)
(C) 3.75 m (D) 4.00 m
27. A man is slipping on a frictionless inclined plane & (A) 2 –1 (B) 2 + 2
a bag falls down from the same height. Then the
speed of both is related as:- (C) 2+ 3 (D) 3 +2
(A) VB > Vm
33. A stone falls freely such that the distance covered
(B) VB < Vm
by it in the last second of its motion is equal to the
(C) VB = Vm
(D) VB and Vm can't related distance covered by it in the first 5 seconds. It
28. A body released from a height falls freely towards remained in air for :–
earth. Another body is released from the same (A) 12 s (B) 13 s
height exactly one second later. The separation
(C) 25 s (D) 26 s
between the two bodies two second after the release
of the second body is :– 34. When a ball is thrown up vertically with velocity v0, it
(A) 9.8 m (B) 49 m reaches a maximum height of 'h'. If one wishes to
(C) 24.5 m (D) 19.6 m triple the maximum height then the ball should be thrown
29. A body is released from the top of the tower of
with velocity –
height H. It takes t time to reach the ground. Where
t (A) 3 v0 (B) 3v0
is the body time after release :–
2
H (C) 9v0 (D) 3/2v0
(A) At metres from ground
2
95
Motion in a Plane
35. A stone thrown upward with a speed 'u' from the
top of the tower reaches the ground with a velocity x x
'3u'. The height of the tower is :– (A) (B)
t t
3u2 4u2
(A) (B)
g g

6u2 9u2 x x
(C) (D)
g g (C) (D)
36. The graph between the displacement x and time t t t
for a particle moving in a straigh line as shown in
figure. During the interval OA, AB, BC and CD, 39. A person is moving in a circle of radius r with
the acceleration of the particle is : constant speed v. The change in velocity in moving
from A to B is
displacement

D B
C O 40° A
A B x
time

OA AB BC CD
(A) + 0 + + (A) 2 v cos 40° (B) 2 v sin 40°
(B) – 0 + 0 (C) 2 v cos 20° (D) 2 v sin 20°
(C) + 0 – + 40. A bus accelerates from the rest for time 't1' at a
(D) – 0 – 0 constant rate a and then retards at a constant rate
37. Which of the llowing situation is represented by the b for time 't2' and comes to rest, then 't1/t2' :–
velocity-time graph the show in the diagram :– a b+a
v (A) (B)
b+a 2

t
b+a b
(C) (D)
a a

(A) A stone thrown up vertically, returning back to 41. The velocity verses time graph of a body moving
the ground along a straight line is as shown in figure. The ratio
of displacement and distance curved by body in 5
(B) A car decelerating at constant rate and then
second is :–
accelerating at the same rate
(C) A ball falling from a height and then bouncing 2
1
back
0
1 2 3 4 5 t
(D) Back of the above –1

38. A car starts from rest accelerates uniform for –2

4 second and then moves with uniform velocity


(A) 2 : 3 (B) 3 : 5
which of the x-t graph represent the motion of the
(C) 1 : 1 (D) 1.5 : 5
car

96
Motion in a Plane
42. A body is projected vertically upward from the surface 46. A person walks along an east-west street and a graph
of the earth, then the velocity time graph is :– of his displacement from home is shown in figure.
His average velocity for the whole time interval is

v
v
(A) (B) 40
t

x (metre)
t 20
19
0
3 6 9 12 15 18 21
–20 t (sec)
v v
(C) (D) –40

t t (A) 0 ms–1 (B) 23 ms–1

43. A student is standing at a distance of 50 metres (C) 8.4 ms–1 (D) None of above
behind a bus. As soon as the bus begins its motion 47. I n the f ollowing velocity time graph of a body
the distance travelled by the body and its
with an acceleration of 1 ms–2, the student starts
displacement during 5 second in meter will be :-
running towards the bus with a uniform velocity u.
Assuming the motion to be along straight road, the
minimum value of u, so that the student is able to
catch the bus is :-
(A) 5 m s–1 (B) 8 m s–1
(C) 10 m s–1 (D) 12 m s–1
44. Figure shows the displacement of a particle going
along x-axis as a function of time. The acceleration
of the particle is zero in the region :– (A) 75, 75 (B) 110, 70
(C) 110, 110 (D) 110, 40
dis placement

48. A body is projected upwards with a velocity u. It


B C
E passes through a certain point above the ground
after t1 second. The time after which the body
A D
passes through the same point during the return
time
journey is
(a) AB (b) BC (c) CD (d) DE
u é u ù
(A) a, b (B) a, c (C) b, d (D) c, d (A) - t12 (B) ê2 g - t1 ú
g ë û
45. The figure shows the displacement time graph of a
particle moving on a straight line path. What is the u2 u2
(C) 3 - t1 (D) 3 2
- t1
g g
average velocity of the particle over 10 s
49. For motion of a particle acceleration time graph is
shown in figure then the velocity time curve for the
60 duration of 0-4 sec is :–
40
3
x (in metre)

30 2
a(in m/s2)

1
20 0
–1 1 23 4 t(s)
10 –2
–3
0
2 4 6 8 10 t sec)

(A) 2 ms–1 (B) 4 ms–1 (C) 6 ms–1 (D) 8 ms–1


97
Motion in a Plane
53. A body dropped from the top of the tower covers a
6
5 distance 7x in the last second of its journey, where
6 4
velocity
5 3 x is the distance covered in first second. How much
4 2
3
time does it take to reach the ground ?
1

velocity
(A) 2 (B) 0 t (A) 3s (B) 4s
1 –1 12 3 4
0 t –2 (C) 5s (D) 6s
1 2 3 4
–3
54. Two balls are dropped from different heights at
different instants. Second ball is dropped 2 sec. after
3 3
2 2 the first ball. If both balls reach the ground
a(in m/s2)

1 1 simultaneously after 5 sec. of dropping the first ball,


(C) 0 (D) 0
1 23 4 t(s) t(s) the the difference of initial heights of the two balls
–1 –1
–2 –2
–3 –3 will be:– (g=9.8m/s2)
(A) 58.8 m (B) 78.4 m
50. A particle start from rest and move with constant
(C) 98.0 m (D) 117.6 m
acceleration. Than velocity displacement curve
55. A particle falling with constant acceleration covers
is :-
3m distance in third second of its motion. The
distance covered in first 5 second shall be :–
v v
(A) 5 m (B) 15 m
(A) (B)
(C) 30 m (D) 45 m
s s
56. Two cars are moving in the same directions with
the same speed of 30 km/hr. They are separated
v v
by 5 km. What is the speed of car moving in the
(C) (D)
opposite direction if it meets the two cars at an
s s interval of 4 minute :–
51. For a body of 50 kg. mass, the velocity time graph (A) 45 km/hr. (B) 60 km/hr
is shown in figure. Then force acting on the body (C) 105 km/hr (D) None
is :– 57. A body is projected vertically up at t = 0 with a
velocity of 98 m/s. Another body is projected from
2
velocity (m/s)

the same point with same velocity after time


1 4 seconds. Both bodies will meet after :–
(A) 6 s (B) 8 s
1 2 t(s) (C) 10 s (D) 12 s
(A) 25N (B) 50 N 58. A bird is flying with a speed of 40 km/hr. in the
(C) 12.5 N (D) 100 N north direction. A train is moving with a speed of 40
52. A body is projected with a velocity of 98 ms–1. If km/hr. in the west direction. A passenger sitting in
acceleration due to gravity be 9.8 m/s2, then :– the train will see the bird moving with velocity
(A) 40 km/hr in N-E direction
(A) It will go upto the maximum height of 980 m.
after 10 second. (B) 40 2 km/hr in N-E direction
(B) It will go upto the height of 490 m. is 5 seconds. (C) 40 km/hr in N-W direction
(C) It will come back to earth in 10 seconds.
(D) 40 2 km/hr in N-W direction
(D) Its speed reduced to 49 ms–1 after 5 seconds.

98
Motion in a Plane
59. A man is walking on a road with a velocity 64. A particle moving in a straight line covers half the
3 km/hr suddnly rain starts falling. The velocity of distance with speed of 3 m/s. The other half of the
rain is 10 km/hr in vertically downward direction. the distance is covered in two equal time intervals with
relative velocity of the rain with respect to man is speed of 4.5 m/s and 7.5 m/s respectively. The
(A) 13 km/hr (B) 7 km/hr average speed of the particle during this motion is
(A) 4.0 m/s (B) 5.0 m/s
(C) 109 km/hr (D) 13 km/hr
(C) 5.5 m/s (D) 4.8 m/s
60. A man is walking at a speed 3 m/s rain drops are falling
65. The following figures show velocity v versus time t
vertically with a speed 3 m/s :–
curve. But only some of these can be realised in
(i) What is the velocity of rain drop with respect to practice. These are-
the man ?
(ii) At what angle from vertical, the man should hold v v

his umbrella ?
(A) 2.42 m/s, 30° in forward direction [a] [b]
(B) 4.24 m/s, 45° in forward direction O t O t

(C) 1.24 m/s, 60° in forward direction


(D) None of these v v
61. A boat takes 2 hours to go 8 km and come back in
still water lake. With water velocity of 4 km/hr,the [c] [d]
time taken for going upstream of 8 km and coming
O t O t
back is :–
(A) 140 min (B) 150 min (A) Only a, b and d (B) Only a, b, c
(C) 160 min (D) 170 min (C) Only b and c (D) All of them
62. A man wishes to swim across a river 0.5 km. wide. If 66. A stuntman plans to run across a roof top and then
he can swim at the rate of 2 km/h. in still water and horizontally off it to land on the roof of next
the river flows at the rate of 1 km/h. The angle building. The roof of the next building is 4.9 metre
(w.r.t. the flow of the river) along which he should below the first one and 6.2 metre away from it.
swim so at to reach a point exactly opposite his starting What should be his minimum roof top speed in
point, should be :– m/s, so that he can successfully make the jump ?
(A) 60° (B) 120° (A) 3.1 (B) 4.0
(C) 145° (D) 90° (C) 4.9 (D) 6.2
63. A boat man can row with a speed of 10 km/hr in 67. The angle which the velocity vector of a projectile,
still water. The river flow steadilty at 5 km/hr and thrown with a velocity v at an angle q to the
the width of the river is 2 km. If the boat man cross horizontal, will make with the horizontal after time t
the river with reference to minimum distance of of its being thrown up is
approach then time elapsed in rowing the boat will
-1 æ q ö
be :– (A) q (B) tan ç ÷
ètø
2 3 2 -1 æ v cos q ö -1 æ v sin q - gt ö
(A) hour (B)
5 3
hour (C) tan ç ÷ (D) tan ç ÷
5 è v sin q - gt ø è v cos q ø
68. A particle is projected at an angle of 45° from 8m
3 2 5 2 before the foot of a wall, just touches the top of the
(C) hour (D) hour
5 3

99
Motion in a Plane
wall and falls on the ground on the opposite side at then v is:–
a distance 4 m from it.The height of wall (A) u cos q (B) u tan q
is : (C) u cot q (D) u sec q
u
2 4 8 3 75. Two balls A and B are thrown with speed u and
(A) m (B) m (C) m (D) m 2
3 3 3 4
respectively. Both the balls cover the same
69. A projectile is thrown with an initial velocity of horizontal distance before returning to the plane of
r
v = a î + b ĵ , if the range of projectile is double of projection. If the angle of projection of ball B is 15°
maximum height reached by it then : with the horizontal, then the angle of projection of A
(A) a = 2 b (B) b = a (C) b = 2a (D) b = 4a is :–

70. A projectile is thrown into space so as to have the 1 1 1


-1
maximum possible horizontal range equal to 400m. (A) sin (B) sin -1
8 2 8
Taking the point of projection as the origin, the
coordinates of the point where the velocity of the 1 1 1 1
(C) sin -1 (D) sin -1
projectile is minimum are : 3 8 4 8
(A) (400, 100) (B) (200, 100)
76. A projectile can have the same range R for two
(C) (400, 200) (D) (200, 200)
angles of projection. If t1 and t2 be the times of
71. A ball is projected upwards from the top of a tower
flight in the two cases :–
with a velocity of 50 ms–1 making an angle of 300
with the horizontal. The height of the tower is 70m. (A) t1t2 µ R2 (B) t1 t2 µ R
After how much time from the instant of throwing
will the ball reach the ground ? 1 1
(C) t1 t 2 µ (D) t1 t 2 µ
(A) 2 s (B) 5 s R R2

(C) 7 s (D) 9 s 77. Two particles separated at a horizonal distance x


72. At what angle with the horizontal should a ball be as shown in fig. they projected at the same line as
thrown so that its range R is related to the time of shown in fig. with different initial speeds. The time
flight as R = 5T2. Take g = 10 ms–2 : after which the horizontal distance between them
become zero :–
(A) 30° (B) 45°
(C) 60° (D) 90° u u
3
73. A particle is projected with a velocity v, so that its
range on a horizontal plane is twice the greatest 30° 60°
height attained. If g is acceleration due to gravity, x
then its range is :
x u
2 (A) (B)
4v 4g u 2x
(A) (B) 2
5g 5v
2u
(C) (D) none of these
4v 3 4v x
(C) (D) 5g2
5g 2 78. An object is projected with a velocity of 20 m/s
making an angle of 45° with horizontal. The
74. A particle is projected with a velocity u making an 2
equation for the trajectory is h = Ax – Bx , where
angle q with the horizontal. At any instant, its
‘h’ is height, ‘x’ is horizontal distance, A and B are
velocity v is at right angles to its initial velocity u;
100
Motion in a Plane
2
constants. The ratio A : B is- (g = 10 m/s ) u sin a
(A) 1 : 5 (B) 5 : 1 (A) (B) u cos a
2
(C) 1 : 40 (D) 40 : 1
u u sin a
79. A particle is thrown with velocity u making an angle (C) (D)
2 4
q with the vertical. It just crosses the top of two 84. A stone is projected from the ground with a velocity
poles each of height h after 1 s and 3 s respectively.
of 25 m/s. 2 sec. later, it just clears wall 5 m. high.
The maximum height of projectile is.
Then angle of projection of the stone and the
greatest height reached are- [Neglect air resistance,
u P Q 2
Assume g = 10 m/sec ]
q (A) 30°, 7.8 m (B) 60°, 8.7 m
h h
(C) 30°, 8.7 m (D) 60°, 8.7 cm
85. A particle moves along positive branch of a curve

(A) 9.8 m (B) 19.6 m x2 t2


y= , where x = . x and y are measured in
2 2
(C) 39.2 m (D) 4.9 m
80. The equation of motion of a projectile are given by metres and t in seconds. Velocity of particle at
2
x = 36 t metre and 2y = 96 t – 9.8 t metre. t = 2 s is -
The angle of projection is.
(A) (2iˆ - 4ˆj)m / sec (B) (2iˆ + 4ˆj) m / sec
-1 æ4ö -1 æ3ö
(A) sin ç 5 ÷ (B) sin ç 5 ÷ (C) (2iˆ + 2j)
ˆ m / sec (D) (4iˆ - 2j)
ˆ m / sec
è ø è ø

86. A stuntman plans to run along a roof top and then


æ4ö
-1 æ3ö
-1
(C) sin ç 3 ÷ (D) sin ç 4 ÷ horizontally off it to land on the roof of next
è ø è ø
building. The roof of the next building is 4.9 metres
81. A cannon ball has a range R on a horizontal plane.
below the first one and 6.2 metres away from it.
If h and h' are the greatest heights in the two paths
What should be his minimum roof top speed in m/
for which this is possible, then-
s, so that he can successfully make the jump ?
4h
(A) R = 4 (hh ') (B) R = (A) 3.1 (B) 4.0 (C) 4.9 (D) 6.2
h'
87. A particle is projected horizontally with a speed of
(C) R = 4h h' (D) R = (hh ')
20
82. A ball is thrown from ground level so as to just clear a m/s, from some height at t = 0. At what time
3
wall 4 metres high at a distance of 4 m. and falls at a
will its velocity make 60° angle with the initial
distance of 14 m. from the wall, then the magnitude
velocity
of the velocity of the ball is-
20 m/s
(A) 281 m/sec (B) 812 m/sec 3

(C) 182 m/sec (D) None of the above


83. A particle is projected with velocity u at an angle a (A) 1 sec (B) 2 sec (C) 1.5 sec (D) 2.5 sec
from a point P on a horizontal plane strikes the plane
88. A particle moves on a circular path of radius (r)
at Q. The average velocity of the praticle for its
with speed (v) if its speed and radius both are
journey from P to Q is-
doubled than centripetal force is :–
101
Motion in a Plane
93. A particle moving along a circular path. The angular
(A) same (B) doubled
velocity, linear velocity, angular acceleration and
(C) quadrapuled (D) eight times certripetal acceleration of the particle at any instant
r r r r
89. If the radius of circular path of two particles of respectively are w , v , a are a c. Which of the
same masses are in the ratio of 1 : 2 and have equal following relation is/are correct :–
® ® ® ®
centripetal force their velocities should be in the (a) w ^ v (b) w ^ a
ratio of :– ® ®
(c) v ^ a c
®
(d) w ^ a c
®

(A) 1 : (B) (A) a,b,d (B) b,c,d (C) a,b,c (D) a,c,d
2 2 : 1 (C) 4 : 1 (D) 1 : 4
94. A particle (P) is moving in a circle of radius (a) with
90. The radius of the circular path of a particle is doubled
a uniform speed (v). C is the centre of the circle and
but its frequency of rotation kept constant. If the
AB is a diametre. The angular velocity of particle
initial centripetal force be F, then the final value of when it is at point B about (A) and (C) are in the
centripetal force will be :– ratio :–
F (A) 1 : 1 (B) 1 : 2
(A) F (B) (C) 4F (D) 2F
2
91. A particle of mass 'm' describes a circle of radius (C) 2 : 1 (D) 4 : 1
(r). The centripetal acceleration of the particle is
4
. The momentum of the particle :–
r2

2m 2m 4m 4m
(A) (B) (C) (D)
r r r r
92. Let 'q' denote the angular displacement of a simple
pendulum oscillating in a verticle plane. If the mass of
the bob is (m), then the tension in string is mg cos q :–
(A) always
(B) never
(C) at the extreme positions
(D) at the mean position

102
Motion in a Plane

NUMERICAL VALUE TYPE QUESTIONS


95. Two cars are moving in the same direction with
same speed 30 km/hr. They are separated by a
distance of 5km. What is the speed of a car moving
in the opposite direction if it meets these two cars
at an interval of 4 minutes.
96. A police van moving on a high way with a speed of
30 km/hr. fires a bullet at a thief car which is 99. A particle moves in a straight line from origin.
speeding away in the same direction with a speed Its velocity time curve is shown. Find the distance
of 190 km/hr. If the muzzle speed of the bullet is of particle at t = 8s from the origin and the
150 m/s, find speed of the bullet the theif's car will distance travelled by the particle during first 8
Observe. seconds.
97. Velocity time graph for a particle moving in a line is
given. Calculate the displacement of the particle and
distance travelled in first 4 seconds.

100. A particle starts with an initial velocity 2.5 m/s


along the positive x-axis and it accelerates
uniformly at the rate 0.5 m/s 2 . (a) Find the
98. The velocity time graph of a body moving in a straight distance travelled by it in the first two seconds.
line is as follows. Find the magnitude of displacement (b) How much time does it take to reach the
of body and distance travelled by body during first velocity 7.5 m/s ? (c) How much distance will it
6 seconds. cover before reaching the velocity 7.5 m/s?
JEE MA INS QUESTIONNA IR E A NSWE R KE Y E XE R CISE - III
Que. 1 2 3 4 5 6 7 8 9 10
Ans. A C C A A D A D B D
Que. 11 12 13 14 15 16 17 18 19 20
Ans. D B B D A B C C C C
Que. 21 22 23 24 25 26 27 28 29 30
Ans. B D A D B C C C C C
Que. 31 32 33 34 35 36 37 38 39 40
Ans. C B B A B B A D D D
Que. 41 42 43 44 45 46 47 48 49 50
Ans. B B C B A A B B A B
Que. 51 52 53 54 55 56 57 58 59 60
Ans. B D B B B A D B C B
Que. 61 62 63 64 65 66 67 68 69 70
Ans. C B B A A D D C C B
Que. 71 72 73 74 75 76 77 78 79 80
Ans. C B A C B B A D B A
Que. 81 82 83 84 85 86 87 88 89 90
Ans. A C B A B D B B A D
Que. 91 92 93 94 95 96 97 98 99 100
6.0m
4m, 6m,
Ans. B C D B 45 km/h 380 km/h 0, 4m 10s,
6m 18m
50m
103
Motion in a Plane

PREVIOUS YEAR QUESTIONNAIRE EXERCISE - IV

MHT-CET 7. At the top of the trajectory of a projectile, the


acceleration is-
1. The co-ordinates of a moving particle at any (A) Maximum (B) Minimum
2 2
time t are given by x = ct and y = b t the speed (C) Zero (D) g
of the particle is given by-
8. A ball is projected with velocity 80 m/s and an
(A) 2t [c + b] (B) 2t angle 30° from horizontal. the range will be-
c 2 - b2
(A) 562 m (B) 286 m
(C) t (c 2 + b 2 ) (D) 2t (c 2 + b 2 ) (C) 188 m (D) 1130 m
9. The time of flight of a projectile is 10s and range is
2. The horizontal and verticle distances travelled 500m. Maximum height attained by it is-[g = 10
by a particle in time t are given by x = 6t and m/s²]
2 2
y = 8t – 5t . If g = 10 m/sec , then the initial (A) 25 m (B) 50 m
(C) 82 m (D) 125 m
velocity of the particle is-
10. Range of a projectile is R, when the angle of
(A) 8 m/sec (B) 10 m/sec projection is 30°. Then the value of the other
(C) 5 m/sec (D) Zero angle of projection for the same range, is-

3. If a body 'A' of mass 'M' is thrown with velocity (A) 40° (B) 50°
'v' at an angle of 30° to the horizontal and an- (C) 60° (D) 45°
other body 'B' of the same mass is thrown with 11. The speed of a projectile at its maximum height
the same speed at an angle of 60° to the hori-
3
zontal, the ratio of horizontal range of A to B is times its initial speed. If the range of the
will be- 2
projectile is ‘P’ times the maximum height at-
(A) 1 : 3 (B) 3 :1 tained by it, P =
(C) 1 : 3 (D) 1 : 1 4
(A) (B) 2 3
4. An arrow is shot in air, its time of flight is 5 sec 3
and horizontal range is 200m. The inclination 3
of the arrow with the horizontal is (C) 4 3 (D)
4
–1 5 –1 8
(A) tan (B) tan 12. The angle of projection of projectile, for which
8 5 the horizontal range and the maximum height
–1 1 are equal, is-
(C) tan (D) 45°
8
(A) tan -1( 3 ) (B) tan-1 ( 4)
5. A projectile of mass m is projected from point
A at angle q with the vertical. When the projec- -1æ 1 ö
(C) tan -1( 2 ) (D) tan çç ÷÷
tile passes through the point B in the same hori- è 3ø
zontal plane with A, the magnitude of change
in momentum will be- 13. Four bodies P, Q, R and S are projected with
(A) 2mu sin q (B) 2mu cos q equal velocities having angels of projection 15°,
(C) mu tan q (D) Zero 30°, 45° and 60° with the horizontal respectively.
The body having shortest range is-
6. Two bodies are projected with the same veloc-
ity if one is projected at an angle of 30º and the (A) P (B) Q
other at an angle of 60º to the horizontal, the (C) R (D) S
ratio of the maximum heights reached is- 14. For a projectile the ratio of maximum height
(A) 3 : 1 (B) 1 : 3 reached to the square of flight time is-
–2
(C) 1 : 2 (D) 2 : 1 (g = 10 ms )
(A) 5 : 4 (B) 5 : 2
104
Motion in a Plane
(C) 5 : 1 (D) 10 : 1 [JEE(Main-2019, January]

15. Two projectiles of same mass and with same (1) 30 m (2) 10 m
velocity are thrown at an angle 60° and 30° with
(3) 40 m (4) 20 m
the horizontal, then which will remain same-
(A) Time of flight *19. The position co-ordinates of a particle moving in a
(B) Range of projectile 3-D coordinate system is given by
(C) Max height acquired x = a coswt
(D) All of them
y = a sinwt
and z = awt
JEE-MAIN
The speed of the particle is :
(
*16. A particle moves from the point 2.0iˆ + 4.0 ˆj m, at ) [JEE(Main-2019, January]

( )
t = 0, with aninitial velocity 5.0iˆ + 4.0 ˆj ms–1. It is acted (1) aw (2) 3 aw

upon by a constant force which produces a constant (3) 2 aw (4) 2aw

( )
acceleration 4.0iˆ + 4.0 ˆj ms–2. What is the distance 20. ( )
A particle is moving with a velocity v = K yiˆ + xjˆ ,
where K is a constant. The general equation for its
of the particle from the origin at time 2 s ?
path is : [JEE(Main-2019, January]
[JEE(Main-2019, January]
(1) xy = constant
(1) 20 2 m (2) 10 2 m (2) y2 = x2 + constant
(3) 5 m (4) 15 m (3) y = x2 + constant
*17. A particle starts from the origin at time t = 0 and (4) y2 = x + constant
moves along the positive x-axis. The graph of 21. A body is thrown vertically upwards. Which one of
velocity with respect to time is shown in figure. What the following graphs correctly represent the velocity
is the position of the particle at time vs time? [JEE(Main-2017]
t = 5s ? [JEE(Main-2019, January]

v v
v
(m/s) (1) t (2)
3 t

2
1
v v
1 2 3 4 5 6 7 8 9 10 t (s) (3) t (4) t

(1) 6 m (2) 9 m (3) 3 m (4) 10 m


22. Two stones are thrown up simultaneously from the
*18. A piece of wood of mass 0.03 kg is dropped from
edge of a cliff 240 m high with initial speed of
the top of a 100 m height building. At the same time,
10 m/s and 40 m/s respectively. Which of the
a bullet of mass 0.02 kg is fired vertically upward, with
following graph best represents the time variation
a velocity 100 ms–1, from the ground. The bullet gets
of relative position of the second stone with respect
embedded in the wood. Then the maximum height to
to the first?
which the combined system reaches above the top
of the building before falling below is : (g =10ms–2)
105
Motion in a Plane
(Assume stones do not rebound after hitting the velocity u and the guard's room passes with velocity
ground and neglect air resistance, take v. The middle wagon of the train passes the pole with
g = 10 m/s2) [JEE(Main-2015]
a velocity :- [AIEEE - 2012 (Online)]
(The figure are schematic and not drawn to
æ u 2 + v2 ö 1 2
scale) (1) ç 2 ÷ (2) u + v2
è ø 2
(y 2–y1)m (y 2–y1)m u+v
240 240 (3) uv (4)
2
(1) (2) *26. An object, moving with a speed of 6.25 m/s, is
t(s) t(s)
8 12 8 12 dv
decelerated at a rate given by = –2.5 v
dt
where v is the instantaneous speed. The time taken
(y 2–y1)m (y2–y1)m by the object, to come to rest, would be :-
240 240
[AIEEE - 2011]
(3) (4)
(1) 4 s (2) 8 s
t(s) t(s)
t® 8 12 12
(3) 1 s (4) 2 s
*27. The velocity of a particle is v = v0 + gt + ft2. If its
*23. From a tower of height H, a particle is thrown
position is x = 0 at t = 0, then its displacement after
vertically upwards with a speed u. The time taken
unit time (t = 1) is- [AIEEE - 2007]
by the particle, to hit the ground, is n times that taken
(1) v0 + 2g + 3f (2) v0 + g/2 + f/3
by it to reach the highest point of its path.
(3) v0 + g + f (4) v0 + g/2 + f
The relation between H, u and n is : 28. The co-ordinates of a moving particle at any time t
[JEE(Main-2014] are given by x = at3 and y = bt3. The speed of the
particle at time t is given by- [AIEEE - 2003]
(1) 2g H = nu2(n – 2)
(2) g H = (n – 2)u2 (1) 3t a 2 + b2 (2) 3t2 a 2 + b2

(3) 2g H = n2u2
(3) t2 a 2 + b2 (4) a 2 + b2
(4) g H = (n – 2)2u2
*29. Two guns A and B can fire bullets at speeds
24. The distance travelled by a body moving along a line 1 km/s and 2 km/s respectively. From a point on a
in time t is proportional to t3. horizontal ground, they are fired in all possible
The acceleration-time (a, t) graph for the motion of directions. The ratio of maximum areas covered by
the body will be :- [AIEEE - 2012 (Online)] the bullets fired by the two guns, on the ground is
: [JEE (Main) - 2019, January]
a a (1) 1 : 2 (2) 1 : 4
(1) (2) (3) 1 : 8 (4) 1 : 16

*30. A projectile is given an initial velocity of ( ˆi + 2ˆj ) m/s,


t t

a a where î is along the ground and ĵ is along the


(3) (4) vertical. If g = 10 m/s2, the equation of its trajectory
t t
is : [JEE (Main) - 2013]
*25. A goods train acceleration uniformly on a straight (1) y = x – 5x2 (2) y = 2x – 5x2
railway track, approaches an electric pole standing (3) 4y = 2x – 5x2 (4) 4y = 2x – 25x2
*31. A boy can throw a stone up to a maximum height
on the side of track. Its engine passes the pole with
of 10 m. The maximum horizontal distance that the

106
Motion in a Plane
boy can throw the same stone up to will be :- *35. Two particles A, B are moving on two concentric
[AIEEE - 2012] circles of radii R1 and R2 with equal angular speed
w. At t = 0, their positions and direction of motion
(1) 20 m (2) 20 2 m (3) 10 m (4) 10 2 m
are shown in the figure :
*32. A particle is moving with velocity v = K(y $i + x $j ),
®
[JEE(Main-2019, January]
where K is a constant. The general equation for its
path is : [AIEEE - 2010]
(1) y2 = x2 + constant (2) y = x2 + constant
(3) y2 = x + constant (4) xy = constant
*33. Two cars of masses m1 and m2 are moving in circles
of radii r1 and r2, respectively. Their speeds are such
that they make complete circles in the same time t.
The ratio of their centripetal acceleration
r r p
is : [AIEEE - 2012] The relative velocity uA - uB at t = is given by
2w
(1) 1 : 1
(1) -w ( R1 + R 2 ) ˆi (2) w ( R1 + R 2 ) ˆi
(2) m1 r1 : m2 r2
(3) m1 : m2
(4) r1 : r2 (3) w ( R1 - R 2 ) ˆi (4) w ( R 2 - R1 ) ˆi

*34. A particle is moving along a circular path with a


constant speed of 10 ms–1. What is the magnitude
of the change is velocity of the particle, when it
moves through an angle of 60° around the centre
of the circle? [JEE(Main-2019, January]

(1) zero (2) 10 m/s

(3) 10 3 m/s (4) 10 2 m/s

PREVIOUS YEAR QUESTIONS ANSWER KEY EXERCISE - IV


Que. 1 2 3 4 5 6 7 8 9 10
Ans. D B D A B B D A D C
Que. 11 12 13 14 15 16 17 18 19 20
Ans. C B D A B A B C C B
Que. 21 22 23 24 25 26 27 28 29 30
Ans. A A A D A D B B D B
Que. 31 32 33 34 35
Ans. A A D B D
107

You might also like